civil services mentor january 2012 www.upscportal

131
WWW.UPSCPORTAL.COM 1 UPSCPORTAL Current Affairs : http://upscportal.com/civilservices/current-affairs Integrated Guidance Programme for IAS (Pre) - 2012 http://upscportal.com/civilservices/online-course/integrated-free-guidance-programme http://www.upscportal.com

Upload: lokesh-verma

Post on 18-Apr-2015

224 views

Category:

Documents


0 download

TRANSCRIPT

WWW.UPSCPORTAL.COM 1

UPSCPORTAL Current Affairs : http://upscportal.com/civilservices/current-affairs

Integrated Guidance Programme for IAS (Pre) - 2012http://upscportal.com/civilservices/online-course/integrated-free-guidance-programme

http://www.upscportal.com

WWW.UPSCPORTAL.COM2

UPSCPORTAL Current Affairs : http://upscportal.com/civilservices/current-affairs

Integrated Guidance Programme for IAS (Pre) - 2012http://upscportal.com/civilservices/online-course/integrated-free-guidance-programme

LOKPAL BILLA DRAMA UNFOLD..

INDIA-JAPANSTARTING ACTIVE PARTNERSHIP

INDEXINDEX

PAGE NO: 3

PAGE NO: 8

Civil ServicesMentor

January: 2012

Chief Editor:Sachchida Nanda Jha

Editor:Yagya

Designed by:Chandan Kumar011 – 45151781

Disclaimer:

For AdvertisementContact at: 011–45151781

Editor and Publisher are notresponsible for any views,data, figures etc. Expressedin the articles by theauthor(s). Maps are nota-tional.

» National Issues 1 5

» International Issues 2 4

» Economy Issues 3 1

» India & The World 4 7

» Science & Technology 5 3

» Sports 6 7

» Awards & Prizes 7 2

» In the News 7 7

» Special Study Material:SCRA: Exam Practice Paper 9 3

WWW.UPSCPORTAL.COM 3

UPSCPORTAL Current Affairs : http://upscportal.com/civilservices/current-affairs

Integrated Guidance Programme for IAS (Pre) - 2012http://upscportal.com/civilservices/online-course/integrated-free-guidance-programme

The Jan Lokpal Bill campaign beganin January 2011 in the backdrop ofthe publicity that accompanied theseveral mega-scams that surfacedin 2010, notably those relating to theCommonwealth Games and thetelecom spectrum allocations. Itcaught the public imagination withAnna Hazare's fast at Jantar Mantarin New Delhi in April 2011. Thatforced the UPA government to con-stitute a joint drafting committee fora Lokpal bill. The civil society repre-sentatives in the committee pro-posed a bill called the Jan Lokpalbill, which became the basis for dis-cussions. The basic principles onwhich the bill was drafted wereculled from the United Nations Con-vention against Corruption, whichrequired all countries to put in placeanti-corruption investigative agen-cies that would be independent ofthe executive government and wouldhave the jurisdiction to investigateall public servants for corruption.The Lok Sabha passed a historicLokpal and Lokayuktas Bill, 2011, butdefeated a parallel Constitution(116th Amendment) Bill, which

would have given the Lokpal consti-tutional status. On December 22, thegovernment introduced the Billalong with the Constitution (116thAmendment) Bill in the Lok Sabha.However the UPA government didn’tput it to vote in the Upper House.This Bill incorporated some of therecommendations of the Parliamen-tary Standing Committee on Person-nel, Public Grievances, Law and Jus-tice. This was a completely new Billas the government had withdrawnits previous Lokpal Bill, which it in-troduced in the Lok Sabha on August4, 2011, and later referred to theStanding Committee. The StandingCommittee tabled its report on De-cember 9, 2011. The August Bill didnot propose to confer constitutionalstatus on the Lokpal. The StandingCommittee recommended constitu-tional status so that the Lokpal hadhigher stature and increased legiti-macy. The committee believed thatconstitutional status would enhancethe legal and moral authority of theLokpal institution and also insulatethe basic principles of the Lokpalfrom the vicissitudes of ordinary ortransient majorities.

Yet, the Lok Sabha, which passed theLokpal and Lokayuktas Bill with afew amendments, rejected the Con-stitution (116th Amendment) Bill,which required two-thirds majorityof the House present and voting forits passage. The object of the Con-stitution Amendment Bill is laudable

as it seeks to create an autonomousand independent Lokpal at the Cen-tre and Lokayuktas in the States withpowers of superintendence and di-rection over investigation and pros-ecution of public servants accusedof corruption. Yet, it failed to securethe requisite support in the LokSabha because members found ahuge gap between its object and theprovisions of the Lokpal andLokayuktas Bill.

During the debate in Parliament, theopposition was critical of thegovernment's control over the selec-tion and removal of members of theLokpal. But the Standing Commit-tee report shows that it wanted todilute the provisions with regard toselection and removal in the Bill thatwas introduced in August. That Billhad proposed a nine-member selec-tion committee, five of whom wouldhave been government nominees.The Standing Committee recom-mended a four-member selectioncommittee comprising the PrimeMinister, the Speaker of the LokSabha, the Chief Justice of India(CJI), an eminent Indian nominatedunanimously by the Comptroller andAuditor General (CAG), the ChiefElection Commissioner (CEC) andthe Chairman of the Union PublicService Commission (UPSC), and theLeader of the Opposition in the LokSabha. The December Bill proposesfive members, of whom three shouldbe government nominees – the

LOKPAL BILLA Drama Unfold

WWW.UPSCPORTAL.COM4

UPSCPORTAL Current Affairs : http://upscportal.com/civilservices/current-affairs

Integrated Guidance Programme for IAS (Pre) - 2012http://upscportal.com/civilservices/online-course/integrated-free-guidance-programme

Prime Minister, the Speaker, theLeader of the Opposition in the LokSabha – and the CJI or a judge of theSupreme Court nominated by the CJI,and one eminent jurist nominated bythe President. Had the governmentaccepted the Standing Committee'srecommendation, it could havedented somewhat the opposition'scriticism.

Though the Bill kept the CBI with thegovernment, it allowed the Lokpal tohave its own anti-corruption inves-tigative body. It eliminated the needto get prior sanction for investiga-tion from the government. It providedfor the confiscation of the assets ofcorrupt public servants and the re-covery of losses caused by their actsof corruption from them. But it cre-ated a terribly cumbersome proce-dure for investigation, by which apreliminary inquiry and hearing ofthe corrupt public servant were madecompulsory before investigationcould begin. This ended the possi-bility of making surprise raids andseizures on the premises of corruptpublic servants or their abettors. TheBill of August provided that theLokpal would have its own investi-gation and prosecution wings. TheStanding Committee, however,sought to dilute this by recommend-ing instead that the Lokpal conducta preliminary inquiry, after which theCentral Bureau of Investigation(CBI) would investigate. Also, theCBI would have autonomy over itsinvestigation.

The committee also proposed thatthe Lokpal will have a supervisoryrole over the CBI in cases relatingto Group A and B officers. The Billof December further diluted theserecommendations. The Lokpal, it

says, shall refer a preliminary in-quiry against Group A, B, C and Demployees to the Central VigilanceCommission (CVC). The Bill furthersays that after conducting the in-quiry, the CVC shall submit a reportto the Lokpal in the case of Group Aand B employees and proceed ac-cording to specified procedure in thecase of Group C and D staff. TheCVC, according to the current Bill,shall send periodic reports to theLokpal on its cases.

The Rajya Sabha witnessed a sordiddrama. Several parties which hadwalked out in the Lok Sabha (theSamajwadi Party and the BahujanSamaj Party) or had not moved anyamendments there (the TrinamoolCongress) moved amendments inthe Rajya Sabha and their represen-tatives delivered fiery speeches op-posing the provisions of the Bill.When it became clear that at leastthree of the amendments (those re-lating to the selection and removalof Lokpal members, the CBI beingbrought under the administrativecontrol of the Lokpal, and the dele-tion of the chapter on Lokayuktas inthe States) were likely to be passed,the government engineered distur-bances in the House, resorted to fili-bustering and prevented the amend-ments from being voted upon. Andthe House was prorogued with theBill hanging in the air.The drama inthe Rajya Sabha showed that thegovernment was not even willing togo by the will of Parliament. Thisgives rise to fundamental questionsabout the functioning of Indian de-mocracy.The Bill adds that if a prima faciecase exists against a public servant,the Lokpal may refer it to the CBI forinvestigation. Also, it may refer a

case for preliminary inquiry to theCBI (other than Group A, B, C and Dofficers). The Bill also provides thatthe Lokpal shall exercise generalsuperintendence over the CBI (simi-lar to the CVC's supervision cur-rently). These additional dilutions inthe later Bill, according to critics,reduce the Lokpal to just a post of-fice.

The committee recommended thatthe Lokpal conduct only the prelimi-nary inquiry and that it be authorisedto initiate it suo motu. In such cases,the inquiry would have to be doneby a five-member Lokpal Bench thatis not connected with the suo motuinitiation. More important, the ac-cused would not get an opportunityto be heard at this stage, though theBill of August allowed that. The laterBill rejects both these recommenda-tions and sticks to the August ver-sion, which provided that the Lokpalcould initiate an inquiry only on thebasis of a complaint by a citizen. Theonly concession the Bill makes is thatthe Lokpal shall have its own inquirywing to conduct a preliminary in-quiry on a complaint it has receivedand has decided can be inquiredinto.

Under Article 252 or 253

Some parties questioned that lokpalbill should be brought under article253 & not under article 252.Article 252: Article 252. Power ofParliament to legislate for two ormore States by consent and adop-tion of such legislation by any otherState(1) If it appears to the Legislaturesof two or more States to be desir-able that any of the matters withrespect to which Parliament has no

WWW.UPSCPORTAL.COM 5

UPSCPORTAL Current Affairs : http://upscportal.com/civilservices/current-affairs

Integrated Guidance Programme for IAS (Pre) - 2012http://upscportal.com/civilservices/online-course/integrated-free-guidance-programme

power to make laws for the Statesexcept as provided in Articles 249and 250 should be regulated in suchStates by Parliament by law, and ifresolutions to that effect are passedby all the House of the Legislaturesof those States, it shall be lawful forParliament to pass an Act for regu-lating that matter accordingly, andany Act so passed shall apply tosuch States and to any other Stateby which it is adopted afterwards byresolution passed in that behalf bythe House or, where there are twoHouses, by each of the Houses ofthe Legislature of that State.(2) Any Act so passed by Parliamentmay be amended or repealed by anAct of Parliament passed or adoptedin like manner but shall not, as re-spects any State to which it applies,be amended or repealed by an Actof the Legislature of that State

Article 253: Article 253. Legislationfor giving effect to internationalagreements Notwithstanding any-thing in the foregoing provisions ofthis Chapter, Parliament has powerto make any law for the whole or anypart of the territory of India forimplementing any treaty, agreementor convention with any other coun-try or countries or any decision madeat any international conference, as-sociation or other body.

Relation between Union and StatesThe relation between Union andStates is the very bedrock of the In-dian Federal system. India is a fed-eral State with a national govern-ment and a government of each con-stituent state. Although the structureof India is federal in a general way,yet there are certain aspects that areunique to federalism as practiced inIndia. The Indian government fol-

lows a strong central bias.Some of the special features of In-dia are as follows:Agreement between the States: Iftwo states agree that the Parliamentcan legally make laws with respectto the two states, then the Parlia-ment can make laws relating to anylaw or set of laws related to the StateLaws. This is an extension of the Par-liamentary legislative as laid downby the Article 252 of the Indian Con-stitution.

Implementation of Treaties: TheParliament makes laws for theimplementation of treaties, even ifthe subject falls under the legisla-tive power of the State, for the big-ger international interest of thecountry. This power has been givento the Union by the Article 253.

Suggested Amendments: TeamAnna had suggested 34 amendmentsto rectify the government's Bill, andpointed out that four of these werecritical to making the Lokpal a work-able institution. These were that theselection and removal procedureshould be made independent of thegovernment; the CBI should bebrought under the Lokpal's admin-istrative control or, alternatively, theLokpal should have its own investi-gative body; all government ser-vants should be brought under theLokpal's investigative ambit; and theprocedure for investigation shouldbe in line with the normal criminalinvestigation procedure. But the gov-ernment was adamant in not ac-cepting any of these either, andwent on to bulldoze the passage ofits Bill. It rejected all the amend-ments moved by the Opposition inthe Lower House.

Notices for 187 Amend-ments in Rajya Sabha

Notice had been given for as manyas 187 amendments to the Lokpaland Lokayukta Bill, 2011 includingfrom the Trinamool Congress fordeletion of the entire Part III of theBill regarding establishment of theLokayuktas. The Biju Janata Dal andthe Shiromani Akali Dal too had givensimilar notices. By afternoon, theRajya Sabha had received 173amendment notices. The numberswelled to 187 by evening as an-nounced by the Minister of State forPersonnel V. Narayanasamy. Nor-mally notices are taken 24 hours inadvance but on Thursday, noticeswere accepted till late evening, whilethe government took the stand in theHouse that there were too manyamendments which “needed consid-eration.”

Among major amendments were theones given by the Bharatiya JanataParty for deletion of words “not lessthan” 50 per cent of members ofLokpal belonging to ScheduledCastes, Scheduled tribes, OtherBackward Classes, Minorities andWomen in Clause 3. They also askedfor deletion of the word ‘Minorities'in Clause 3 and inclusion of Leaderof Opposition in Rajya Sabha on Se-lection Committee in Clause 4. Theyalso wanted Lokpal to appoint hissecretary and not select from apanel given by the government.

The Communist Party of India (Marx-ist) gave amendment for replace-ment of eminent jurist on the Selec-tion Committee with “an eminentperson to be nominated by the ChiefElection Commissioner, Comptrollerand Auditor General and Union Pub-

WWW.UPSCPORTAL.COM6

UPSCPORTAL Current Affairs : http://upscportal.com/civilservices/current-affairs

Integrated Guidance Programme for IAS (Pre) - 2012http://upscportal.com/civilservices/online-course/integrated-free-guidance-programme

lic Service Commission. They alsowanted the Inquiry Wing under theLokpal to be replaced by “Investiga-tion Wing” with the Central govern-ment providing officers and staffunder exclusive jurisdiction of theLokpal as decided in consultationwith the Lokpal. The Party gave an-other amendment for inclusion un-der the Lokpal of “any corporatebody, its promoters, its officers in-cluding Director against whom thereis a complaint of corruption in rela-tion to grant of government licence,lease, contract, agreement or anyother action to influence govern-ment policy through corrupt means.”They wanted Lokpal to take suomotu action in a compliant of cor-ruption.

Other Angle

Passing of “Lokpal” Bill seems quitean impossible task,because everyparty to it has its interest.The UPAgovernment summarily “check-mated” the social activists anddrove them into existential crisiswhile simultaneously reducing theLokpal bill to a maere a game tool.At the moment, the activists will behappier with the “no bill at all” thanthe current version of the bill tabledby the government. None of the po-litical parties wants the Lokpal bill.Even if individual politicians want it,they can’t do a thing about it be-cause their “High Command” doesnot want it. But the country’s futureis at stake. The politicians are outto maintain the status-quo in orderto protect their corrupt practices andill-gotten wealth. But Team Annahas accepted this mission and is notgoing to relent until their goals aremet. Team Anna rallied the massesand raised hopes of a utopian tomor-

row - one with a clean and indepen-dent CBI under an incorruptibleLokpal. But after an entire year ofwrangling with the government.

Meanwhile, the elections in UP areround the corner, and the Congresshas been expanding its reach withsome crafty electoral mathematics,a specialization offered at theAllahabad University. Ajit Singh isalready in the bag. He was tradedthe lucrative civil aviation ministryin return for joining the Congresselectoral alliance in UP. Likewise, theRailway Ministry is being “dangled”in front of Mulayam Singh Yadav ofthe SP. Doing well in the UP electionswill surely serve as a shot in the armfor the Congress. First, it mightcause a mental breakdown forMayawati, especially after the elec-tion commission decided to coverher statues across the state. Sec-ondly, it sets the stage for RahulGandhi’s coronation as the PM. Be-sides, it will also give the Congressgreater leverage against Didi.

On the other side of this pitchedbattle is the opportunistic BJP, whichrecently threw open its gates forMayawati discards in UP. In contrast,the same BJP passed a strongLokayutkha bill in Uttarkhand. TheBJP’s goal is simple - stall the bill,harp on the government’s incompe-tence, and cash in on the general dis-content in the country, in hope ofriding an anti-corruption wave to getback in power. The battle lines aredrawn. On one side is the Congressparty and its fickle, calculative al-lies. The leading trouble-makeramong the allies is the TMC. TheCongress needs her now and sheknows that. Her primary long-termgoal is to put the CPI(M) out of busi-

ness in W. Bengal. But at present,everything she does is to extract themaximum from the Congress.

Conclusion

India’s politicians again stand dis-credited in the eyes of people, whoare asking questions to which thepoliticians, both the ruling party andthe opposition, do not have an an-swer. If we truly want to make Indiacorruption free we need to look atthe new generation. Yes the govern-ment can help — first by cleaningup their administration and then bypunishing corrupt people so severelythat punishment becomes a deter-rent to corruption. The governmentcan support and reward honesty —reward people who pay taxes in-stead of harassing them and go af-ter the people who do not pay taxesand yet lead opulent lives. Reward-ing honesty and penalising dishon-esty will create an ecosystem thatsupports a life of integrity. Let thehonest man be privileged and ac-knowledged so that people aspire tobe honest.

We need to target schools and col-leges; motivate the youngsters totake charge so that they can startchanging the world around them. Wehave to stop paying the cop for traf-fic offences, and we have to havethe will to fight each time a bribe isasked of us. The people of India needthe media now — the media thatcatapulted the movement to fameneeds to open an action line whereevery act of corruption can be re-ported and exposed. Because thisbattle can be won, this change canbe made and we do not need regu-lation to do so.

WWW.UPSCPORTAL.COM 7

UPSCPORTAL Current Affairs : http://upscportal.com/civilservices/current-affairs

Integrated Guidance Programme for IAS (Pre) - 2012http://upscportal.com/civilservices/online-course/integrated-free-guidance-programme

Advertise yourBusiness Here

Contact Us Onlinehttp://upscportal.com/store/contact

WWW.UPSCPORTAL.COM8

UPSCPORTAL Current Affairs : http://upscportal.com/civilservices/current-affairs

Integrated Guidance Programme for IAS (Pre) - 2012http://upscportal.com/civilservices/online-course/integrated-free-guidance-programme

India-JapanStarting Active Partnership

The determined effort by India andJapan to inject life into their bilat-eral relationship is showing positiveresults. The meeting between PrimeMinister Manmohan Singh and Japa-nese premier Yoshihiko Noda, thesixth of the annual India-Japan sum-mits, is a marker of the steadyprogress. It yielded the first officialconfirmation since Japan's devastat-ing tsunami-earthquake-Fukushimameltdown that the country has notshut the door on a civilian nucleardeal with India. While Mr. Nodastressed the importance of learningthe right lessons from Japan'snuclear accident, it appears thatnegotiations on a deal to assist In-dia develop peaceful nuclear energywill continue.

His pledge of $4.5 billion over thenext five years for the developmentof the Delhi-Mumbai Industrial Cor-ridor, a commitment of financial as-sistance for two more infrastructureprojects — Phase 3 of the DelhiMetro and a biodiversity conserva-tion project in West Bengal — andhis interest in sharing high-speedrailway technology with India are sig-nificant. Japan's enhancement of itscurrency swap arrangement from $3

billion to $15 billion will help Indiabetter manage the rupee's slumpagainst the dollar. This should alsoboost financial cooperation and eco-nomic and trade ties, which are cur-rently far below potential. The In-dian side wants more Japanese in-vestment. For its part, Japan, whichhas just lifted a longstanding ban onthe export of weapons, will be look-ing to sell defence hardware to NewDelhi.

Prime Minister Noda reiterated hisgratitude for the substantial supportextended from India in the wake ofthe Great East Japan Earthquake.Prime Minister Singh expressed hishope that Japan will make a full re-covery as early as possible.The twoPrime Ministers concurred that, asnext year marks the 60th anniversaryof the establishment of diplomaticrelations between Japan and India,they would make best use of theopportunity to strengthen culturaland people-to-people exchanges inorder to deepen mutual understand-ing at their citizens' level.

Prime Minister Noda's visit cappeda year of intense bilateral activity.The Foreign Ministers held a strate-gic dialogue in October, followed bytalks between the defence minis-ters. In February, the two sidessigned the Comprehensive EconomicPartnership Agreement coveringtrade in goods as well as services;it came into effect in August. Earlier

this month, India, Japan, and theUnited States held their first trilat-eral strategic dialogue. It is credit-able that despite the political insta-bility in Japan, and the scandal-in-duced paralysis in India, both coun-tries managed to fit in these high-level exchanges. It is crucial thatgrowing India-Japan ties are viewedindependently of each country's re-lations with China. New Delhi andBeijing are engaged in improvingrelations at various levels, includingtrade, eventually aiming to amica-bly resolve the boundary question.China is Japan's biggest trading part-ner; the two have a bilateral tradeof $340 billion that neither can af-ford to endanger. The fears ex-pressed in a section of China's newsmedia that Mr. Noda's visit is partof a “containment strategy” aimedat Beijing are needless. The Chinesegovernment has done well to playthem down.

History

Exchange between Japan and Indiais said to have begun in the 6th cen-tury when Buddhism was introducedto Japan. Indian culture, filteredthrough Buddhism, has had a greatimpact on Japanese culture, and thisis the source of the Japanesepeople's sense of closeness to In-dia. After World War II, in 1949, In-dian Prime Minister JawaharlalNehru donated two Indian elephantsto the Ueno Zoo in Tokyo. This

WWW.UPSCPORTAL.COM 9

UPSCPORTAL Current Affairs : http://upscportal.com/civilservices/current-affairs

Integrated Guidance Programme for IAS (Pre) - 2012http://upscportal.com/civilservices/online-course/integrated-free-guidance-programme

brought a ray of light into the livesof the Japanese people who still hadnot recovered from defeat in the war.Japan and India signed a peacetreaty and established diplomaticrelations on 28th April, 1952. Thistreaty was one of the first peacetreaties Japan signed after the WorldWar II. Ever since the establishmentof diplomatic relations, the twocountries have enjoyed cordial rela-tions.

Bilateral Treaties andAgreements

» Treaty of Peace (1952)» Agreement for Air Service (1956)» Cultural Agreement (1957)» Agreement of Commerce (1958)» Convention for the Avoidance ofDouble Taxation (1960)» Agreement on Cooperation in thefield of Science and Technology(1985)

In the post World War II period,India's iron ore helped a great dealJapan's recovery from thedevastation. Following JapanesePrime Minister Nobusuke Kishi's visitto India in 1957, Japan startedproviding yen loans to India in 1958,as the first yen loan aid extendedby Japanese government. Since1986, Japan has become India'slargest aid donor, and remains so.Prime Minister Mori's visit to Indiain August 2000 provided themomentum to strengthen the Japan-India relationship. Since then, annualmeetings between prime ministers,including Prime Minister Koizumi'svisit to India in April 2005, PrimeMinister Singh's visit to Japan inDecember 2006, Prime MinisterAbe's visit to India in August 2007,

and Prime Minister Singh's visit toJapan in October 2008, have led tothe establishment of the "Strategicand Global Partnership betweenJapan and India" as well as thefurther strengthening of the bilateralrelationship. Most recently, duringPrime Minister Hatoyama's visit toIndia in December 2009, Japan andIndia singed the Joint Statement"New Stage of Japan-India Strategicand Global Partnership".

While India has become the largestrecipient of Japanese ODA since2003-04, both bilateral trade ingoods and Japan’s FDI into Indiahave almost stagnated during thepast decade. With major Indianexports to Japan being dominatedby products of mineral oils, gems &jewels, marine products (mainlyshrimps) and iron ore, India andJapan have not yet formed such adynamic international division oflabour as can be seen among theEast Asian countries. Japan’scumulative FDI into India is so farlarger than that of Korea, but as faras electronics appliances areconcerned, Japanese companieshave been overwhelmed by Koreancompanies in the Indian market.

India and Japan are highlycomplementary economies, but theirmutual complementarities remainedunderexploited hitherto. Recently,however, a new tide has beenobserved in Japan to expand Indo-Japan relations based on mutualcomplementarities. Japan hasabundant capital and is highlyadvanced in technological skills andproduct development while India isendowed with a huge market andabundant human resources. Japan isoutstanding in terms of

manufacturing abilities, whereasIndia has advantages in IT servicesand bioinformatics. Thanks tovigorous economic growth in thewake of the economic reforms andthe “Look East Policy” introducedafter 1991, India has successfullydeepened its economic relationswith the East Asian countries. Wecan see a remarkable expansion ofIndia’s economic tie-ups with theASEAN countries, China and Koreasince the late 1990s, with onlyIndoJapan relations lagging behind.India-China bilateral trade hasremarkably expanded to be morethan four times larger than India-Japan bilateral trade.

Koizumi’s Visit to Indiain 2005

The significance of visit of JapanesePrime Minister Junichiro Koizumi in2005 cannot be understatedconsidering the fact that this was thefirst visit by a Japanese head of stateafter a hiatus of nearly half adecade. Prior to that, only twoserving Japanese heads of statehave visited India with a 10 year gap.While these statistics amplydemonstrate that a regular exchangeof high-level political visits havebeen a weak link in Indo-Japaneseties, perhaps Prime MinisterKoizumi’s visit is a landmark one asit promises to initiate bilateralcomprehensive ventures into twoareas - economic and strategic,which were lying potentiallyuntapped for several years. Thehighlight of the Prime Minister’s visitwas indubitably the signing of the‘Japan-India Partnership in a NewAsian Era: Strategic Orientation of aJapan-India Global Partnership’ — aneight-fold initiative announced

WWW.UPSCPORTAL.COM10

UPSCPORTAL Current Affairs : http://upscportal.com/civilservices/current-affairs

Integrated Guidance Programme for IAS (Pre) - 2012http://upscportal.com/civilservices/online-course/integrated-free-guidance-programme

towards strengthening Japan-IndiaGlobal Partnership, which includethe following:» Enhanced and upgraded dialoguearchitecture, including strengtheningof the momentum of high-levelexchanges, launching of a High LevelStrategic Dialogue and fullutilization of the existing dialoguemechanisms.» Comprehensive economicengagement, through expansion oftrade in goods and services,investment flows and other areas ofeconomic cooperation, andexploration of a Japan-Indiaeconomic partnership agreement.» Enhanced security dialogue andcooperation.» Science & Technology Initiative» Cultural an academic initiativesand strengthening of people-to-people contacts» Cooperation in ushering a newAsian era» Cooperation in the UN and otherinternational organization, includingearly reform of the UN SecurityCouncil» Cooperation in responding toglobal challenges and opportunities.

While this eight-fold initiativeincorporates many issues addressedby the two countries in their JointDeclaration of December 2001, it is

certainly a more comprehensivedelineation and framework of areasof common interests and concerns.In a nutshell, it can be stated thatthere is a unique paradox that hasdefined India-Japan relations atlarge. On the one hand, there hasbeen the absence of any majordispute. On the other, however, whathas been missing is the richness thatcould have characterized bilateralties. In other words, the relationshipto date can be chronicled as one ofmissed opportunities. It is clear thatthe synergy of Indo-Japanesecooperation is essential not just tofurther bilateral relations, but for thegreater cause of a more secure Asia-Pacific region. Even though the visitby Prime Minister Koizumi was short,it should be used as bedrock forfurthering bilateral ties.

An increased awareness among thegeneral populace of the two nationsas envisaged under the eight-foldinitiative is equally crucial as theextent of media coverage receivedby such landmark steps as Koizumi’svisit and need to be highlighted.While India’s embarkation on a ‘LookEast’ policy in the last decade wassymptomatic of the significanceIndia attaches to the region, the timehas come to exploit the full potentialof interaction with countries likeJapan. The two countries shouldwork together in the areas of

common interests and concerns inthe years to come so that they canlook back at their relations with prideand satisfaction at the end of thenext five decades, when wecelebrate a hundred years of theestablishment of relations.Bilateral trade is expanding in therecent years. However, the speedand scope of expansion are stilllimited. Joint Study Group (JSG),composed of government officialsand representatives of business andacademia from the two countries,held four meetings after July, 2005.JSG submitted its report to bothPrime Ministers when they met inJuly, 2006, which includes arecommendation for launching EPAnegotiations. In December, 2006, thePrime Ministers of the two countriesdecided to launch immediatenegotiations for the conclusion of abilateral Economic PartnershipA g re e m e n t / C o m p r e h e n s i v eEconomic Partnership Agreementaiming to complete in substance assoon as possible in approximatelytwo years. Thirteen rounds ofnegotiations were held in New Delhiand Tokyo respectively as of July2010.

Japan-India Strategic Dialogue onEconomic Issues which reviews thecurrent status of bilateral economicissues discussed at Summitmeetings, and undertakes

Buy Books Online For CSAT, IAS Mains, Bank, SSC, CDS & Other Exam Preparation.CLICK BELOW LINK TO ORDER

http://upscportal.com/civilservices/order-books

WWW.UPSCPORTAL.COM 11

UPSCPORTAL Current Affairs : http://upscportal.com/civilservices/current-affairs

Integrated Guidance Programme for IAS (Pre) - 2012http://upscportal.com/civilservices/online-course/integrated-free-guidance-programme

Japanese private-sector’s interest in India is rising, and, currently, about627 Japanese companies have branches in India. (The figure doubled overthe last 3 years.)

coordination as necessary was held 3 times in New Delhi and Tokyorespectively as of July 2010 since the Dialogue was launched in July 2007.In August 2007, the Business Leaders' Forum was held in New Delhi on theoccasion of Prime Minister Abe's visit to India, and the Second meetingwas held in Tokyo in October 2008.

The two Prime Ministers welcomed the entry into force of the Japan-IndiaComprehensive Economic Partnership Agreement (CEPA) in August thisyear and the commencement of negotiations on an agreement on socialsecurity. With regard to the Delhi-Mumbai Industrial Corridor (DMIC), PrimeMinister Noda announced the intention of the Government of Japan tomake available finance totaling 4.5 billion US dollars in the next five years.The two Prime Ministers concurred that they would promote participationof Japanese companies to realize the DMIC.In this regard, both Governments reached an agreement on such issues asJapan's involvement with the Delhi-Mumbai Industrial CorridorDevelopment Corporation (DMICDC), relaxation of financial restrictionsso as to promote DMIC projects, allocation of natural gas for DMIC projects,and early realization of three model projects, namely, seawater desalinationat Dahej, Gujarat, a microgrid system using large-scale photovoltaic (PV)power generation Project at Neemrana Industrial Area in Rajasthan, andgas-fired independent power producer (IPP) Project in Maharashtra. Thetwo Prime Ministers concurred on the need to strengthen bilateralcooperation on infrastructure development in Southern India, improvementof connectivity between Chennai and Bangalore, and development of India'sHigh-speed railway system. Prime Minister Noda pledged that theGovernment of Japan would extend ODA loans totaling approximately 1.7billion US dollars (approx. 134.3 billion yen) in total for two projectsincluding for the Delhi Mass Rapid Transport System Project. The two PrimeMinisters decided to enhance the earlier bilateral currency swaparrangement from 3 billion US dollars to 15 billion US dollars. Concerningcivil nuclear cooperation, Prime Minister Singh noted that he understoodJapan's sensitivity well. The two Prime Ministers decided to move forward

with the negotiations with a view toconcluding an Agreement forCooperation in the Peaceful Uses ofNuclear Energy in a mutuallysatisfactory manner. With regard torare earths, the two Prime Ministersshared the view that bothGovernments should continue theirdialogue in order for Japanese andIndian enterprises to jointlyundertake industrial activities.The long-awaited India-Japan CEPA/EPA was finally signed in February2011, and came into effect inAugust. Tariffs will be eliminated on90 per cent of Indian and 97 per centof Japanese goods over ten years.Japan is expected to export moreauto-parts and steel products toIndia and import more agriculturaland marine products from India.National treatment, on the side ofJapan, is accorded for theapplication of approvals for therelease of generic medicines. Someimprovement has been made for themovement of people, including Yogainstructors, English teachers andIndian cuisine masters. IndiaJapanbilateral trade will be more thandoubled to $25 billion by 2014 underthe CEPA/EPA.

What is CEPA ?

The second phase of India’s LookEast Policy aims to achieve just that.Against this background, India-Japan ties are appropriatelypositioned for take-off. Thesignificance of Prime MinisterSingh’s visit to Japan can be seen infour areas: conclusion of four yearsof negotiations on a ComprehensiveEconomic Partnership Agreement(CEPA), deciding to speed upnegotiations on a civil nuclear deal,simplifying visa procedures, and

WWW.UPSCPORTAL.COM12

UPSCPORTAL Current Affairs : http://upscportal.com/civilservices/current-affairs

Integrated Guidance Programme for IAS (Pre) - 2012http://upscportal.com/civilservices/online-course/integrated-free-guidance-programme

finding common ground forconvergence of interests in securingpeace and stability in Asia. CEPA isbroader than a free trade agreementbecause it includes steps to promotegreater investment and alsoaddresses intellectual propertyrights. The CEPA will help addressthis imbalance and Japan will haveto reduce tariffs on 97 per cent ofIndian imports, and India will haveto reduce tariffs on 90 per cent ofgoods imported from Japan over thenext decade. Japan will get greatermarket access for most industrialgoods, as well as severalagricultural products such as durian,curry, tea leaves, lumber, shrimp andshrimp products. India will getimproved market access in Japan inauto parts, steel panels, and otherindustrial materials, as well as DVDplayers, video cameras andindustrial machinery.

The two countries began talks on acivilian nuclear deal in June 2010 butthe subject is too sensitive in Japanbecause India has refused to adhereto the NPT and CTBT as it considersthem discriminatory, even thoughIndia has voluntarily suspendednuclear testing. In late 2009, PrimeMinister Singh said that India’sposition on the CTBT could changeif Beijing and Washington were tojoin the pact. China, India and theUS are three of the nine “Annex 2”states that must ratify the treaty forit to enter into force. Japan thusagreed to start negotiation withIndia on a framework for thepeaceful transfer of nuclear powertechnologies to India. PrimeMinister Kan agreed to speed upnegotiations, while seeking India’sunderstanding for Japan’ssentiments. Both leaders agreed

that civil nuclear energy can be amutually beneficial area forcooperation. India welcomesJapanese firms to participate in theexpansion of its nuclear industry forpeaceful purposes.

Until recently, Japanese companieshad a negative image about India’sinvestment environment, whichconstituted a sense of“psychological distance” from India.However, the success of Koreancompanies in gaining a large shareof the Indian consumer electronicsmarket indicates that what Japanesecompanies tended to identify asareas of concern, i.e. adverseinvestment environment, are notnecessarily critical factors thatdeterred Korean or Singaporeancompanies. The number of Japanesecompanies in India has increasedfrom 267 in January 2006 to 438 inJanuary 2008 and from 550 inOctober 2008 to 725 in October2010. Their investment value hasmore than doubled to 189 billion yenin 2007-08 and totalled 809 billionyen in 2008-09, outstrippingJapanese FDI into China.

Japan is globally ranked the secondlargest IT market with an estimatedturnover of $100 billion. Japan isrenowned as one of the mostcompetitive nations inmanufacturing industries orhardware production. As of 2007,Japan had a total of 907,990 ITengineers and was short of 501,000IT engineers including 360,000professionals. Generally speaking,Japanese companies are inclinedtowards in-house procurement andare rather cautious about overseasIT offshoring. But when theyoffshore, they prefer to turn to China

, As for the destination of Japan’s IToffshoring, currently 80 per cent ofJapanese companies offshore toChina and only 25 per cent to India.For India, Japan accounts for onlyless than 2 per cent of its totalsoftware and IT services exports. Asfor Japanese subsidiaries of IBM andAccenture, they are already fullyutilizing Indian IT resources throughoffshoring to their Indiansubsidiaries. It will be strategicallymore important for Japanesecompanies to focus on India as anIT offshore location, especially ifthey want to go global.

The level of people-to-peopleexchange between India and Japanis still far from satisfactory, whichis exemplified by stagnated studentexchanges. It should be rememberedthat there are as many as threemillion Indian Americans, most ofwhom are professionals, playing anindispensable role in deepening andcementing the relationship betweenIndia and USA. Improving thescholarship and Japanese languagetraining schemes for Indianstudents, increasing the number oflectures given in English at Japaneseuniversities, and providing attractivecareer paths for Indian graduates toapply for Prospects and Challengesfor Expanding India-Japan EconomicRelations enlarged employmentopportunities in Japanesecompanies are urgently needed forattracting more Indian students toJapan. Now is a good time, whenentering a new phase of India-Japanrelations, for us to consolidate ourrelations especially throughaccelerated people-to-peopleexchanges. Number of Japanesenationals residing in India: 3,284 (asof October, 2008)

WWW.UPSCPORTAL.COM 13

UPSCPORTAL Current Affairs : http://upscportal.com/civilservices/current-affairs

Integrated Guidance Programme for IAS (Pre) - 2012http://upscportal.com/civilservices/online-course/integrated-free-guidance-programme

Number of Indian nationals residingin Japan: 22,335 (as of December,2008).

The Japanese economy has beenstrained in the aftermath of themassive earthquake and tsunami,accompanied by radiation leak atthe Fukushima nuclear power plant.We cannot ignore the after effectsof the Great East Japan Earthquakeon India-Japan economic relations.F irst, the number of Indian ITengineers working in Japan has beenreduced from 15,000 to 7,000, sincemany Indian IT engineers have notyet returned to Japan. Further, sincedomestic supply chains have beendamaged by the earthquake andgiven the record appreciation of theyen, Japanese companies are undereven greater pressure to reallocatetheir production facilities toemerging economies.

All these factors provide a goodopportunity for further acceleratingJapanese FDI into India. Thefollowing two tasks should beaddressed for further expandingIndia-Japan economic relations.First, the Delhi-Mumbai IndustrialCorridor should be started along theright line, which will be thetouchstone for future India-Japancollaboration. Second, humanexchanges should be promoted byall means for cementing andexpanding bilateral relations.Japanese Aid to India (1,545.7million dollars)» Kerala Water Supply Project (II)(Yen Loan: 273.1 million dollars)» Delhi Mass Rapid TransportSystem Project (Phase 2) (II) (YenLoan: 113.2 million dollars)» Gujarat Forestry Development

Project (Phase 2) (Yen Loan: 146.0million dollars).» Transmission SystemModernization and StrengtheningProject in Hyderabad MetropolitanArea (Yen Loan: 197.5 milliondollars)

Conclusion

As this year marks the fifth yearsince the establishment of theJapan-India Strategic and GlobalPartnership in 2006, with the 60thanniversary of the establishment ofdiplomatic relations next year, thisState visit by Prime Minister Nodahas impressed on all sides thefriendly bilateral relationshipbetween Japan and India, andcontributed to further consolidatinga trusting relationship at the leaders'level. With regard to bilateralrelations, the two Prime Ministersshared the view that they shouldexpand cooperation in the area ofmaritime security in the political andsecurity fields. Substantial progresswas made in many economic areas,including cooperation for the DMIC,infrastructure development inSouthern India including theconnectivity between Chennai andBangalore, enhancement of thebilateral currency swaparrangement, extension of ODAloans including for the Delhi MassRapid Transport System, joint rareearths projects, among others. Withregard to regional situations andglobal issues, the two PrimeMinisters also affirmed that the twocountries continue cooperation invarious regional issues in Asia andother international issues such asthe global economy and climatechange.

Advertise yourBusiness Here

[ Contact Us Online ]http://upscportal.com/store/contact

WWW.UPSCPORTAL.COM14

UPSCPORTAL Current Affairs : http://upscportal.com/civilservices/current-affairs

Integrated Guidance Programme for IAS (Pre) - 2012http://upscportal.com/civilservices/online-course/integrated-free-guidance-programme

Integrated Guidance Programme for IAS (Pre) - 2012Dear Candidates,As Civil Services aspirants, all of you are facing a number of problems, from choosing optional Subjects towhat to read in General Studies and many more. These are the main hurdles between you and yourSuccess. To tackle these problems, some of you are taking the help of Coaching institutions, but most ofyou who don’t have this access are in a very difficult Situation. More over the Coaching institutions areCharging very high amounts from the candidates. However their guidance does not yield any result in thereal examination. So this creates a very dim and confusing situation. To Sought out these problems, we arelaunching a free guidance Programme. Where you can avail the guidance of our experts.

They will help you prepare in these following areas:-Guidance and Study Materials for CSAT Paper I and CSAT Paper IIHow to choose your optional subjects?What to read and how to read?What should be the right approach for this examination?Overall strategy and individual specific strategy.Suggested readings and many more.Along with these guidance programmes we are also providing.Important news and Articles from The Hindu, Times of India and Indian Express on a daily basis.

Important Articles from Press Information Bureau and from various websites of Government Minis-tries on a weekly basis.

Our main concern at the moment, is to create equality among the aspirants because the candidate who arestaying in far flung areas or who are doing self study according to the popular notions are in a verydisadvantageous position as compared with the candidates who are taking Coaching from reputed institu-tions. So by launching this programme, we believe in injecting some ray of hope among the Candidates whoare suffering from this injustice. We hope you all appreciate this small effort of ours.

CLICK BELOW LINK FOR DETAILS:http://upscportal.com/civilservices/online-course/study-kit-for-ias-pre-gs-paper-1-2012

CLICK BELOW LINK FOR DETAILS:http://upscportal.com/civilservices/online-course/study-kit-for-ias-pre-gs-paper-2-2012

Thank YouCourse DirectorS.A. Majid+919911626661, +918800966252 WWW.UPSCPORTAL.COM

WWW.UPSCPORTAL.COM 15

UPSCPORTAL Current Affairs : http://upscportal.com/civilservices/current-affairs

Integrated Guidance Programme for IAS (Pre) - 2012http://upscportal.com/civilservices/online-course/integrated-free-guidance-programme

National IssuesInfant Mortality

Rate is Declining

Infant mortality rate (IMR) hasshown a 3 point decline, droppingfrom 50 deaths per 1,000 live birthsto 47 and moving a step closer toachieving the Millennium Develop-ment Goals (MDG) target of 30. How-ever, IMR increased by one point inKerala, Mizoram and Dadra andNagar Haveli. According to the lat-est Sample Registration System(SRS), conducted by the Registrar-General of India, the two worst per-forming States — Madhya Pradeshand Himachal Pradesh — haveshown an impressive 5 point decline.In the former, the figure droppedfrom 67 in 2009 to 62 in 2010 and inthe latter, it was from 45 to 40 . Whilethe IMR national average is 47, itstands at 51 in the rural areas and31 in the urban regions. However,neo-natal deaths continue to be achallenge where 34 babies are stilldying for every 1,000 born.Union Health and Family WelfareSecretary P. K. Pradhan said Bihar,Gujarat, Odisha, Punjab, Rajasthan,Tamil Nadu, Meghalaya, Sikkim andTripura had shown a four-point de-cline in the IMR. Andhra Pradesh,Assam, Chhattisgarh, Haryana,Karnataka, Maharashtra, Delhi,Nagaland, Uttarakhand andChandigarh have shown a three-point decline. Similarly, in Jharkhand,Uttar Pradesh, West Bengal, Jammu

and Kashmir, Manipur and Andamanand Nicobar Islands, the IMR camedown by two points while it re-mained static in Lakshdweep andPuducherry. A one point decline wasreported from Arunachal Pradesh,Goa and Daman and Diu. Impor-tantly, the government intended toset up facilities such as Special NewBorn Care Units, New BornStabilisation Units and New BornBaby Corners at different levels withat least one at the district level. Themother and child tracking systemhad evoked a huge response, with1.32 crore women and 82.6 lakh chil-dren already registered.

50 % of HIV Patients inAsia live in India

India has half of Asia's HIV patientsand is way ahead of China in dis-ease burden. It also finds a place inthe list of 22 countries prioritised forpreventing mother to child transmis-sion infection, according to the lat-est UNAIDS report, drafted jointlywith the United Nations Children'sFund (UNICEF) and the World HealthOrganisation (WHO). About 48 lakhpeople were living with HIV in Asiain 2010 and nearly half of them – 49per cent to be precise – are in India,says the report released on the eveof World AIDS Day. The percentageof pregnant women who tested posi-tive for HIV infection in India alsorose from 2 per cent in 2005 t0 23per cent in 2010.

Seven Asian countries report an es-timated 100,000 or more people liv-ing with HIV in 2009, collectivelyaccounting for more than 90 per centof people with HIV in the region. In-dia tops the list followed by China,Thailand, Indonesia, Malaysia,Myanmar and Vietnam, though thehighest prevalence rate, one percent, was observed only in Thailand.In Asia, the rate of HIV transmissionappears to be slowing down. Theestimated 3,60,000 people who werenewly infected with HIV in Asia in2010 were considerably fewer thanthe 4,50,000 estimated for 2001.

Ex-Im Bank of IndiaAmendment Bill, 2011

Rajya Sabha approved the Export-Import Bank of India AmendmentBill, 2011, on 27 December 2011. Itaims at promoting internationaltrade by raising the capital funds ofoverseas trading from two thousandcrore rupees to ten thousand crorerupees. The Bill was passed by theLok Sabha earlier.

Lokpal and LokayuktaBill, 2011

The Lok Sabha on 27 December 2011,approved the Lokpal and LokayuktaBill, 2011 with the government mak-ing it clear that setting up ofLokayuktas by the states would notbe mandatory. However, the Consti-tutional Amendment Bill, designed

WWW.UPSCPORTAL.COM16

UPSCPORTAL Current Affairs : http://upscportal.com/civilservices/current-affairs

Integrated Guidance Programme for IAS (Pre) - 2012http://upscportal.com/civilservices/online-course/integrated-free-guidance-programme

to confer constitutional status on theanti-corruption watchdog, fellthrough, as the government failed toget two-thirds support for it.

The Lokpal and Lokayukta Bill, 2011was approved after the governmentmoved a few other key amend-ments, including keeping the De-fence Forces and Coast Guard per-sonnel out of the purview of the anti-graft ombudsman and increasing theexemption time of former MPs fromfive to seven years. A number ofamendments moved by the Opposi-tion, including Corporates, Mediaand NGOs receiving donations, weredefeated. The government rejectedthe opposition's demand to bring CBIunder Lokpal.

Regulation of Factor(Assignments Receiv-able) bill, 2011 Ap-

proved

Rajya Sabha approved the Regula-tion of Factor (Assignments Receiv-able Bill, 2011) on 27 December2011 to help micro, small and me-dium enterprises.The bill aims atregulating assignment of receiv-ables by making provision for regis-tration of the rights and obligationsof parties to contract. It applies toall types of industry whether it issmall, medium or big. It will helpmitigate the payment problem of theMSME units. Factors will be regu-lated by the Reserve Bank of India.The Lok Sabha had already passedthe bill.

A bill to Add MoreScheduled Tribes to

List passed

Rajya Sabha of Indian Parliament on22 December 2011 passed thebill the Constitution (ScheduledTribes) Order (Amendment) Bill,2011 to amend the list of ScheduledTribes and add more tribes fromNorth-Eastern India. The bill includessix more tribes namely Thangal,Zeme, Mate, Inpui, Liangmai,Rongmei from Manipur and substi-tutes Galo in place of Galong in thelist of Scheduled tribes in ArunachalPradesh. At present, 26 communitiesexist in the list of ScheduledTribes in Arunachal Pradesh, whichincludes Galong as well. The LokSabha had passed the Bill on 19December 2011.

Indian Army Blindedby Controversial

Equipment

The Indian Army's imagery interpre-tation capabilities, critical to provid-ing information on the locations ofenemy troops and their military as-sets, have been compromised byflawed contracts placed with a com-pany that has failed to provide criti-cal software upgrades, an investiga-tion by The Hindu has found. Docu-ments obtained by The Hindu fromthe Ministry of Defence show thatthe firm responsible for supplyingand integrating software used incritical image intelligence analysiswas relieved of its responsibility toprovide free upgrades in 2008 —and is now on the verge of receivinga Rs.165-crore contract for the sup-ply of software it may no longer have

licensing rights for. MI17 — the su-per-secret military intelligence de-partment that analyses data pro-vided by India's spy satellites — re-lies on software provided by globalsoftware giants Intergraph, Oracle,and Bentley.

Rolta, an Indian company, suppliedphotogrammetry and geographicalinformation system software li-censed from these firms to the Armyin 1996, integrating them into asingle package to meet MI17's spe-cific needs. From then to 2008,things went well — when a new con-tract for 14 photogrammetry andgeographical information systempackages came up to be signed. Theearlier contract bound Rolta to pro-vide software “updates and up-grades” free of cost, as part of amaintenance contract. In 2008,though, the phrasing was changedto just “updates”— freeing Rolta ofthe obligation to provide the mostrecent software released by theoriginal equipment manufacturer.Rolta was paid Rs.506.45 crore forequipment purchased between 1998and 2008. In addition, it receivedannual maintenance contracts forequipment purchased during thisperiod; as of December 2008, theircumulative value was Rs. 40.66crore per annum.

But by early this year, highly-placedmilitary sources said, MI17's image-processing speeds had fallen to justa seventh of those being obtainedby the National Technical ResearchOrganisation, which also analysesthe same data using similar softwarewith the latest upgrades, Intergraph-Erdas. The records of the contractnegotiation committee, or CNC,show a series of questionable deci-

WWW.UPSCPORTAL.COM 17

UPSCPORTAL Current Affairs : http://upscportal.com/civilservices/current-affairs

Integrated Guidance Programme for IAS (Pre) - 2012http://upscportal.com/civilservices/online-course/integrated-free-guidance-programme

sions led to this outcome. In thefourth meeting of the CNC, onemember noted that an odd changehad been made to the name of thesoftware being supplied to MI17:“the vendor,” he observed, “hadadded [the] company name ‘Rolta'in all the software being provided byhim.” The change of name impliedthat the equipment being suppliedwas not the same as was purchasedin 1996, which would have necessi-tated fresh acquisition procedures tobe initiated.

Free Medicare For Poorsin Maharashtra

Maharashtra government will spendRs. 800 crore on the first phase of ahealth scheme for the poor, whichis slated to benefit nearly 50 lakhfamilies from eight districts,Maharashtra Deputy Chief MinisterAjit Pawar said at a function. Underthe ‘Rajiv Gandhi JeevandayeeArogya Yojana,' any family earningbelow Rs. 1 lakh per annum can getfree medical treatment and careworth Rs. 1.5 lakh a year.Maharashtra Chief MinisterPrithviraj Chavan said that no otherState had implemented such ascheme on such a wide level. He,along with the Deputy Chief Minis-ter and State Health Minister SureshShetty distributed identity cards toneedy families from the city at thefunction here.

4.5 Percent Quota ForMinorities

The Union cabinet of India on 22December 2011 approved 4.5 per-cent share for minorities within the27% OBC quota in jobs and univer-

sity seats. The approval will comein force from 1 January, 2012. It sug-gests that minorities can get 4.5 jobsout of every 100 government jobsand university seats. As per the datareleased by the National Commis-sion for Religious and Linguistic Mi-norities, the OBC population was 52percent of India’s population, ofwhich minorities constitute 8.4 per-cent. Justice Rajinder Sachar Com-mittee Report-released in November2006- had found the minority com-munity under-represented in allspheres of professional and publiclife.

Notices for 187 Amend-ments in Rajya Sabha

Notice had been given for as manyas 187 amendments to the Lokpaland Lokayukta Bill, 2011 includingfrom the Trinamool Congress fordeletion of the entire Part III of theBill regarding establishment of theLokayuktas. The Biju Janata Dal andthe Shiromani Akali Dal too had givensimilar notices. By afternoon, theRajya Sabha had received 173amendment notices. The numberswelled to 187 by evening as an-nounced by the Minister of State forPersonnel V. Narayanasamy. Nor-mally notices are taken 24 hours inadvance but on Thursday, noticeswere accepted till late evening, whilethe government took the stand in theHouse that there were too manyamendments which “needed consid-eration.”Among major amendments were theones given by the Bharatiya JanataParty for deletion of words “not lessthan” 50 per cent of members ofLokpal belonging to ScheduledCastes, Scheduled tribes, OtherBackward Classes, Minorities and

Women in Clause 3. They also askedfor deletion of the word ‘Minorities'in Clause 3 and inclusion of Leaderof Opposition in Rajya Sabha on Se-lection Committee in Clause 4. Theyalso wanted Lokpal to appoint hissecretary and not select from apanel given by the government.The Communist Party of India (Marx-ist) gave amendment for replace-ment of eminent jurist on the Selec-tion Committee with “an eminentperson to be nominated by the ChiefElection Commissioner, Comptrollerand Auditor General and Union Pub-lic Service Commission. They alsowanted the Inquiry Wing under theLokpal to be replaced by “Investiga-tion Wing” with the Central govern-ment providing officers and staffunder exclusive jurisdiction of theLokpal as decided in consultationwith the Lokpal. The Party gave an-other amendment for inclusion un-der the Lokpal of “any corporatebody, its promoters, its officers in-cluding Director against whom thereis a complaint of corruption in rela-tion to grant of government licence,lease, contract, agreement or anyother action to influence govern-ment policy through corrupt means.”They wanted Lokpal to take suomotu action in a compliant of cor-ruption.

Courts Can Interfere inthe Case of

Contractual Job

The Supreme Court of India on 20December 2011 ruled that courts caninterfere in the case of contractualjob if the decision taken by the man-agement is arbitrary. A bench of theapex court, however, said that scopeof such judicial review is not all per-

WWW.UPSCPORTAL.COM18

UPSCPORTAL Current Affairs : http://upscportal.com/civilservices/current-affairs

Integrated Guidance Programme for IAS (Pre) - 2012http://upscportal.com/civilservices/online-course/integrated-free-guidance-programme

vasive. It said a writ Court is entitledto judicially review of the action anddetermine whether there was anyillegality, perversity, unreasonable-ness, unfairness or irrationality thatwould vitiate the action. The courtpassed the order on a petition filedby Grid Corporation of Orissa Ltdchallenging the Orissa High Court'sorder. In its order Orissa high courtasked the grid corporation to rein-state one of its employee whosecontractual job was terminated.

National Food SecurityBill approved

The Union Cabinet of India on 18December 2011 approved a draftNational Food Security Bill thatseeks to give legal entitlement ofcheaper food-grains to 62.5 per centof the country's population. The to-tal financial liability to implement thelaw would be 3.5 lakh crore rupees,as funds will be required to raiseagriculture production, create stor-age space and publicity amongothers. The bill aims to provide In-dian citizens access to nutritionalfood at affordable prices. This billproposes to give a legal entitlementto food to 75 percent of our ruralpopulation and 50 percent of oururban population.

The bill also provides for 7 kg of rice,wheat and coarse grains per personper month to priority households at3 Rupees, 2 rupees and 1 rupee perkg respectively.

Ancient numbers in theland of Ramanujan

Literary sources in Tamil mentionnumbers only in words. However, wedo have ancient literary referencesspecifically mentioning eN (‘nu-meral') as distinguished fromezhuttu (‘letter of the alphabet,' forexample, Tolkappiyam 655.4,Tirukkural 392). But as palm leafmanuscripts decay with time andhardly last for more than 200 years,we have to turn to durable stone orpottery inscriptions to know what theancient Tamil numerals looked like.We illustrate two of them here, onefrom each class (Figs. 1 & 2). Theoldest stone inscription featuring anumeral is the Tamil-Brahmi caveinscription from Tondur, nearGingee, in Villupuram district, as-signed to about the 3rd century C.E.(Fig. 1). The numeral 3 engraved atthe end of a short two-line inscrip-tion in the cave is represented bythree horizontal parallel lines. Theinscription records that the villageof Agalur gifted three stone beds inthe cave chiselled by Mosi. The giftwas made to the Jaina monks resi-dent in the cave. The village stillexists with the same name Agalur,near Tondur, both of which still havesizeable Jaina populations. The nu-meral 3 has the same form as in con-temporary Prakrit inscriptions in theBrahmi script in North India.

A well-preserved pottery inscriptionfrom Alagankulam nearRameswaram has only the numeralincised in fairly large size (Fig. 2).The inscription is dated to the 1st or2nd century C.E. The number is read408, from right to left, following theancient convention of reading thedigits from the right (ankaanaam

vaamato gatih). The first digit atright looking like the cross is the sym-bol for 4. It is followed by the sym-bol for 100 (resembling the Brahmiletter sa) and the last symbol at leftis 8, incised in reversed direction.As there is no accompanying text,we do not know the significance ofthe number. The find is still interest-ing for the absence of the place-value system. The convention of us-ing symbols for 10, 100, and 1000 inexpressing the higher numerals wascurrent in Tamil Nadu until the ad-vent of printing and the adoption ofthe international form of Indian nu-merals with place-value system. Thepottery inscription is also good evi-dence for widespread literacy, in-cluding numeracy, in the ancientTamil country.

Animals also Entitled toAccident Compensation

A bench of the Supreme Court in-cluding Justices BS Chauhan and TSThakur on 16 December 2011 ruledthat even animals are entitled toaccident compensation. The courtupheld an award of 13.48 lakh ru-pees for the death of a temple el-ephant. The animal was hit by aKSRTC (Kerala State Transport Cor-poration) bus. The court refused toaccept the contention that the Mo-tor Vehicles Act would be applicableto human beings and not to animals.It clarified that the definition ofproperty in the Motor Vehicles Actis very inclusive and wide. There-fore, animal should be included un-der this act. The court added that theelephant was owned by the templeand it was God’s property.

WWW.UPSCPORTAL.COM 19

UPSCPORTAL Current Affairs : http://upscportal.com/civilservices/current-affairs

Integrated Guidance Programme for IAS (Pre) - 2012http://upscportal.com/civilservices/online-course/integrated-free-guidance-programme

5 Girls Gailed as ‘icons'by President

The girls, with little education andalmost no support, turned downmarriage proposals and faced theanger of their families and the com-munity. Their stories of courage im-pressed the President so much thatshe got them invited to theRashtrapati Bhavan. They earnedpraise from the President, who de-scribed them as “icons” and askedthem to share their stories and en-courage girls to say no to under-age marriages. For Afsana and Sunita,it was their second meeting with thePresident who encouraged them tocampaign against child marriageamong their peers.

While education opens up new av-enues for the girl child, an earlymarriage brings with it a cycle ofmisery, poor health and poverty, shesaid. A young mother with childrenoften become victim of poor healthand mortality. She urged the districtauthorities to create more aware-ness of what the girls have done bytaking them on a tour of their dis-trict during their school holidays toinspire others.

10 % of the Group B Postsin Paramilitary Forces to

Ex-Servicemen

The Union Ministry of Home Affairsagreed to fill 10% of the Group ‘B’posts in Central Paramilitary Forcesfrom among Ex-Servicemen. Pres-ently, the scheme caters to 38 lakhbeneficiaries including 12 lakhEx-Servicemen and over 26 lakh depen-dents. This scheme is currently be-ing implemented through 247

polyclinics. Initially, 227 polyclinicshad been approved by the UnionCabinet of India. Later, in 2008 an-other 199 polyclinics were sanc-tioned, out of which 20 are func-tional and 179 polyclinics are in vari-ous stages of implementation. Dur-ing the year 2010-11, a total of 89lakh patients attended the polyclin-ics. With the commissioning of allthe sanctioned polyclinics, the en-tire fraternity of Ex-Servicemenwould be covered comprehensively.

Cable Television Net-works Regulation Act

amended

The Lok Sabha on 13 December 2011unanimously passed a Bill to furtheramend theCable Television Net-works Regulation Act, 1995. It re-places a Presidential Ordinance of25 October, 2011. As per theamendments, there will be a checkon objectionable contents.Doordarshan will be able to providemore free-on-air channels in theDTH mode to audiences across thecountry. Cable TV operators violat-ing the provisions of the Bill will facepenalty.

The legislation will reduce depen-dence on the television rating points,TRP. By 2014, the entire cable net-work will be digitized. Those Cableoperators who will violate the CableTelevision Network RegulationAct,1995 Government will have theright to consider about the continu-ity of the Licence of those cable op-erators. Till date, this kind of provi-sion was not in existence in the formof Act.

Kalam FavoursLinking of Rivers

Former President of India A.P.J. AbdulKalam advocated interlinking riversand dams to form a smart nationalwater grid management scheme asIndia cannot afford civil war or con-flict over water. He said rivers, wa-terways and water resources mustbe nationalised on the lines of roadsand power sectors. “Water is nowin the control of States. It has to belinked by forming a national watergrid management just like NationalHighways and power grid,” he said.Inaugurating ‘Agricon 2011', a con-ference on agriculture R&D trends2020, organised by CII, he said, “Wa-ter cannot be a [source of] conflictin India. India cannot afford to havea civil war on water when we haveto work unitedly for development.Nation is bigger than the individualparties (political parties) and States.Indian rivers have to be operated bythe Army, Navy or the armed forcesfor equitable distribution of water todouble food production.”

About 170 million hectare arableland will be reduced to 100 millionhectare due to increasing populationgrowth and afforestation and envi-ronmental preservation activities, DrKalam said and urged agriculturalscientists and technologists to workwith organisations such as IFFCO toenhance average productivity perhectare from 1.1 tonnes to 3.4tonnes of the available land for cul-tivation with less need of water. Re-ferring to precision farming projectof the State, he said by doing theright thing, in the right place at theright time, Tamil Nadu had createda great awareness among the farm-ers which gave higher productivity,

WWW.UPSCPORTAL.COM20

UPSCPORTAL Current Affairs : http://upscportal.com/civilservices/current-affairs

Integrated Guidance Programme for IAS (Pre) - 2012http://upscportal.com/civilservices/online-course/integrated-free-guidance-programme

market access and made farmers asentrepreneurs.

When the entire nation was strug-gling to achieve even two per centgrowth in agriculture, Gujaratposted seven to nine per cent overthe past six years. The Gujarat gov-ernment was able to do it by imple-menting mission mode action. Dr.Kalam also cited Bihar's experiencein doubling the productivity and said“these experiences clearly show that‘we can do it', but what we need isa mission mode approach across thenation with respect to agriculture;and we need a creative leadershipwho has a vision for leading the sec-ond green revolution for the next 10years.”

Mullaperiyar Dam’sWater Level Should Not

Exceed 136 Feet

The Supreme Court of India directedTamil Nadu to ensure that the waterlevel in the Mullaperiyar dam doesnot exceed 136 feet but declined toentertain Kerala’s plea for reducingthe level to 120 feet. A five-JudgeConstitution Bench, headed by Jus-tice DK Jain gave the order. The Su-preme Court asked Tamil Nadu andKerala to maintain restraint on theirstatements on the Mullaperiyar damrow. Urging for sanity and sensitiv-ity, the court lamented that they wereadding fuel to the fire instead ofdousing it. The Bench also asked theCentre to clarify its position on TamilNadu's plea for deployment of Cen-tral Industrial Security Force to pro-tect the dam from possible vandal-ism.

Karnataka Now Third inSugar Production

Karnataka has now emerged as oneof the top sugar producing States inthe country, after Maharashtra andUttar Pradesh. The State's sugar pro-duction has shot up to 37.14 lakhtonnes, an increase of 12.25 lakhtonnes in just under a year, accord-ing to reports received in the Stateheadquarters about a fortnight ago.The sugar year commences in Octo-ber and ends in September. Evensugar recovery has improved dra-matically, and Karnataka now standssecond, after Maharashtra. Sugarrecovery is the yield of sugar for ev-ery tonne of sugarcane crushed infactories. The average recovery inthe State is now 10.95 per cent. It is11.50 per cent in Maharashtra and9.7 per cent in Uttar Pradesh. It wasa happy augury that Karnataka hademerged as the third largest sugarproducer in the nation. It was onlysome years ago that the State over-took Andhra Pradesh and TamilNadu. It now has almost 60 sugarfactories, a quantum jump over thepast decade. Sugarcane productionin the State has increased from 228lakh tonnes to 339.03 lakh tonnesin just about a year although thearea under sugarcane has only mar-ginally increased.Amendments to the Petroleum andMinerals Pipeline ActThe Lok Sabha on 12 December 2011passed the amendments to the Pe-troleum and Minerals Pipelines (Ac-quisition of Right of User in Land)Act, 1962. The Bill aims to makeprovisions for sufficient deterrenceto criminals from committing theoffence of pilferage or sabotage. Theexisting sub-section (2) of section 15provides that whoever willfully re-

moves, displaces, damages or de-stroys any pipeline, shall be punish-able with rigorous imprisonment ofa term which shall not be less thanone year, but which may extend tothree years and shall also be liableto fine. Section 16 provides that of-fence under sub-section 15 shall bedeemed to be cognizable under theCod of Criminal Procedure,1973. The proposed amendmentsprovide for higher quantum of pun-ishment in terms of imprisonmentand provision for death penalty inrare cases where the Act of sabo-tage is dangerous and is likely tocause death of any other person. TheBill further proposes to amend sec-tion 16 of the aforesaid Act to makethe offences under sub-section (2),(3) of section 15 of the said Act tobe cognizable and non-bailable. Theamendment to the act was neces-sary to prevent pilferage of petro-leum by organized gangs and sabo-teurs.

Supreme Court directionfor Night Shelters for

Homeless People

The Supreme Court directed stategovernments to build adequate num-ber of night shelter to ensure thatno homeless person has to sleepunder sky this winter. A bench of jus-tices Dalveer Bhandari and JusticeDipak Misra asked the governmentsto file their detailed report by 3January 2012 on the status of nightshelters, which are operating in theirstates. The bench, which passedseparate orders for different states,asked the Chief Secretaries to en-sure construction of sufficient num-ber of shelters for the homeless andpoor people.

WWW.UPSCPORTAL.COM 21

UPSCPORTAL Current Affairs : http://upscportal.com/civilservices/current-affairs

Integrated Guidance Programme for IAS (Pre) - 2012http://upscportal.com/civilservices/online-course/integrated-free-guidance-programme

NCT of Delhi Laws(Special Provisions) Bill

of 2011

The Lok Sabha on 12 December 2011unanimously passed the NationalCapital Territory of Delhi Laws (Spe-cial Provisions) Bill of 2011, whichgranted a three-year term extensionto the Master Plan of Delhi (MPD).It is meant to ensure continuedimplementation of all the ongoingschemes for its betterment. The re-vised master plan of Delhi will havea long-term vision of 25 years toaddress the problems speciallyfaced by the weaker sections of thesociety. The government intends toimplement the second phase of theJawaharlal Nehru Urban RenewalMission after its current phase isover. The objective of the bill is toprevent sealing and demolition ofunauthorised structures in Delhi till31 December, 2014. It seeks tomaintain the status quo regardingunauthorised colonies, which in-cludes storages, warehouses, andgodowns for farm produce, villageabadis (settlements)and theirextensions.Commercial establish-ments in rural areas and urbanisedvillages of Delhi are expected to geta reprieve after the proposed bill isenacted.

Village Water andSanitation Committee inEach Gram Panchayat

The Government decided to set upa village water and sanitation com-mittee in each Gram Panchayat, vil-lage and ward to ensure safe drink-ing water supply. The committee willbe set up as a standing committeein each Gram Panchayat for plan-

ning, monitoring, implementationand maintenance of water supplyschemes in the area with active par-ticipation of the villagers. The mem-bership of the committee will con-sist of 6 to 12 persons comprisingelected Panchayat members and atleast 50 percent women with duerepresentation to Scheduled castesand Tribes. The committee will bean integral part of the villagepanchayat. This initiative has beentaken under the National Rural Drink-ing Water Programme to decentral-ize power and responsibilities andto give greater focus on water andsanitation issues.

ConstitutionalAmendment to Set up

NJC Suggested

The Parliamentary panel on law andjustice, headed by Abhishek Singhvistated that the Constitution shouldbe amended to set up a NationalJudicial Commission with powers toinitiate criminal prosecution andsacking of corrupt judges. The con-cluded that judiciary cannot be leftunpoliced in matters of corruption.The report prepared by the Parlia-mentary panel recommended set-ting up of a National Judicial Com-mission (NJC) to create a broad-based and comprehensive model ofjudges appointments. If necessary itshould include amendments in theConstitution in the process of com-prehensive model of judges appoint-ment.Recommendations of the Parlia-mentary Panel

• The NJC has to be entrusted withpowers of both transfer and crimi-nal prosecution of judges for corrup-

tion.• The panel stated that Judicial Ac-countability Bill is meant tostrengthen the existing in-house pro-cedure of the judiciary to deal withwrong behaviour of judges.• The panel also recommended thatthe former judges working as chair-persons and members of tribunalsshould also be brought under theNJC.• It may also be given the power fordismissal or removal of judges byrelevant amendments in the consti-tution.

Tamper-Free HighSecurity Number Plates

for Vehicles

Supreme Court of India on 8 Decem-ber 2011 directed all states to en-sure vehicles carry tamper-free highsecurity number plates within fourweeks. Giving last opportunity tostate governments to implement thescheme, a bench of the apex court ruled that contempt proceedings willbe initiated against the state au-thorities in case of non-complianceof the scheme within the time limitfixed by it.

The bench added that no further timewill be granted to put in force thescheme and directed the state au-thorities to file their affidavits oncompliance of its order within fourweeks. The court passed the orderon a PIL seeking its direction to in-troduce tamper-proof number platesand licences for vehicles.

WWW.UPSCPORTAL.COM22

UPSCPORTAL Current Affairs : http://upscportal.com/civilservices/current-affairs

Integrated Guidance Programme for IAS (Pre) - 2012http://upscportal.com/civilservices/online-course/integrated-free-guidance-programme

Inspection ofMullaperiyar Dam by

Empowered Committee

The Empowered Committee in itsmeeting on 5 December 2011 de-cided to send two technical mem-bers to inspect the Mullaperiyardam before finalising its report fol-lowing apprehensions raised byKerala about the safety of the damafter mild tremors occurred in thearea. The Empowered Committee isheaded by former Chief Justice ofIndia, A.S. Anand. The committeehad conducted a spot inspection ofthe dam in December 2010. Duringits meeting on 5 December, the com-mittee considered the reports of thestudies and investigations con-ducted by various agencies it con-stituted to study the dam's safety. Italso discussed the applications filedrespectively by Kerala and TamilNadu and took note of their submis-sions.The committee, according to Keralaought not to have relied on the Su-preme Court's order as it was notbarred from receiving evidence inrespect of seismic threat to thedam. Following Kerala’s argumentthe committee agreed to hear theState's arguments on this count.Since Kerala filed an application forbringing on record the recent trem-ors and sought an oral hearing, thecommittee decided to send two ofits members to conduct a spot in-spection of the dam. Kerala's pleafor oral hearing was also accepted.The committee directed Tamil Naduand Kerala to make submissions on2 January 2012. On 3 January thecommittee will consider the modali-ties for finalising its report to theSupreme Court.

During the months of July-November2011, 25 earthquakes were reportedin the Mullaperiyar dam region .Some of the earthquakes on 26 Julyand 18 and 26 November were morethan magnitude 3 on the RichterScale. The highest magnitude 3.8was recorded on 26 July 2011. Thefrequent earthquakes caused panicamong 50 lakh people in the down-stream region of the Periyar dam.The fear was further heightened bythe incessant rain in the catchmentsof Mullaperiyar shooting the storageabove 136 feet mandated by the pro-visions of the Amendment Act of2006. If the Mullaperiyar dam breaksdue to flood or earthquake, a massdisaster would follow, wherein notonly the life and property of 50 lakhpeople will be affected in Kerala, butlakhs of farmers of Tamil Nadu willalso lose irrigation of about 2.5 lakhacres. Kerala therefore feels thatTamil Nadu should agree for the con-struction of a new dam offered byKerala as a precautionary measure.

Adultery LawBiased against Men

The Supreme Court of India on 1December 2011 stated that sec-tion497 of the Indian penal Code isbiased against men. The sectionpunishes a man alone for adulteryfor having consensual sex with amarried woman. As per the observa-tion of the court, the provision ofsection 497 reduces a marriedwoman to a property of the husbandand it punishes man only despite thefact that the woman with whom hehad consensual sex was an equalpartner in the alleged crime. Thecourt added that the provision isunder criticism from certain sectionsfor showing a strong gender bias. A

bench of the Supreme Court includ-ing Justices Aftab Alam and R MLodha made the above observation.

Section 497 of IPC: Section 497 ofIPC states that whoever has sexualintercourse with a person who is andwhom he knows or has reason tobelieve to be the wife of anotherman, without the consent or conniv-ance of that man, such sexual inter-course not amounting to the offenceof rape, is guilty of the offence ofadultery, and shall be punished withimprisonment of either descriptionfor a term which may extend to fiveyears, or with fine, or with both. Insuch case, the wife shall not be pun-ishable as an abettor.

21 Anti-RetroviralTreatment Plus Centres

The National AIDS ControlOrganisation (NACO) decided to rollout 21 Anti-Retroviral Treatment(ART) Plus centres to meet the in-creasing demand of providing sec-ond line drugs to HIV patients whofailed to respond to treatment pro-vided previously. The NACO’s ARTprogramme is headed by Dr B BRewari. 21 more ART Plus centreswill be rolled out exclusively for suchHIV positive people who require sec-ond line drug therapy. 2600 peopleare currently on second line drugsfor HIV as they were critical despitetaking medication. Presently a totalof 4.48 lakh people are availing freeof cost drugs at 324 ART centres.Centres were proposed in Surat,Patna, Guwahati, Lucknow,Ahmedabad, and Jaipur. The govern-ment earmarked Rs 350 crore for theentire project of providing free ofcost drugs at all ART centres. Ofthese 21 centres, seven are already

WWW.UPSCPORTAL.COM 23

UPSCPORTAL Current Affairs : http://upscportal.com/civilservices/current-affairs

Integrated Guidance Programme for IAS (Pre) - 2012http://upscportal.com/civilservices/online-course/integrated-free-guidance-programme

functional. 10 centres had also been set up to provide the facility for second line drugs and seven paediatric HIVcentres in the country. In addition to the 324 ART centres and other second line ones, there are 678 link centresand 259 community care centres.

NACO streeses on strengthening the link between the ART centres and more than 5000 Integrated Counsellingand Testing Centres where people are initially identified and detected with the virus. As per the National FamilyHealth Survey-3, there are 2.47 million HIV infected people in India of which 3.8 per cent are children. Nearly 40per cent of the people are unaware of their HIV status. According to NACO, there are around 30000 patientsavailing of medication in the private sector. The World AIDS Day theme was getting to zero, but for the NACO itwas also providing 100 per cent access to anti-retroviral drugs.

(IMP) Get UPSCPORTAL, IAS, Jobs,Results, Notification ALERTS in Email.

Step-1: Fill Your Email address in LINK below. You will get a confirma-tion email within 10 min.

Step-2: Verify your email by clicking on the link in the email.(IMP: check Inbox and Spam folders)

Step-3: Done! Now you will regular Alerts on your email.

Click Below Link to get Free Newsletter:

http://upscportal.com/civilservices/newsletter

WWW.UPSCPORTAL.COM24

UPSCPORTAL Current Affairs : http://upscportal.com/civilservices/current-affairs

Integrated Guidance Programme for IAS (Pre) - 2012http://upscportal.com/civilservices/online-course/integrated-free-guidance-programme

International IssuesPakistan Budges on

Bonn Meet

Pakistan on Wednesday hinted at thepossibility of participating in thecoming Bonn Conference on Af-ghanistan but ruled out any high-level representation on the groundthat Afghan soil had been used byNorth Atlantic Treaty Organisation(NATO) to attack the country in whatthe Army calls a “deliberate” act ofaggression. Agreeing to considerGerman Chancellor Angela Merkel'srepeated requests for Islamabad'sparticipation, Prime Minister SyedYusuf Raza Gilani said he would re-fer the suggestion of havingPakistan's Ambassador in Germanyattend the deliberations to the Par-liamentary Committee on NationalSecurity. Ms. Merkel called Mr. Gilanito impress upon him the importanceof Pakistan's participation at themeeting to make it meaningful. AsMr. Gilani was unwilling to budge onhigh-level participation, she sug-gested the Ambassador be permit-ted to represent Pakistan so that itsseat at the table was not left vacant.In view of bilateral relations and thefact that the German Foreign Minis-ter was among the first to person-ally call his Pakistani counterpart toexpress solidarity with Pakistan andcondole the death of 24 PakistanArmy soldiers in the NATO firing atPakistani outposts on Saturdaymorning, Mr. Gilani agreed to refer

the request to the ParliamentaryCommittee. Meanwhile, the formalcommunication to the U.S. asking itto vacate the Shamsi airbase hasbeen sent with December 11 set asthe deadline. Pakistan has releasedfootage of two posts which cameunder fire from helicopters of thecoalition forces in Afghanistan andwanted to know where the NATOcasualties were in case there wasfiring from the Pakistani side.

EU to ban Import ofIranian Crude Oil

The member states of EuropeanUnion(EU) on 4 January 2012,agreed in principle to ban import ofIranian crude oil to put pressure onIran for its nuclear programme. How-ever, the time-frame to implementthis was not decided. The UnitedStates, which recently imposed freshsanctions on Iran, has welcomed thenews. Meanwhile, Iran has dis-missed the threat of new sanctionsand denies Western claims that it istrying to develop a nuclear weaponsprogramme. Western powers accuseIran of trying to develop nuclearweapons under the cover of a civil-ian energy programme. The EU is oneof Iran’s biggest markets for oil. Thisgenerates most of the Iraniangovernment’s revenue. China is thetop buyer of Iranian oil.The EU is apolitical and economical union of 27member states.

Ban lifted byMaldives on Spas

Maldives on 4 January 2012 liftedban on spas in the upmarket touristdestination following its verificationthat these spas were not being usedfor prostitution. Earlier, it was al-leged that these spas were the hubof prostitution. Following this, thetourism ministry ordered all mas-sage centres to close six days ago.The tourism industry is an importantforeign exchange earner and em-ployer in the Maldives and the banon spas could harm the industry.Maldives in 2011 received more than850000 tourists.

Israel-Palestine talks

The Israel-Palestine talks ended inthe first week of January 2012 withthe international mediators fromQuartet (the United Nations, theUnited States, the European Union,and Russia) and Jordan ended with-out any breakthrough in Amman,capital of Jordan. The talks and theoutcome were positive and the twosides agreed to carry forward thediscussions. The Amman round ofpeace talks may not have yielded abreakthrough, yet it has broken theice between the Israeli and Pales-tinian sides to discuss resumptionof the stalled peace talks. Israelcharged the Palestinians with rag-ing propaganda campaigns and uni-

WWW.UPSCPORTAL.COM 25

UPSCPORTAL Current Affairs : http://upscportal.com/civilservices/current-affairs

Integrated Guidance Programme for IAS (Pre) - 2012http://upscportal.com/civilservices/online-course/integrated-free-guidance-programme

lateral diplomatic initiatives againstit while Palestine asked Israel to stopsettlement construction and acceptthe 1967 borders. USA and SaudiArabia inked an Agreement worth29.4 billion US Dollars

White Paper by China

China on 29 December 2011 issueda white paper entitled China’s SpaceActivities in 2011. The white paperis on the development of space in-dustry since 2006 and the majortasks for the next five years. It wasthe third white paper on China’sspace activities. The white paperwas issued by the State Council In-formation Office. China has madethe space industry an important partof the nation’s overall developmentstrategy with the objective of explor-ing and utilizing outer space forpeaceful purposes. The Major tasks, which were listed in the white pa-per for the next five years includespace transportation system, Earthsatellites, human spaceflights anddeep-space exploration. The whitepaper stated that China would worktogether with the international com-munity to promote world peace anddevelopment.

India wanted 358items removed

India is one of only four countrieswhich, during the first half of 2011,requested Google to remove contenton the basis that it was critical ofthe government. Google refused tocomply. The other countries wereThailand and Turkey -- where Googlerestricted local users from access-ing the offending content -- and theUnited States, where it refused. Ac-

cording to Google's TransparencyReport for January to June 2011, theInternet search giant received re-quests from the Indian government– which seems to include State andCentral governments, police andcourts – to remove 358 items. In abreakdown of reasons for such re-quests, 255 items were classifiedunder the “government criticism”category. It is not clear if Googlewould classify offensive items abouta political leader under the categoryof defamation or government criti-cism. Interestingly, the biggestchunk of this is accounted for by asingle “request from a local law en-forcement agency to remove 236communities and profiles from [so-cial networking site] orkut that werecritical of a local politician.” Googledid not identify this politician, but itdid state that “we did not complywith this request, since the contentdid not violate our Community Stan-dards or local law.” Google's statis-tics gain significance in the light ofits alleged refusal to comply with theIndian government's recent demandto block the publication of incendi-ary hate speech from its sites. OnMonday, Union Minister for Commu-nications and Information Technol-ogy Kapil Sibal summoned execu-tives of Google, Yahoo, Facebookand Microsoft for a meeting afterthey stonewalled repeated requeststo block incendiary communal ma-terial being posted on social net-working sites they operated.

However, Google's TransparencyReport data seems to indicate thatonly 8 items were requested to beremoved under the category of hatespeech. Instead, 39 items were re-quested to be removed on groundsof defamation, 20 due to privacy and

security concerns, 14 due to imper-sonation, three identified as porno-graphic items, and one request dueto national security reasons. How-ever, the single largest category isgovernment criticism; apart from the236 items on orkut, the governmentalso asked for 19 items on YouTubeto be removed on these grounds.Overall, Google says it complied fullyor partially with 51 per cent of therequests. “We received requestsfrom state and local law enforce-ment agencies to remove YouTubevideos that displayed protestsagainst social leaders or used offen-sive language in reference to reli-gious leaders,” said the Google re-port. “We declined the majority ofthese requests and only locally re-stricted videos that appeared to vio-late local laws prohibiting speechthat could incite enmity betweencommunities.”

Iranian Embassy shutdown by UK

The United Kingdom shut down theIranian embassy in London and ex-pelled all its staff in retaliation tothe storming of the British diplo-matic compound by an angry mob.They were asked to leave the UKwithin 48 hours. The British govern-ment also shut its embassy in Iranand evacuated the staff. France andGermany have both recalled theirambassadors to Iran for consulta-tions following the attack on the U.K.embassy in Tehran.

Arab League Observersin Syria

A team of 50 Arab League Observ-ers arrived in Damascus in Syria on

WWW.UPSCPORTAL.COM26

UPSCPORTAL Current Affairs : http://upscportal.com/civilservices/current-affairs

Integrated Guidance Programme for IAS (Pre) - 2012http://upscportal.com/civilservices/online-course/integrated-free-guidance-programme

26 December 2011. The team con-sisting of experts from Algeria andTunisia is led by Sudanese GeneralMustafa Daby. It would take about aweek for Arab League to find out ifthe peace plan is being implementedin totality. The Arab Observer mis-sion to monitor the implementationof League mediated peace planwould visit the worst hit cities inphases. However, the group willmaintain an element of surprise byonly announcing the specific areasthey would be visiting on the sameday of departure. The 50-membergroup consists of politicians, law-makers and military officials. Theywill split themselves into ten groupsfor different cities. The team is likelyto visit Homs amidst reports of kill-ings of 23 people by the securityforces.

Iranian Exiles in Iraq to be relocatedThe United Nations (UN) and theIraqi Government on 24 December2011 signed an agreement to relo-cate Iranian exiles living in a campin northeastern part of Iraq. The Iraqigovernment has been insisting onclosing the camp by the end of thismonth. Under the agreement, UNHigh Commission for Refugees willmonitor the relocation while IraqiGovernment will be responsible fortheir security. UNHCR would decidethe refugee status for the residentsof Ashraf here. This will be the firststep toward resettlement to thethird countries other than the nativeor the host countries. Camp Ashrafwas the base of dissident Iraniansled by the People's Mujahedeen Or-ganization of Iran. The group movedto Camp Ashraf during the regimeof Saddam Hussein and sided withIraq in the war against Iran in the1980s. The camp, now home to over

3400 dissident Iranians will beshifted to Camp Liberty, a former U.S.military base near the Baghdad In-ternational Airport.

CSTO agreed toTighten Rules

The leaders of the CSTO (CollectiveSecurity Treaty Organisation) na-tions on 21 December 2011 agreedthat the deployment of foreign basesin their territory would be done withthe approval of all partners of thedefence alliance. The CSTO summitwas held in Moscow. BesidesBelarus and Kazakhstan, CSTO in-cludes Armenia, Russia, Tajikistan,Kyrgyzstan and Uzbekistan.Kazakhstan succeeded Belarus asthe President of CSTO. However, theCSTO agreement does not apply toexisting facilities such as a Germanair transit facility in Uzbekistan,French military aircraft in Tajikistanand US transit centre in Kyrgyzstan.

A unique protest inChina

It is the farmers who are usuallyevicted in land conflicts in China'svillages. But in the southern Chinesevillage of Wukan this week, a first-of-its-kind rebellion by 20,000 villag-ers over a land conflict has forcedout the entire local government,Communist Party leadership andpolice forces, sparking clashes thatresulted in an unprecedented siegeof the village, located in prosperoussouthern Guangdong province, onWednesday. According to accountsfrom witnesses posted on Chinesewebsites, the village has now beensurrounded by thousands of riot po-lice, while villagers had barricaded

themselves in by blocking roads withtrees and rocks. The siege of Wukanwas the denouement of more thanthree months of protests over landacquisition, seen as the biggestdriver of the more than 180,000“mass incidents” Chinese authori-ties record every year.

Local authorities had recently ap-peared willing to address grievancesover what villagers said was inad-equate land compensation, whenthey welcomed a group of residents,elected by the local community torepresent the farmers' interests, tonegotiate. However, the chief nego-tiator, Xue Jinbo, was subsequentlyheld by authorities and died in cus-tody this week. The Lufeng city gov-ernment, which oversees Wukan,said in a statement he died of heartproblems, though Wukan's residentswidely suspected he was murderedwhile in custody. Following Xue'sdeath and after months of stallednegotiations with local officials, theresidents of Wukan decided enoughwas enough, storming the local po-lice station and clashing with police.By Wednesday, authorities had beendriven out of the village, with boththe local police station as well asthe Communist Party's local officesreported as being deserted, andWukan village — in a likely first inChina's modern history — being en-tirely controlled and administered byits residents.

Villagers in Wukan have demandedthat officials return Xue's body, aswell as suspend plans to acquirelands for a development project. Itremains unclear whether the localofficials will concede to the de-mands, or on the other hand usegreater force to disperse the pro-

WWW.UPSCPORTAL.COM 27

UPSCPORTAL Current Affairs : http://upscportal.com/civilservices/current-affairs

Integrated Guidance Programme for IAS (Pre) - 2012http://upscportal.com/civilservices/online-course/integrated-free-guidance-programme

tests. This week's incident came asChina's highest leaders met inBeijing for an annual economic workconference. The meeting, chaired byPresident Hu Jintao, concluded witha declaration that the focus ofChina's economic development inthe coming year would be “makingprogress while maintaining stabil-ity.”

Hillary Clinton visitedMyanmar

Making a diplomatically risky trip tothe long-isolated Southeast Asiannation of Myanmar, U.S. Secretary ofState Hillary Rodham Clinton saidshe wanted to see for herselfwhether new civilian leaders weretruly ready to throw off 50 years ofmilitary dictatorship a test that in-cludes rare face-to-face meetingswith former members of the junta.During her visit, Ms. Clinton will alsoencourage Myanmar to sever mili-tary and nuclear ties with North Ko-rea. Ms. Clinton arrived in the capi-tal of Naypyidaw on the first trip bya U.S. Secretary of State to Myanmarin more than 50 years. She is tomeet senior Myanmar officials onThursday before heading to the com-mercial capital of Yangon, where shewill see opposition leader Aung SanSuu Kyi.

Most Uncharitable Na-tion in South Asia is India

As per the Charities Aid Foundation’sWorld Giving Index, India ranked asthe most uncharitable nation ofSouth Asia in 2011. India is the worstperformer in South Asia with a glo-bal ranking of 91. India was rankedat 134 in 2010. Pakistan was ranked

to 34th position in 2011 while SriLanka ranked 8th. Bangladesh wasplaced at 78th position and Nepalat 84th. Thailand was the most gen-erous nation, with 85 per cent of itspeople. The United Kingdom was thesecond most generous nation, with79 per cent regularly giving tocharity.Charities Aid Foundation’sWorld Giving Index ranked nationson the strength of monetary dona-tions and charitable acts.

Switzerland may FreeNuclear Smugglers

In an unexpected turn of events,Swiss authorities announced a de-cision to enter into a plea bargainwith the notorious Tinner familymembers, who have been in jail overnuclear smuggling charges in the il-licit network of disgraced Pakistaniscientist Abdul Qadeer Khan. TheTinners, Friedrich and his two sonsUrs and Marco, were also accusedof being informants for the CIA andone of the father-sons team wasactually said to have been a CIA con-tractor. The Office of the AttorneyGeneral of Switzerland has notedthat it had formally filed chargesagainst the Tinnners under the WarMaterial Act, an indictment based ontheir alleged aiding of “the illegalnuclear weapons programme of anunknown state through various ac-tivities”.

However, the Swiss authority alsosought to close the case against theTinners after the courts found ver-dicts of guilt “in relation to offencesunder the WMA and against one ofthe sons for forgery of documents”.The prosecutors said the proceed-ings in respect of other offences hadalready been dropped and the court

was “requested to accept a pleabargain between the parties cover-ing sentences, the allocation ofcosts, the forfeiture of assets andother matters”.

Sanctions on Libya liftedby USA

The USA government on 16 Decem-ber 2011 lifted most of its sanctionson Libya, unfreezing some of Libya'sassets held in the United States. TheUSA government freed about 30 bil-lion dollars in assets owned by theLibyan government and 2 Libyanbanks that had been blocked sinceFebruary. But holdings of the lateleader Muammar al-Qadhafi and hisfamily, as well as those of his aides,will be kept frozen. The US decisioncoincided with a similar action by theUN Security Council, which decidedto end a freeze on the assets ofLibya's financial institutions, includ-ing the central bank.

Fai Pleads Guilty toConspiracy

Syed Ghulam Nabi Fai (62), aKashmiri-origin U.S. citizen arrestedby the Federal Bureau of Investiga-tion on July 19, 2011 for allegedlyacting as an unregistered lobbyist ofPakistan's Inter-Services Intelli-gence, pleaded guilty to conspiracyand tax violations. Fai's guilty pleapertained to a decades-long schemeto conceal the transfer of at least$3.5 million from the government ofPakistan to fund his lobbying effortsin America related to Kashmir, theU.S. Department of Justice (DoJ) saidin a statement. Specifically Faipleaded guilty on two counts of“criminal information,” the DoJ re-

WWW.UPSCPORTAL.COM28

UPSCPORTAL Current Affairs : http://upscportal.com/civilservices/current-affairs

Integrated Guidance Programme for IAS (Pre) - 2012http://upscportal.com/civilservices/online-course/integrated-free-guidance-programme

ported, explaining that the first countwas conspiracy to falsify, concealand cover up material facts he hada duty to disclose; and to defraudthe Treasury Department by imped-ing the lawful functions of the IRS inthe collection of revenue. The sec-ond count to which he admitted guiltwas the charge of endeavouring toimpede the administration of taxlaws. In a case that rocked the al-ready dismal U.S.-Pakistan relation-ship over the summer the arrest ofFai was followed by revelations thathe had served as the Director of theKashmiri American Council (KAC), anon-governmental organisation inWashington, D.C.

NATO ended its TrainingMission in Iraq

NATO ended its training mission inIraq on 17 December 2011. The mis-sion was aimed at assisting in thedevelopment of Iraqi security forcestraining structures and institutions.It was comprised of 120 soldiersfrom 12 countries. All of them willleave by the end of 2012. The movefollows a day after US militaryhanded over the last military basein Iraq and officially announcedshutdown of its operations in thecountry.

Earlier, the alliance had been askedby Prime Minister Maliki to extendits training mission until the end of2013 and it had accepted the requestin principle. However, Baghdadmade it clear that it can't grant im-munity to the soldiers on its own asthe matter has to be taken to thecabinet and then parliament for ap-proval.

Sanctions on Libya’sCentral Bank lifted by

UNSC

The UN Security Council on 16 De-cember 2011 lifted sanctions onLibya's central bank and thecountry's foreign investmentsbanks. The move is aimed at easingthe current cash crisis in Libya. TheUnited States has also lifted mostof the sanctions against Tripoli. TheLibyan banks' assets abroad werefrozen in early 2011 as part of sanc-tions against former Libyan leaderCol. Muammer Gaddafi. The USArolled back most sanctions on thegovernment of Libya to keep its com-mitment to the Libyan people.

OPEC to increase itsProduction

OPEC, (The Organisation of the Pe-troleum Exporting Countries) in itssummit held in Vienna, on 14 De-cember 2011 decided to increase itsproduction ceiling to 30 million bar-rels a day, which is the first changein three years. Earlier, the produc-tion output was 24.88 million bar-rels per day. OPEC agreed to the newlimit but it won't set individual quo-tas for each member country. OPECis increasing its quota to match upactual production. The last OPECsummit in June failed to reach con-sensus when six members includingIran and Venezuela opposed theidea to pump more oil by SaudiArabia and three other Gulf coun-tries, who increased their oil produc-tion to make up for Libyan exportswhich was stopped following thecivil war.OPEC will need to produce 30.1 mil-lion barrels a day in 2012 to balance

world supply and demand of oil.OPEC is an organisation of twelveoil-exporting countries consisting ofIran, Iraq, Kuwait, Qatar, SaudiArabia, the United Arab Emirates,Nigeria, Angola, Venezuela, Libya,Algeria and Ecuador. Theorganisation has its headquarters inVienna since 1965, and hosts regu-lar meetings among the oil ministersof its member nations.

India signed LoanAgreement with ADB

The Government of India and theAsian Development Bank (ADB) on15 December 2011 signed an agree-ment for the first tranche (113 mil-lion US dollars) under the 350 mil-lion US dollars Himachal PradeshClean Energy Transmission Invest-ment Programme, which is a multi-tranche financing facility. The totalestimated investment cost of theprogramme is 437.85 million USdollars which includes financing of87.85 million US dollars by the stategovt. of Himachal Pradesh. Theprogramme is for the upgradation oftransmission system to helpHimachal Pradesh take greater ad-vantage of its vast hydropower re-sources. It will help expand the sup-ply of power to industry, commerceand households, within and outsidethe state, and thereby contribute tojob creation and poverty reduction.

US Mission in Iraq ended

The US military officially ended itsmission in Iraq on 15 December2011. As per Pentagon statistics,approximately 4487 US soldiers losttheir lives in Iraq war, with another32226 Americans wounded in action.

WWW.UPSCPORTAL.COM 29

UPSCPORTAL Current Affairs : http://upscportal.com/civilservices/current-affairs

Integrated Guidance Programme for IAS (Pre) - 2012http://upscportal.com/civilservices/online-course/integrated-free-guidance-programme

Despite the official declaration ofending its mission in Iraq, US mili-tary still has two bases in Iraq and4000 troops. While in 2007, therewere 505 bases and more than170000 troops. However, these twomilitary bases will be closed and thefinal US troops will be withdrawn by31 December 2011.

Veiled Women barredFrom taking Oath of

Citizenship by Canada

The Government of Canada an-nounced on 12 December 2011 thatwomen who aspire for Canadian citi-zenship can’t wear burqa or Islamichijab when they take oath of citizen-ship. Belgium, France, Australia andNetherlands have also bannedwomen from covering their face inpublic. The government receivedcomplaints from lawmakers andjudges that they found it difficult toknow whether women who maskedtheir faces were actually reciting theoath or not.

4th International TaxDialogue

The 4th International Tax Dialogue(ITD), a global conference on Tax andInequality was inaugurated by theFinance Ministry of India from 7December to 9 December 2011. In-dian Finance Minister PranabMukherjee inaugurated the three-day Tax meet. More than 400 seniortax policymakers from almost 90countries attended it includingDeputy Managing Director of theInternational Monetary Fund – MinZhu, World Bank Vice President –Otaviano Canuto, and Deputy Sec-retary General of the Organization

for Economic Cooperation Develop-ment – Rintaro Tamaki. The ITD is ajoint initiative of variousorganisations working on tax issues– the IMF, the OECD, the World Bank,the European Commission, the IADB,and CIAT. This conference providesan opportunity to address how taxa-tion can be seen as a part of thesolution to growing inequalities inincome and wealth around theworld. The use of plenary and paral-lel sessions, with active country par-ticipation, allows a peer dialoguebetween Ministers of Finance andHeads of Revenue Administrationinternationally.

Issued discussed• Design of growth-enhancing andequitable tax systems• Administrative challenges, issuesand solutions for fair tax systems• Income taxes, progressivity andinequality across regions• Fair tax systems: vital for state-building and an exit from aid depen-dency• Informality, inequality and the Roleof the tax systems• Gender friendly tax systems andinequality• Taxation of elites and inequality

Highlights• More global cooperation is re-quired to tackle black money in In-dia: annual illicit capital outflowsfrom emerging and developingeconomies has been estimated at$725-810 billion• Direct Taxes Code (DTC) has beenproposed in India: DTC seeks to re-place the archaic Income Tax Act,1961 and thereby modernize thetaxation regime. It’s supposed to beimplemented from 1 April 2012• Uruguay signed 7 new agreements

providing for the exchange of taxinformation.

Japan decided toImpose Sanctions

Against Iran

Japan announced on 9 December2011 that it is imposing a freshround of financial sanctions againstIran. Japan will freeze the assets of106 organizations, one individualand three Iranian banks. More than 350 Iran-based entitiesare now subject to Japanese sanc-tions. However, the move will notrestrict imports of Iranian crude oil,a step many Western nations havebeen urging. Resource-starved Ja-pan relies on Iranian oil for 10 per-cent of its energy supply.

New Sanctions againstSyria by Turkey

Turkey on 8 December 2011 an-nounced a new set of sanctionsagainst Syria. Turkey would impose30 percent tax on goods comingfrom Syria. The move as a responseto Damascus imposing 30 percentcustoms duty on goods importedfrom Turkey besides suspending thefree trade agreement with Ankara. Turkey earlier announced a series ofeconomic sanctions on Syria for itscrackdown on anti-regime protests.These include an immediate ban ontransactions with the Syrian Govern-ment and its central bank and freez-ing the Syrian government assets inTurkey. Turkey has also been seek-ing alternative routes to bypass Syriafor trade with the Middle East. TheTurkish government was consideringthree alternative routes throughEgypt’s Alexandria, Lebanon andIraq to bye pass Syria.

WWW.UPSCPORTAL.COM30

UPSCPORTAL Current Affairs : http://upscportal.com/civilservices/current-affairs

Integrated Guidance Programme for IAS (Pre) - 2012http://upscportal.com/civilservices/online-course/integrated-free-guidance-programme

Austerity budgetFor 2012 Approved in

Greece

Greek Parliament on 7 December2011 approved a budget for 2012pledging tough fiscal goals de-manded by European Union partnersin return for fresh loans. The aus-terity budget projects a modest pri-mary surplus excluding interest pay-ments on debt. A broad majority ofthe parties backing LucasPapademos' caretaker administra-tion secured the economicblueprint's passage by 258 votes to41, after the vote concluded. Greekhas been relying on loans from othereurozone countries and the IMF (In-ternational Monetary Fund) sinceMay 2010. In return, Greece cutsalaries and pensions and hike taxesto reduce budget deficits.

New InternationalBuddhist Organization in

India to be setup

The first Global Buddhist Congrega-tion which was aimed to provide ajoint platform to Buddhist commu-nities across the world and also toimpact geo-politics in Asia was con-cluded in New Delhi on 30 Novem-ber 2011. The four-day congregationdecided to set up a new interna-tional Buddhist organization in India.It was attended by heads and repre-sentatives from Buddhist Sanghas,national Buddhist federations, orga-nizations and institutions from 46different countries of the world. Theinternational Buddhist organizationin India is to serve as a common plat-form for Buddhists worldwide. Thecongregation in India was organized

by the Asoka Mission. The new bodywas named as the International Bud-dhist Confederation which was de-cided to be based in India as Bud-dhists all over the world recognizeIndia as the home of Buddha. Thismove could be proved to be an im-portant one for a diplomatic rowbetween India and China.

WWW.UPSCPORTAL.COM 31

UPSCPORTAL Current Affairs : http://upscportal.com/civilservices/current-affairs

Integrated Guidance Programme for IAS (Pre) - 2012http://upscportal.com/civilservices/online-course/integrated-free-guidance-programme

Foreign Exchange Re-serves slipped below

$300-billion

According to data released by theReserve Bank on 6 January 2012,foreign exchange reserves fell byover $4.18 billion to $296.69 billion,slipping below the long-held $300-billion mark in te week ended 30December 2011. The drop in the re-serves was attributed to the fall inthe core foreign currency assets(FCAs) and gold reserves. The over-all reserves slipped by $1.23 billionto $300.86 billion in the previousreporting week. FCAs, a major com-ponent of the forex kitty, fell by $2.72billion to $262.93 billion for the weekended 30 December. Gold reserveswere down by $1.42 billion to $26.62billion. Faced with massive volatil-ity in the rupee the value of whicheroded by 20 per cent since August2011, the apex bank resorted sell-ing dollar reserves to limit or arrestthe depreciation. The reserves de-clined by $24 billion since early Sep-tember 2011. The foreign exchangekitty stood at $321 billion on 2 Sep-tember 2011. The dip was primarilydue to revaluation in foreign currencyassets and a fall in the value of goldreserves.

Two strong reasons was identifiedfor the fall in the reserves:• The first one is the continuous in-tervention by the regulator to curb

Economic Issuesextreme volatility in the rupee. • The second reason is the dollarhas been appreciating against allcurrencies.

Special drawing rights (SDRs) andthe reserve position in the Interna-tional Monetary Fund (IMF) also fell.While SDRs came down by $19 mil-lion to $4.4 billion, the reserve posi-tion in the IMF was down by $12million to $2.7 billion. Some expertsopined that the dip in foreign ex-change reserve also resulted fromthe payout on account of governmentexpenditure might be accentuatingthe fall. While during September-December 2011, fall in the foreignexchange reserves was around $24billion, the reserves fell by $34 bil-lion in the comparable period of2008.

ADB Loan to FinanceRoad Projects inNaxal-hit Areas

The Union government in January2012 cleared an external loan to fi-nance part of the programmelaunched by the Ministry of RuralDevelopment in left wing extrem-ism-affected villages. The clearanceis for a loan of $500 million from theAsian Development Bank (ADB) tospeed up construction of rural roads.Union Ministry of Rural Develop-ment (MoRD) issued directions fornegotiating and early signing of the

loan, which his Ministry to gatherresources to give thrust to thePradhan Mantri Gram Sadak Yojana(PMGSY). The ADB, which has al-ready extended a loan of $800 mil-lion was petitioned with a fresh pro-posal for rural connectivity invest-ment programme to construct orupgrade 7000 km of roads connect-ing eligible habitations in Maoist-affected States of Bihar,Chhattisgarh, Madhya Pradesh,Odisha, West Bengal, besidesAssam where too the PMGSY hasprogressed with little to cheer.

The demand for the loan was madein the backdrop of the MoRD's multi-winged programmes in the left wingextremism-affected areas, underwhich Central forces assist execu-tion of welfare and developmentschemes to wean the local peoplefrom the path of naxalism. The MoRDhas been providing incentives andassistance to the local people, par-ticularly tribals, to reduce povertyand ensure economic growth of theregion. Rural connectivity is consid-ered pivotal to the success of thisstratagem. As per the programmeproposed by the MoRD, the Uniongovernment will supplement with acontribution of $127.6 million, inaddition to the $5000 million to fi-nance the project that includes set-ting up of training and research cen-tres pertaining to rural roads.

WWW.UPSCPORTAL.COM32

UPSCPORTAL Current Affairs : http://upscportal.com/civilservices/current-affairs

Integrated Guidance Programme for IAS (Pre) - 2012http://upscportal.com/civilservices/online-course/integrated-free-guidance-programme

IRDA introduced UniformAsset-liability

Management Norms

Insurance regulator IRDA on 4 Janu-ary 2012 introduced uniform asset-liability management norms for mar-ket players to ensure their solvency.Insurance Regulatory and Develop-ment Authority (IRDA) announced abroadly-defined uniform frameworkfor reporting asset liability manage-ment activities adopted by life andnon-life insurance companies. Theregulator also asked firms to under-take stress tests to ascertain theirability to meet financial obligationsin the event of a crisis. IRDA has is-sued these guidelines to bring aboutuniformity in the ALM norms beingfollowed by both life and non-lifeinsurance companies.

IRDA guidelines: The IRDA guide-lines require the ALM (asset liabil-ity management) policy to be ap-proved by the board of the insurer.Such board-approved policy is to besubmitted to the IRDA within 90days. While approving the ALMpolicy, the board is to take into ac-count the asset-liability relation-ships, the insurer's overall risk tol-erance, risk and return needs, sol-vency positions and liquidity require-ments. The guidelines also make itmandatory for the board to fre-quently review the ALM policy of theinsurer. Any change in the policymust be reported to the regulator.Under the uniform framework, insur-ers have to put in place an effectivemechanism to monitor and managetheir asset-liability positions. Theobjective is to ensure that their in-vestment activities and assets po-

sitions are in sync with their liabili-ties, risk profiles and solvency posi-tions. The guidelines, which wouldcome into effect from 1 April 2012,make it mandatory for insurancecompanies to prepare an ALM policyas well as get it approved by the In-surance Regulatory and Develop-ment Authority (IRDA) by end ofMarch 2012.

Benefits of ALM Policy

The Asset-Liability Management(ALM) norms are critical for thesound management of the financesof the insurers that invest to meettheir future cash flow needs andcapital requirements. The ALMpolicy will enable the insurers tounderstand the risks they are ex-posed to and develop ALM policiesto manage them effectively. TheALM can be used to measure theinterest rate risk faced by insurers.

No Floating InterestRates on Small Savings

Schemes

The Finace Ministry on 4 January2012 clarified that the rates appli-cable on small savings instrumentsschemes would be announced onApril 1 each year and the rate wouldremain valid till the maturity of thescheme. The Ministry stated thatbarring the Public Provident Fund(PPF), the rates of interest on allsmall savings schemes will remainfixed throughout the tenure of in-vestment. To clear the confusionover the returns on investment insmall savings schemes, the FinanceMinistry pointed out that the rateprevailing at the time of investmentswill remain fixed and unchanged till

the maturity of the investment. Anyrevisions in interest rates in the sub-sequent years would only be appli-cable to the investments made inthe relevant period.However, therate of interest for the 15-year PPFscheme would not remain fixed forthe entire period as the interest ac-cruals in the PPF account each yearwould vary, depending on the inter-est rate announced for that particu-lar year. For PPF, the interest ratefixed every year will be applicableto all PPF accounts.

The government had hiked the inter-est rates on small savings depositsschemes of various maturities witheffect from 1 December 2011 tochanel the outflow of funds fromsmall savings schemes administeredby the National Small Savings Fund(NSSF) in view of the investor pref-erence for bank term deposits.

The clarification from the FinanceMinistry came in the face of fearsthat the revision of interest rates onsmall savings schemes from 1 De-cember 2011, are floating rates andthat the rates will undergo changein sync with fluctuations in yields ongovernment securities. It had alsohiked the interest rates on PPF de-posits from 8 per cent to 8.6 per centwhile raising the ceiling on annualcontributions to the fund to Rs.1 lakhfrom Rs.70000. Interest rates onPost Office Savings Accounts rose to4 per cent from 3.5 per cent. Simi-larly, interest rates on deposits ofvarious maturities of one year, twoyears and five years too were raisedfrom December.

WWW.UPSCPORTAL.COM 33

UPSCPORTAL Current Affairs : http://upscportal.com/civilservices/current-affairs

Integrated Guidance Programme for IAS (Pre) - 2012http://upscportal.com/civilservices/online-course/integrated-free-guidance-programme

DoT emphasised on Ca-pacity Constraint

The report of the Department of Tele-communications, prepared by theworking group on Telecom for the12th F ive Year Plan (2012-17),emphasised on the the urgent needfor creating a secure infrastructurefor government communication. Thesecurity of governemnt communica-tion need to be ensured by an alter-native mechanism because neitheris it advisable to wait for such amandating and capacity building noris it appropriate to rely on the levelof security provided by such ascheme given that communicationwill continue to remain in the opendomain of Internet. The report high-lighted the capacity constraint of thecurrent Indian system that allowstraffic to flow outside the country.As per the report an investment ofRs.450 crore is needed to developand deploy a pan-India secure net-work and network-based servicessuch as email, VoIP mobile commu-nication through a survivable andavailable network architecture forsecured communication for govern-ment use. Rs.300 crore is to be spenton establishing a fixed and wirelesscommunication system for high se-curity of classified government com-munication, Rs.100 crore would bespent for operational expenditure forthe next five years and Rs.50 crorefor research and development.

The Defence Research and Develop-ment Organisation (DRDO) has aninternal system of communicationthat acts like a private Internet sys-tem, connecting various centresworking on crucial defence projects,thus securing it from any kind ofhacking or interception. Also, the

Defence Scientific Information &Documentation Centre (DESIDOC)'smain function is collection, process-ing and dissemination of relevanttechnical information for DRDO sci-entists. It also provides secure e-mail and Internet connectivity toDRDO labs through the networks ofERNET (of Dept. of Electronics) andNICNET (of the Planning Commis-sion).

Liquidity Eased by RBI

The Reserve Bank of India on 3 Janu-ary 2011 decided to conduct an openmarket operation (OMO) to injectmore liquidity into the system. TheRBI will buy up to Rs 12000 crore ofgovernment bonds via open marketoperations on 6 January 2012, includ-ing the 10-year paper which till re-cently was the benchmark paper. Thecentral bank has decided to ease li-quidity by buying back gilts for anamount of R10,000 crore in the back-drop of banks accessing the ReserveBank of India (RBI)’s borrowing win-dow for more than R1 lakh croreeach day. RBI announced an auctionfor R10,000 crore worth of bonds,otherwise known as open marketoperation (OMO).

The OMO announcement came af-ter the market trading hours. theReserve Bank of India decided toconduct open market operationsconsistent with the stance of themonetary policy and based on thecurrent assessment of prevailingand evolving liquidity conditions.Banks have been borrowing in ex-cess of R 1 lakh crore a day from theRBI's liquidity adjustment facility(LAF) or repo window. The liquiditydeficit in the system in recent weekshas been way beyond the limit of 1%

of the net liabilities of the system,or around Rs 55000 crore.

SEBI allowed Auctioningof Securities through

Stock Exchanges

The capital market regulator SEBI on3 January 2012 allowed auctioningof securities through stock ex-changes and introduced a newmethod for institutional placementof stocks. The move was directed tokick-start government's divesmentprogramme as well as help promot-ers of companies to sell a part oftheir holdings. As per the auction-ing route, a special window can beused by promoter stakeholders tosell at least 1% of the paid-up capi-tal of a company. It is similar to theblock-deal mechanism for secondarystock market transactions, but withlesser restrictions. The auctionmethod can be only used by promot-ers of top 100 companies based onaverage market capitalisation forsale of their stakes.

Institutional Placement ProgrammeUnder the institutional placementprogramme (IPP), shares can be soldonly to qualified institutional buy-ers. Exchanges will provide a sepa-rate window for the offer for sale ofshares which will co-exist with thenormal trading hours. promoter orpromoter group of companies how-ever will not be allowed to bid forthe shares. Allotment will be doneeither on price priority or clearingprice basis proportionately andwould be overseen by the ex-changes. SEBI’s measure is consid-ered to be very progressive step to-wards creating an organised andeffective mechanism that will notonly facilitate fund raising but also

WWW.UPSCPORTAL.COM34

UPSCPORTAL Current Affairs : http://upscportal.com/civilservices/current-affairs

Integrated Guidance Programme for IAS (Pre) - 2012http://upscportal.com/civilservices/online-course/integrated-free-guidance-programme

assist companies to comply with thelisting norms in a non-disruptivemanner. There shall be at least 10allottees in every IPP issuance. Nosingle investor shall receive allot-ment for more than 25% of the offersize.

Bill to Amend ForwardContracts Regulation

Act 1952

The Parliamentary Standing Commit-tee submitted its report on a bill toamend the Forward Contracts Regu-lation Act 1952. ParliamentaryStanding Committee on consumeraffairs, food and public distribution,chaired by Congress MP VilasBaburao Muttemwar, submitted itsreport on the FCRA (Amendment) Bill2010 to Parliament on 22 December2011. The current department-re-lated standing committee (DRSC),set up in 2009, was asked by the LokSabha speaker in December 2010 toprepare a report on the bill and sub-mit it to the Lok Sabha Secretariat.The committee in its report recom-mended a doubling of the maximumpenalty for trading rule violations toRs 50 lakh. The standing committeereport suggested raising the upperlimit on penalties for offences likeinsider trading to Rs 50 lakh from Rs25 lakh stipulated in the ForwardContracts Regulation Act (FCRA)Amendment Bill 2010. Insider trad-ing involves using unpublished pricesensitive information for personalgain.

ReportThe report recommended that op-tions be introduced for the benefitof stakeholders. The inclusion of theclause was one of the reasons why

the bill in its earlier avatar duringthe UPA I regime faced resistance.Those who had opposed the bill thenespecially the Left parties arguedthat options would increase specu-lation in commodities. The reportsuggested that options will actuallymake it easier for farmers andsmaller users to participate in thederivatives market as trading lotsizes will be lower than in futurescontracts, where the minimumtraded quantity for most farm prod-ucts is 10 tonne.

investing in an option also tends tominimise losses as only the premiumto buy (call option) or sell (put op-tion) is forgone in the event of pricesmoving adversely. a futures positiontaken by a trader is on the other handmarked to market daily. Marking tomarket involves daily settlement ofthe difference between the prioragreed price and the daily futuresprice. It can thus lead to huge lossesalongside supernormal profits.

Implementationof Levy on Railway

Freight Service deferred

The implementation of levy on rail-way freight service was put off onceagain in the backdrop of high infla-tion. The levy is now likely to comeinto force from 1 April instead of 1January as announced earlier. Thelevy on transport of goods by railwas deferred for the sixth time. Fi-nance Minister Pranab Mukherjee inthe 2010-11 Union Budget hadbrought transport of goods by rail-way under the service tax net from1 April 2010. However, the proposalwas vehemently opposed by RailwayMinistry fearing adverse impact on

goods movement, forcing the gov-ernment to defer it repeatedly. Rail-way Ministry is of the opinion thatany levy on freight service wouldadversely impact the industry. Move-ment of coal and cement among oth-ers goods would become costlierwith the imposition of service tax.

RIL’s $1.529 billion In-vestment Plan approved

The Union government on 3 January2012 approved Reliance Industries'(RIL) $1.529 billion investment planfor developing four satellite fields inthe flagging KG-D6 block. RIL’s in-vestment plan will boost falling out-put in the Krishna-Godavari BasinKG-D6 block. The investment pro-posal was signed by the three part-ners in the block- RIL, UK's BP Plcand Niko Resources of Canada andthe representative of DGH. The KG-D6 block oversight committee, whichincludes officials from the Oil Min-istry and its technical arm, the Di-rectorate General of Hydrocarbons(DGH), met for the third time in threemonths on 3 January to finally ap-prove the proposal. The MC ap-proval, which is the final approvalan operator needs before beginningwork, put a cap on the cost of devel-oping the four fields that surroundthe currently producing Dhirubhai-1and 3 (D-1 & D-3) fields in the KG-D6 block. The cost cannot vary bymore than 15%. The MC had at itstwo previous meetings in Novemberand December 2011 refused to ap-prove the field development plan(FDP) for the Dhirubhai-2, 6, 19 and22 (D-2, D-6, D-19 and D-22) fieldsafter the government representativeraised certain objections. RIL agreedto cap spending on the four fields at$1.529 billion, plus or minus 15%.

WWW.UPSCPORTAL.COM 35

UPSCPORTAL Current Affairs : http://upscportal.com/civilservices/current-affairs

Integrated Guidance Programme for IAS (Pre) - 2012http://upscportal.com/civilservices/online-course/integrated-free-guidance-programme

The four fields can produce 10 mil-lion cubic metres of gas per day by2016, which will help shore up out-put from the block, which has seena 35% decline in production in thepast 15 months.

Export Duty on Iron OreExports raised

The Union government raised the advalorem duty (export duty) on ironore exports to 30 per cent from 20per cent. The decision is expectedto step up finances of cash-strappedgovernment by around Rs 8500-9000crore. The Federation of Indian Min-eral Industries, the apex body ofminers however complained that In-dian ore would no longer be com-petitive internationally. The increasein export tax could lower the profitmargin of Sesa Goa Ltd., India's larg-est iron-ore exporter by volume.Shipments from the South Asiancountry decreased 28% betweenApril and November to 40 milliontons, according to the Federation ofIndian Mineral Industries.

Volumes were hit by a mining ban inthe southern state of Karnataka, afreeze on sale of old stocks in west-ern Goa state and transport bottle-necks in the eastern state of Orissa.India exported 97.64 million tons ironore in 2012. The Supreme Court hadin early 2011 banned mining in themajor iron-ore producing districts ofKarnataka to prevent illegal miningand environmental damage. In Goa,moves to reduce environmental im-pact and illegal mining affected pro-duction. The two states account foraround 70% of India's iron-ore ex-ports.

RBI allowed NRIs tohedge their CurrencyRisk in respect of ECB

The Reserve Bank of India on 29December 2011 allowed non-resi-dents to hedge their currency risk inrespect of external commercial bor-rowings (ECB) denominated in ru-pees, with AD Category-I (authoriseddealer) banks in India. The apexbank mentioned that the amount andtenor of the hedge cannot not ex-ceed that of the underlying transac-tion. Besides, it should be in conso-nance with the extant regulationsregarding tenor of payment orrealisation of the proceeds. TheNGOs (non-Governmentorganisations) engaged inmicrofinance activities were permit-ted to avail themselves of ECBs des-ignated in Indian rupee under theautomatic route from overseasorganisations and individuals as perthe ECB guidelines. According to theRBI, once cancelled the contractscannot be rebooked. The contractsmay, however, be rolled over on orbefore maturity. On cancellation ofthe contracts, gains may be passedon to the customer.

Financial Packageof Rs 257 crore

Recommended for NEPA

The Board for Reconstruction of Pub-lic Sector Enterprises (BRPSE) rec-ommended a financial package of Rs257 crore to revive the sick news-print unit NEPA. The initiative wastaken after the government shelvedits plan to divest majority stake inthe company and revive it through ajoint venture with the privatesector. The government holds

97.75% in NEPA. BRPSE alsofavoured the waiving off NEPA's in-terest and statutory dues worth Rs304 crore. The Board for Reconstruc-tion of Public Sector Enterprises sug-gestion was made to NEPA's admin-istrative ministry, the Department ofHeavy Industry. BRPSE suggestedthe department to infuse Rs 175crore in the form of fresh equity tomeet part-finance of total expendi-ture of Rs 318 crore for Revival andMill Development Plan (RMDP).It also recommended sanction ofnon-plan loan of Rs 22.48 crore to-wards cash loss from production forthe first year of production. TheMadhya Pradesh-based companysuffered a loss of Rs 70.40 crore in2010-11.

Union GovernmentQuadrupled Limits on

Loans

The Union government quadrupled(four times) the limits on loans thata bank’s internal committee can ap-prove. The giovernment’s move isexpected to quicken credit clearanceat 26 state-run banks, including theBank of Baroda and Punjab NationalBank. The government directedbanks to set up a credit approvalcommittee — comprising chairman,executive directors and three chiefgeneral managers who is to handlecredit, finance and risk managementfunctions. The group can approvecredit proposals up to Rs 400 crore.Currently, any loan above Rs 100crore has to be vetted by the man-agement committee of the board,which meet once a month, or 20days. Under the old regime, a man-agement committee of the board,which included a Reserve Bank ofIndia nominee and two independent

WWW.UPSCPORTAL.COM36

UPSCPORTAL Current Affairs : http://upscportal.com/civilservices/current-affairs

Integrated Guidance Programme for IAS (Pre) - 2012http://upscportal.com/civilservices/online-course/integrated-free-guidance-programme

directors appointed by rotation, thebank’s chairman and managing di-rector and executive directors, tookthese decisions. The prescribed limit is applicable on Category Abanks with a business of Rs 3 lakhcrore, while smaller public sectorbanks can use the same structure toapprove loans up to Rs 250 crore. Ifa loan under consideration is higherthan these limits, it would be taketo the management board.

RBI Ordered Banks toKeep More CapitalFor Investments inFinancial Entities

The Reserve Bank of India (RBI) or-dered banks to set aside more capi-tal for their investments in financialentities such as insurance with anobjective to strengthens the ringfence around banks. However themove can strain capital resourceswhich are increasingly gettingscarce. The RBI proposed the raisein risk weight to prevent banks fromgetting affected because of theirholdings in other finance entities.The banks are to set aside 25% morecapital following the central bankraise of the risk weight for buying orholding of equity in financial enti-ties. Banks’ investments in paid-upequity of financial entities, even ifthey are exempted from the capitalmarket exposure norms, will thus beassigned a 125 percent risk weight.The proposal is expected to leadbanks, which at present set aside Rs 9 for every Rs 100 of investment infinancial entities, to keep asideabout Rs 11. RBI opined that con-solidation in the banking sectorwould pave the way for stronger fi-nancial institutions with the capac-

ity to meet corporate and infrastruc-ture funding needs, and to rescuedistressed lenders. However, it pre-scribed a non-operative bank hold-ing company structure to avoid cre-ation of complex institutions.

Uniform Licence Fee of 8% of AGR recommended

Telecom Commission, the decision-making body of the Department ofTelecommunications recommendeda uniform licence fee of 8 per centof adjusted gross revenues (AGR).Uniform license fee of 8 % was rec-ommended as against the prevalentrate of 6-8 per cent depending uponthe type of service and circle a firmis operating. The Commission’smove will put more financial pres-sure on telecom operators. TheTelecom Commission is also likely toimpose one-time charges on extra2G spectrum that operators havebeen holding beyond the contractuallimit of 6.2 MHz. The Telecom Regu-latory Authority of India (TRAI) alsorecommended a charge of 8 per centof AGR for deciding the license fee.TRAI recommended that each MHzof additional spectrum (beyond 6.2MHz of contractual radio waves) heldby operators should have one-timecost of Rs.4571.87 crore (pan-India).The Telecom Commission futherspecified that in future additionalspectrum would be allotted throughthe auction route.

The Telecom Commission had ac-cepted the TRAI recommendation onmerger and acquisition (M&A),which according to the Commissionwould help consolidation of the mo-bile sector that currently has arounda dozen players, the highest in theworld.

The Commission accepted the TRAI'srecommendations on spectrum shar-ing as well. The spectrum sharing would be permitted between any twolicensees holding spectrum, subjectto the condition that the total band-width would not cross the permis-sible limit under mergers. The per-mission would be for five years, sub-ject to renewal for one more term offive years. As per the new regula-tion, pre-2008 operators will need topay one-time additional spectrumcharge. The operators are currentlypaying about 6-10%, depending onthe circles they operate in. The newfigure of 8% is much more than whatthe TRAI had earlier suggested at6%.

Cheques to be issuedconforming to ChequeTruncation System 2010

Standard

The Reserve Bank on 27 December2011 directed all banks to issuecheques conforming to Cheque Trun-cation System (CTS) 2010 standardwith uniform features from 1 April2012 onwards. All banks providingcheque facility to their customerswere advised to issue only CTS-2010standard cheques not later than 1April 2012 on priority basis in north-ern and southern region. The two-northern and southern region will bepart of the northern and southernCTS grids respectively. CTS-2010standard cheques are to be issuedby banks across the country by 30September 2012 through a timebound action plan. The Indian BanksAssociation ( IBA) and National Pay-ment Corporation of India ( NPCI)were vested with the responsibilityof coordinating and implementing

WWW.UPSCPORTAL.COM 37

UPSCPORTAL Current Affairs : http://upscportal.com/civilservices/current-affairs

Integrated Guidance Programme for IAS (Pre) - 2012http://upscportal.com/civilservices/online-course/integrated-free-guidance-programme

the uniform cheque standard acrossthe country by all participatingbanks.

Need for the CTSThe introduction of new cheque stan-dards 'CTS 2010' was needed onaccount of several developments inthe cheque clearing: growing use ofmulti-city and payable-at-parcheques at any branch of a bank,increasing popularity of Speed Clear-ing for local processing of outstationcheques and implementation of gridbased Cheque Truncation System(CTS) for image-based cheque pro-cessing etc.

Advantage of the CTSThe new cheque standard CTS 2010with set of minimum security fea-tures will ensure uniformity acrossall cheque forms issued by banks inthe country as well as help present-ing banks while scrutinising andrecognising cheques of draweebanks in an image-based process-ing scenario, RBI said in a notifica-tion. The homogeneity in securityfeatures is also expected to act as adeterrent against cheque frauds.Also, the standardisation of fieldplacements on cheque forms wouldenable straight-through-processingboth under CTS and MICR clearing.

RBI tightened theCapital AdequacyNorms for all NBFCs

The Reserve Bank on 26 December2011 tightened the prudential normsfor the non-banking financial com-panies (NBFCs) under which theNBFCs will have to account for riskstowards off-balance sheet itemswhile computing capital adequacy

requirement. The NBFCs can thusparticipate in the credit default swapmarket only as users. As users, theNBFCs would be permitted only tohedge their credit risk on corporatebonds they hold. However, they arenot permitted to sell protection. Theyare therefore not permitted to enterinto short positions in the credit de-fault contracts. NBFCs are howeverpermitted to exit their bought CDSpositions by unwinding them withthe original counter-party or by as-signing them in favour of buyer ofthe underlying bond. RBI also tight-ened the capital adequacy norms forall NBFCs. The rule tightening exer-cise comes in the wake of theirstepped-up exposure to off- balancesheet items. The RBI revised capitaladequacy norms for non-banking fi-nance companies (NBFCs) with anobjective to improve their capacityand help manage off-balance sheetexposure. The regulatory frameworkwas expanded to have greatergranularity in the risk weights andcredit conversion factors for differ-ent types of off-balance sheet items.In the normal course of their busi-ness, NBFCs are exposed to creditand market risks due to asset-liabil-ity transformation as the Indian mar-kets are now more integrated withglobal ones.

Tea Imports Declined

According to the Tea Board data re-leased in December 2011, tea im-ports declined by 14 per cent to 9.91million kg in the April-October pe-riod of 2011. Imports of the brew fellby 15% to 14.15 million kg from16.57 million kg in January-October2010. The country had imported11.55 million kg of tea in same pe-riod in 2010. India, the world's larg-

est consumer of tea imports tealeaves solely for re-export to othercountries. The dip in imports there-fore signals lower re-exports. Theinbound shipments of tea from mostcountries, except Argentina, Iran andthe UK, registered a decline in thefirst seven months of the 2011-12financial year. India imports tealargely from Kenya, Malawi, Nepal,Argentina, Iran, Sri Lanka, China andIndonesia, among other countries.India is the second-biggest producerof tea in the world and accounts forabout 28% of global output and 14%of trade. There are around 1600 teaestates in India and the industryemploys more than two millionpeople.

IT Sector created mostJobs in the last 5 Years

The information technology (IT) sec-tor led by the top three listed com-panies, TCS, Infosys and Wipro, cre-ated the most jobs in the five yearsending 2011 compared with othersectors. Increased employment inthe sector was boosted by an overtwo-fold jump in aggregate revenue.The data is based on the hiring trendof a sample of 600 listed companiesthat reported annual financials alongwith headcount information since2006. The findings provide a reflec-tion of the changing trend in India’sGDP composition. There was amarked shift from the agrarianphase to the services phase. Thedata revealed that the proportion ofservices sector jobs in the totalheadcount of the sample rose to46.5% in 2011 from 41.8% in 2006.The sample companies also ex-panded the aggregate headcount by48% to 43.8 lakh employees be-tween 2006 and 2011.

WWW.UPSCPORTAL.COM38

UPSCPORTAL Current Affairs : http://upscportal.com/civilservices/current-affairs

Integrated Guidance Programme for IAS (Pre) - 2012http://upscportal.com/civilservices/online-course/integrated-free-guidance-programme

IT SectorThe analysis by Economic Times (ET)Intelligence Group of the trend in jobcreation by the organised sector re-flected the rising clout of servicescompanies. Of the 14.3 lakh jobscreated between Financial 2006 and2011, over 8 lakh (56%) were addedby companies in the services sec-tors, which includes banking and fi-nance, healthcare, hospitality, tech-nology, telecom, trading and retail.These companies created four outof seven jobs in the country over thepast five years thereby outpacing themanufacturing sector. IT sector play-ers led the service sector, adding asmany as 4.5 lakh employees. TCS,Infosys and Wipro together added2.4 lakh people, or more than halfthe total additions for the sector.

First Pan-IndiaSatellite Survey

The first pan-India satellite surveyjointly commissioned by IndianSugar Mills Association (ISMA) andthe National Federation of Co-opera-tive Sugar Factories Ltd (NFCSF)pegged the cane area for 2011-12crop year starting October at 51.82lakh hectares (lh). For the first timethe survey was carried out State-wise and district-wise for the areaunder sugarcane, through satellitemapping on such a large-scale. Sat-ellite images of the cane area pro-cured from the National RemoteSensing Agency, Hyderabad, wereanalysed using the Geographical In-formation Systems software bySouth India-based firm. A satellitemapping of sugarcane acreage car-ried out for the first time across In-dia showed an increase of 3% overthe government estimate. The data

showed acreage to be the same inUttar Pradesh and higher by 3% inMaharashtra as compared to the fig-ures projected by the ministry ofagriculture for the two states. Ac-cording to the satellite data, acre-age for 2011-12 has been estimatedat 51.82 lakh hectare. The figure ishigher than the estimates mademanually by the sugar industry at50.79 lakh hectare, by the ministryof agriculture at 50.93 lakh hectareand the ministry of food and publicdistribution at 50.25 lakh hectare.India joined the ranks of major sugarproducers such as Brazil and Thai-land in leveraging remote sensingtechnology to estimate the canearea.

Illegal Mutual RoamingAgreements to be Scrap

India's telecom ministry on 22 De-cember 2011 informed mobile phoneoperators that they must scrap ille-gal mutual roaming agreements thatallow them to provide seamless na-tionwide 3G services. As per theministry, the pacts that let the op-erators offer 3G services outsidetheir licensed zones are in violationof terms and conditions of their li-cences.

The government complained thattelecom operators wereusing the 3Groaming deals to offer services inareas where they have not paid forthe spectrum. Leading mobile opera-tors such as Bharti Airtel, Vodafoneand Idea Cellular had struck dealswith each other to offer ultra-fast 3Gservices in areas where they did notacquire spectrum in a costly band-width auction in 2010. The firmsentered into the deals because nonecould afford nationwide 3G spec-

trum in the high-priced sale.Bharti has 3G bandwidth in 13 ofIndia's 22 telecom zones while Ideahas access in 11 areas andVodafone's India unit in nine. Theannouncement by the ministry dealta a blow to the companies, whichhad hoped to recover their 3G auc-tion payments by providing high-pre-mium 3G data services across thecountry in India's fiercely competi-tive telecom market.

Earlier operators had noted that incase they could not offer nationwideroaming, the government should re-fund the sums paid for 3G spectrumor restage the auction as it wouldalter the basis on which 3G bidswere made. Third-generation ser-vices, or 3G, allow mobile phoneusers to surf the Internet, video con-ference and download music, videoand other content at a much fasterpace than the current second-gen-eration service offered in India.

The government had reaped $15 bil-lion from auctioning the 3G licencesin 2010. Bharti and Idea paid 123billion rupees ($2.3 billion) for li-cences while Vodafone paid 116 bil-lion rupees. Indian telecom compa-nies which currently generate onlysmall revenues from data servicesexpect the market to grow exponen-tially as less than 10 percent of the1.2 billion population has access toInternet at the moment.

India has some 881.4 million mobileand 33.2 million fixed-line subscrib-ers with total teledensity at 76, upfrom 2.5 in 2000.

WWW.UPSCPORTAL.COM 39

UPSCPORTAL Current Affairs : http://upscportal.com/civilservices/current-affairs

Integrated Guidance Programme for IAS (Pre) - 2012http://upscportal.com/civilservices/online-course/integrated-free-guidance-programme

Moody's upgradedCredit Rating of IndianGovernment's Bonds

Credit rating agency Moody's on 21December 2011 upgraded the creditrating of the Indian government'sbonds from the speculative to in-vestment grade. According to a re-lease issued by the Finance Minis-try, Moody's unified India's local andforeign currency bond ratings atBaa3. The ratings agency initiallyhad separate rating for investorswho choose to buy bonds in foreigncurrency and separate rating forthose who have a rupee exposure.the ratings agency had a Baa3 for-eign currency rating and a Ba1 localcurrency rating till September 2011.

Moody's Investor Service upgradedits local currency rating for Indiangovernment bonds to Baa3 which isinvestment grade as compared tothe earlier Ba1 which is junk orspeculative grade. India's Baa3 rat-ing incorporates credit strengthssuch as a large, diversified economy,robust medium term growth pros-pects and a strong domestic savingspool that facilitates the financingand refinancing of the government'srelatively high debt burden. India'sforeign currency bond ceiling is un-changed at Baa2, and the foreigncurrency bank deposit ceiling isBaa3. The local currency bond andbank deposit ceilings are unified atA1. In addition, the government'slocal currency short-term rating hasbeen changed to P-3 from NP, indi-cating the government's ability torepay short-term debts.

Moody's expected India's growthdownturn to persist for the next twoquarters, but the GDP growth would

be above average with respect tothe similarly rated peers. Giving therationale for the upgrade, Moody'smentioned that diverse sources ofIndian growth have enhanced its re-silience to global shocks. Thepresent slowdown could reversesome time in 2012-13, as inflationcools from the current 9 per cent lev-els. Moody’s upgrade is expected to attract Foreign Institutional Inves-tors (FIIs) to the Indian bond marketand boost the gloomy economic out-look. The last time Moody's up-graded any Indian long-term sover-eign debt instrument from thespeculative to investment grade wasin 2004.

India exited from$1 trillion Group

Fears of continuing economic slow-down, lack of decision making at thecentre, rising fiscal deficit combinedwith not-so-impressive revenue col-lections upset Dalal Street investorson 20 December 2011 leading to a204 points loss in sensex to end at15175. The loss of 204 points repre-sented a 28-month low for the in-dex and its fifth consecutive sessionof losses, during which it has lost827 points, or 5.2%. The day’s slidewas triggered by FII selling whichrecorded a net outflow of Rs 526crore, taking the month’s total netoutflow to about Rs 1300 crore. TheIndian rupee which had weakenedfurther to go below 53 to a US dollarlevel again, closed at level of Rs52.89 on 19 December 2011.

The combined effect of the market’sslide and the depreciation of the ru-pee forced India to exit from the se-lect group of countries with a $1 tril-lion market capitalization. With

BSE’s market cap currently at Rs52.53 lakh crore, India’s market capin dollar terms works out to $993billion. On 28 May 2007, when therupee was showing unusual strengthagainst the dollar and hoveredaround the 40 mark that India hadfirst entered the $1-trillion marketcap league. However, lately the twineffect of rupee’s weakness and theslide of the market combined to dragit below the trillion dollar mark. Ac-cording to Bloomberg data, India’smarket cap so far in 2011 went downby 38%, making it the worst per-forming market among the 12 coun-tries that were in the trillion dollarclub.

India's Energy SecurityUnder Pressure

As per a joint study by the Federa-tion of Indian Chambers of Com-merce and Industry ( FICCI) andconsultancy firm Ernst and Young,India's energy security is under se-vere pressure due to reasons likeincreasing dependence on importedoil, regulatory uncertainty and natu-ral gas pricing policies. The reportmentioned that a small pool ofskilled manpower and poor up-stream infrastructure are also exert-ing pressure.

ReportAccording to the report, there existsa dire need to address the supplyissue through a slew of policy re-forms, as well as to launch a mas-sive awareness campaign on thedemand side management, and thepricing of products to incentivise in-vestments for raising domestic pro-duction. As per the Integrated EnergyPolicy of the Government, India'srequirement of primary commercial

WWW.UPSCPORTAL.COM40

UPSCPORTAL Current Affairs : http://upscportal.com/civilservices/current-affairs

Integrated Guidance Programme for IAS (Pre) - 2012http://upscportal.com/civilservices/online-course/integrated-free-guidance-programme

energy is projected to increase from551 million tonne of oil equivalentin 2011-12 to 1823 mt of oil equiva-lent in 2031-32.

The increase in oil price by $10 abarrel is likely to slow the GDPgrowth by 0.2 per cent and may in-flate the current account deficit by0.4 per cent. Also, the recent depre-ciation of the rupee raised the crudeoil imports costs, impacting tradedeficit and domestic inflation. Con-sequently the import of crude oil andoil products rose from $50.3 billionin 2005-06 to $115.9 billion in 2010-11. In the current financial year (tillOctober 2011), oil imports touched$75 billion. The country meets 73percent of its fuel needs through oilimports.

Suggestions MadeThe study sugggested the need tointroduce reform and favourablepolicies for the private sector to se-cure foreign oil and gas assets. In-dia, though has surplus oil refiningcapacity, it still needs major invest-ments to upgrade the existing pro-duction infrastructure. It alsostressed on the need to shore up theenergy security initiatives in an en-vironment of unstable geopoliticalsituation in the Middle East andNorth Africa, from where India gets60 percent of its oil imports.

DGH approved Cairn India’s ProposalThe directorate general of hydrocar-bons (DGH), the technical arm of theoil ministry approved Cairn India’sproposal to commence productionfrom Bhagyam, the second-largestoil field in the Rajasthan block. Thecompany, which currently operatesthe block with a 70% stake, waitedfor a year to obtain approvals to start

production.The DGH gave its approv-als to commence production fromBhagyam, along with both the man-agement committee and the opera-tions committee. The managementcommittee comprises Cairn India,ONGC and representatives of theDGH and the petroleum ministerwhile the OC is composed of only JVpartners Cairn India and ONGC.The representatives of the oil min-istry also gave their in-principle nodto commence production. Currently,Mangala, the biggest of the 18 dis-coveries in the Rajasthan block, isproducing 125000 bpd. Bhagyam istargeted to produce a peak outputof 40000 bpd by the end of 2011,which would help Cairn achieve theapproved peak output of up to175000 bpd. The approvals werestuck for more than a year as previ-ously, the ministry wanted all priorentitlement issues related to theVedanta deal to be resolved beforegiving Cairn a go-ahead to start pro-duction from the oil field.

Indian Mobile PhoneMarket grew 12%

According to the International DataCorporation's (IDC) Quarter 3 2011Mobile Phone Tracker release, theIndian mobile phone market grew12% in units shipped in the July-Sep-tember quarter of 2011 to clock47.07 million units. Year-on-year too,there was a shipment growth of13.8%. The shipments were pro-pelled by the dual-SIM handsets,which grew by 25.2 per cent over theprevious quarter (April-June). Themobile phone shipments witnesseda spurt, as vendors built channel in-ventories ahead of a long festivalseason. Dual-SIM handset ship-ments were notable with a sequen-

tial growth of 25.2% over the previ-ous quarter(april-June quarter).However there was a sharp declinein the mobile service subscriptionadds during July-Sept 2011.

According to the release, the over-all, smartphones shipment for theIndia market showed an impressivegrowth of 21.4% over the previousquarter and 51.5% year-on-year. Thishelped the segment grow its contri-bution to the mobile phone shipmentto 6.5% in third quarter from 5.6% inthe second quarter. From an operat-ing system (OS) perspective, Androidovertook Symbian to emerge as thetop platform in India for the firsttime, with a share of 42.4% of thesmartphone market. The platform(Android) saw a growth of 90% overthe previous quarter. The Apple iOSconsolidated further, with a 3.09%share of the smartphone market,compared to a share of 2.6% in sec-ond quarter (April-June) of 2011.

Repo RateUnchanged at 8.5 %

The Reserve Bank of India on 16December 2011 left its policy rateunchanged at a three-year high of8.5 per cent. RBI paused the hikeafter 13 consecutive rate hikes sinceMarch 2010. The Reserve Bank ofIndia kept its policy repo rate un-changed at 8.5 percent at its mid-quarter review two days after datashowed November wholesale priceindex inflation at 9.11 percent, farlower than the 9.73 percent clockedin October. The RBI also left the cashreserve ratio unchanged at 6 percent,despite market specualtion that itmight cut the ratio in order to boostmarket liquidity. The central banknoted that while inflation remained

WWW.UPSCPORTAL.COM 41

UPSCPORTAL Current Affairs : http://upscportal.com/civilservices/current-affairs

Integrated Guidance Programme for IAS (Pre) - 2012http://upscportal.com/civilservices/online-course/integrated-free-guidance-programme

on the projected trajectory, down-side risks to growth clearly in-creased. It reiterated that furtherrate hikes might not be warrantedas the growth momentum was mod-erating.

Retaining its option to raise ratesagain if inflationary expectationspersist, the RBI mentioned that thetiming and magnitude of further ac-tion would depend on how thingspanned out on the inflation and ru-pee fronts.

RBI’s policy statement stated thatinflation risks remain high and infla-tion could quickly recur as a resultof both supply and demand forces.Also, the rupee remains understress. The central bank that theGDP growth declined to 6.9 per centin the second quarter (July-Septem-ber) from 7.7 per cent in the first(April-June), and key deficit indica-tors worsened mainly due to higherexpenditure and lower revenues.Noting that liquidity conditions weretight, consistent with the policy in-tent, the RBI mentioned that it wouldconduct open market operations asand when it felt the need for it.

SEBI DirectedIntermediaries

Market regulator SEBI on 15 Decem-ber 2011 directed intermediaries notto outsource their core business ac-tivities and compliance functions.According to SEBI, core businessactivities may include execution oforders and monitoring of tradingactivities of clients in case of stockbrokers, dematerialisation of secu-rities in case of depositary partici-pants, investment-related activitiesin case of mutual funds and portfo-

lio managers. Intermediaries wouldbe responsible for reporting any sus-picious transactions to the FinancialIntelligence Unit or other competentauthority in respect of activities car-ried out by the third parties.

Recommendation ofSEBI’s Committee on MF

A committee on mutual funds, con-stituted by SEBI in December 2011recommended to the regulator’sboard to break down the bifurcationwithin the fee structure known asexpense ratio. According to the com-mittee the measure will allow mu-tual funds to manage their expensesbetter and possibly improve theirprofits. In the absence of bifurca-tions within the expense ratio, fundhouses will try to reduce recurringexpenses which, in turn, will increasetheir profitability. Currently mutualfunds are allowed to charge up to2.25% as expense ratio. Of the 2.25%charged as expense ratio, fundhouses are allowed to accept only1.25% as asset managementcharges. The remaining 1% has tobe mandatorily used to meet recur-ring expenses, which include pay-ment of annual trail fees, auditor &registrar charges and dealingcharges to empanelled brokers.Funds with large corpuses currentlycharge 1.75% as expenses charges.There exists scope for these fundsto charge lower expense ratios. TheSEBI panel also suggested that mu-tual funds, with higher assets undermanagement, will have to bringdown their expense ratio proportion-ately. SEBI was suggested to keepoverall scheme expenses unchangedat 2.25% for schemes with assetsunder management not exceedingRs 400 crore. Panel members aslo

proposed an alternative slab system,which will bring down expense ratiowith a proportionate increase in theasset base of the fund. There willbe no change in expense ratios ofdebt funds. The panel suggestedSEBI to come out with wider slablimits which bring down expenses onlarge funds even further. Expenseratio is more than important in debtschemes where the rate of return isnot high. In the case of equity funds,the expense ratio becomes criticalin times of lacklustre markets.

CIL’s Board approvedAcquisition of Stakes

The world's largest coal miner CoalIndia Ltd (CIL's) board on 14 Decem-ber 2011 approved a proposal toacquire stakes in unlisted firmsoverseas, provided the offers werevalid. The proposal was approved inthe wake of Finance Ministry’s ap-proval for the public sector firm toproceed with its plan to acquire un-listed firms overseas. CIL plans totake up three offers - in Australia,Indonesia and the US. The PSU puttogether a war-chest of Rs.6,000crore for acquisition of mines. b CILsought clarifications from the Fi-nance Ministry before entering intodiscussions with owners, having re-ceived proposals offering an IRRbetween 9 percent and 12 percent.CIL had request for a relaxation inthe PSU guidelines stipulating aminimum 12 percent internal rate ofreturn (IRR) on investments. In re-sponse to the request, the FinanceMinistry mentioned that CIL couldproceed with such proposals, if theyare of strategic nature, and subjectto government clearance. CIL alsosought to sidestep the rule that onlythe mines of listed companies

WWW.UPSCPORTAL.COM42

UPSCPORTAL Current Affairs : http://upscportal.com/civilservices/current-affairs

Integrated Guidance Programme for IAS (Pre) - 2012http://upscportal.com/civilservices/online-course/integrated-free-guidance-programme

should be acquired, which was per-mitted by the Finance Ministry.

RBI announced Non-direct Intervention

Measures

The Reserve Bank of India an-nounced non-direct interventionmeasures in the wake of steadyweakening of the rupee against thedollar. The non-direct interventionmeasures are aimed at curbingspeculative positions in the foreignexchange market. Re-booking can-celled forward contracts, whateverthe type and tenor of the underlyingexposure, by resident and foreigninstitutional investors has been dis-allowed. Forward contracts bookedto hedge current account transac-tions regardless of the tenor wereallowed to be cancelled andrebooked. Such facility was alsoavailable to hedge capital accounttransactions that were falling duewithin one year. The apex bankthrough the new measures made itclear that forward contracts oncecancelled cannot be rebooked. Thecentral bank also modified the cur-rency risk hedging norms for import-ers and exporters.

Importers were earlier allowed tohedge currency risk on the basis ofa declaration of an exposure basedon past performance up to the aver-age of the previous three financialyears' actual import/export turnoveror the previous year's import/exportturnover, whichever is higher. Also,contracts booked in excess of 75 percent of the eligible limit were to beon a deliverable basis and could notbe cancelled. The apex bank revisedthese norms.

As a result of the revision the ear-lier 75 per cent facility stands re-duced to 25 per cent of the limit ascompounded by above for importerswho avail themselves of the pastperformance facility. If importershave already used up in excess ofthe revised or reduced facility, theywill not be allowed further bookings.the RBI also specified that this fa-cility will be available on fully deliv-erable basis only. The RBI reducedthe net overnight open position limit(NOOPL) of authorised dealersacross the board with an objectiveto prevent speculations in the for-eign exchange market. It assertedthat the intra-day open position/daylight limit of authorised dealersshould not exceed the RBI-approvedNOOPL. The apex bank indicatedthat these arrangements would bereviewed periodically in line with theevolving market conditions.

49% FDI in InsuranceSector rejected

A parliamentary committee on 13December 2011 rejected almost allthe key changes proposed in the In-surance Laws (Amendment) Bill2008, including the key reform toallow 49% foreign direct investmentin the sector. The the InsuranceLaws (Amendment) Bill was intro-duced in the Rajya Sabha in Decem-ber 2008 to bring about improvementand revision of laws pertaining to theinsurance sector in the changed sce-nario of private sector participationand was subsequently referred tothe standing committee. The panelheaded by BJP leader and formerFinance Minister (during NDA re-gime) Yashwant Sinha maintainedthat the move to hike the FDI cap inthe insurance sector might not have

the desired effect and could exposethe economy to global vulnerability.The standing panel pointed out in itsreport that the proposal to increasethe FDI cap to 49 per cent in insur-ance companies appeared to havebeen decided upon without anysound and objective analysis of thestatus of the insurance sector fol-lowing liberalisation.

Increased role of foreign capital itwas feared would lead to the possi-bility of exposing the economy to thevulnerabilities of the global market,flight of capital outside the countryand also endanger the interest of thepolicy holders. The government'smove was also opposed by both in-surance regulator IRDA and GIPSAon grounds that that foreign insur-ers would be at an advantage overtheir domestic counterparts in thematter of regulations.

Health insuranceBusiness

The panel also rejected the proposalto halve the minimum paid up capi-tal required to start exclusive healthinsurance business to Rs 50 crore.The committee mentioned that theamount may be inadequate as aninsurance company needs to be fullyequipped with modern infrastructureand other facilities. Another proposalto empower the insurance compa-nies to appoint agents and do awaywith the system of licensing ofagents by the regulator, IRDA, wasrejected. The panel maintained themeasure is inappropriate andfraught with the danger of leadingto ineffective regulation of the pro-fession, particularly in instances ofunscrupulous act on the part of theagents as also insurance companies.

WWW.UPSCPORTAL.COM 43

UPSCPORTAL Current Affairs : http://upscportal.com/civilservices/current-affairs

Integrated Guidance Programme for IAS (Pre) - 2012http://upscportal.com/civilservices/online-course/integrated-free-guidance-programme

Profitability of Textile Companies toImprove.

CRISIL Research released its reporton profitability of textile companieson 14 December 2011. According tothe research firm, profitability ofcotton yarn and man-made fibre(MMF) players are expected to im-prove over the next few quarters onaccount of decline in input costs andmoderate demand growth. Duringthe first quarter of 2011-12, the tex-tiles companies witnessed severeprofitability pressures which led tosignificant erosion in their marketcapitalisation. cotton yarn and MMFplayers have registered a negativereturn of 48% and 37%, respectivelyin the past one year. CRISIL Researchopined that the current valuation ofplayers discounts the current nega-tive sentiments around the sectorand offers good scope for upside.Also, stocks of ready-made garment(RMG) companies are fairly pricedin spite of being at historical highs,as they offer relatively high andstable returns among the textilecompanies during the present uncer-tain times. The stocks of brandedRMG companies have out-performedthe S&P CNX NIFTY significantly andposted 25% return on a one-yearbasis.

The slow-down in demand in bothdomestic and export markets andthe anticipation of a spurt in globalcotton production resulted in sharpcorrection in cotton and yarn pricesin the first half of 2011-12. This re-sulted in cotton yarn players report-ing losses in the same period as theywere carrying high cost cotton inven-tory from the last season. However,the sharp drop in cotton yarn pricesalso enhanced its price competitive-

ness vis-a-vis polyester (a substitutefor cotton) thereby limiting the flex-ibility of MMF players to pass on thehike in the costs of their inputs,which are derivatives of crude oil.CRISIL Research covered seven tex-tile stocks– Nahar Spinning Mills andMaharaja Shree Umaid Mills in thecotton yarn segment, JBF Industries, Sangam (India) , Alok Industries andShri Lakshmi Cotsyn in the MMF seg-ment, Kewal Kiran in the RMG space.Of these, most companies have avaluation grade of 5/5, indicating thatthese stocks have a strong upside(more than 25%).

CCEA approvedNational Electricity Fund

The Cabinet Committee on EconomicAffairs( CCEA) on 13 December 2011approvednational electricity fund toprovide subsidy of 8466 crore rupeesfor projects of electricity distributionsector for a period of 14 years. Thefund will be operational within aperiod of six months to one year. Thefund is being set up to provide in-terest subsidy on loans to be dis-bursed to the distribution companiesboth in the private and the publicsector. The objective is to improvethe distribution network for areas notcovered by Rajiv Gandhi GraminVidyutikaran Yojna (RGGVY) andRestructured Accelerated PowerDevelopment and ReformsProgramme (R-APDRP) project ar-eas.

Mobile Subscriber Basein India increased to

881.4 million

According to the data released bythe regulator TRAI on 8 December2011, mobile subscriber base in In-

dia increased to 881.4 million byOctober 2011 from 873.61 million inSeptember 2011 registering a growthof 0.89%. The overall teledensity(telephones per 100 people) in In-dia reached 76.03%. Telecom opera-tors added 7.79 million mobile sub-scribers in October 2011, taking thetotal number of telephone users inthe country to 914.59 million. Thenumber of active mobile subscribers,according to the visitor location reg-ister (VLR) data, during the monthof was 626.18 million. VLR numbersprovide details on active customersat any given point of time, exclud-ing switched-off and out-of-the-cov-erage area customers.

Performance by theTelecom Operators

The growth in the wireless categorywas led by Uninor, which added 2.66million new users, taking its sub-scriber base to 32.31 million by theend of October 2011. TataTeleservices on the other hand lost0.93 million and its subscriber basestood at 87.83 million. The country’slargest private operator Bharti Airteladded 0.94 million subscribers, tak-ing its user base to 173.73 million.Vodafone added 0.92 million newcustomers to take its user base to145.91 million. Idea Cellular andAircel added 1.63 million and 0.48million users, respectively, duringthe period. Idea's subscriber basestood at 101.81 million at the endof October 2011, while that of Aircelstood at 60.28 million. RCom (Reli-ance Communications) added 1.03million new subscribers to take itsuser base to 148.11 million, whileSSTL added 0.74 million new usersto take its total userbase to 14.01million.State-run telcos BSNL and

WWW.UPSCPORTAL.COM44

UPSCPORTAL Current Affairs : http://upscportal.com/civilservices/current-affairs

Integrated Guidance Programme for IAS (Pre) - 2012http://upscportal.com/civilservices/online-course/integrated-free-guidance-programme

MTNL added 0.40 million and 31788new users in October. The subscriberbase of BSNL stood at 96.19 millionand 5.61 million, respectively.

Mobile NumberPortability (MNP)

As per TRAI’s report, about 25.38lakh subscribers submitted requestfor MNP in October 2011. Since theintroduction of MNP in November2010, about 231.66 lakh subscribersacross India had submitted re-quests, till October, for changingtheir service providers while retain-ing their mobile numbers. In MNPZone-I (Northern and Western India),maximum number of requests werereceived in Gujarat (22.39 lakh), fol-lowed by Maharashtra (19.10 lakh).In MNP Zone-II (Southern and East-ern India), maximum number of re-quests have been received in AndhraPradesh (19.46 lakh) followed byKarnataka Service area (19.18 lakh).Broadband

Wireline category subscriber basedeclined to 33.19 million in Octoberfrom 33.31 million in September.Broadband subscription reached12.98 million in the reported monthfrom 12.84 million in September2011.

Coal Ministry decided toAuction 54 Blocks on

The coal ministry decided to auction54 blocks on upfront payment basis.The ministry however might not of-fer mines to power companies. Theministry is currently considering aproposal to earmark blocks to statesthat can call competitive bids forpower supply. The coal ministry also

decided that blocks will not be givenfree to government companies.Though competitive bidding routewould not be applicable to centreand state government projects, PSUswill have to pay reserve price forcoal blocks. Preference would begiven to companies setting up enduse projects in the state that hoststhe coal block and agree to matchthe highest bid. The blocks lie in thecoal belts of seven states ofChhattisgarh, Jharkhand,Maharashtra, West Bengal, Orissa,Madhya Pradesh and AndhraPradesh.

Blocks with over 18,000 milliontonnes of reserves is set to go un-der hammer in the first round of com-petitive bidding. The governmenthas not awarded single coal or lig-nite block for captive use to privatecompanies since October 2008. Thecoal ministry decided to initiate auc-tion by putting the list of blocks onits website. Information on esti-mated reserves, exploration statusand environmental clearances wouldalso be provided. The ministry’s ob-jective is to avoid double bidding forpower companies that after January2011 have to participate in tariff-based bidding to bag power supplycontracts from states.

ICICI’s & IDBI’sLaunch of CDS

India’s largest private lender by as-sets, ICICI Bank and IDBI Bank, theseventh largest public sector bankin India together launched India’sfirst credit default swap (CDS) on 7December 2011. CDS was launchedseven days after the product wascleared by the Reserve Bank of In-dia on 30 November 2011. Public

sector undertaking Rural Electrifica-tion Corporation (REC) bought theCDS cover for its Rs 5 crore loan fromICICI Bank. The launch of the CDSwas a landmark transaction for thedomestic corporate debt market andmarked the formal introduction oflocal currency CDS market in India.IDBI Bank became the country’s firstPSU bank to underwrite a CDS trans-action in the domestic market formanaging credit risks associatedwith Indian corporate bonds.

This is the first transaction of itskind entered by any public sectorbank with another bank in India onselling protection in the domesticmarket on corporate bonds. The cen-tral bank, RBI had issued prudentialguidelines on CDS transactions oncorporate bonds on 30 November2011. The guidelines refered to CDStransactions underwritten by Indianoperations of foreign banks, Indianbanks and overseas branches/sub-sidiaries/joint ventures of Indianbanks.

BenefitsThe launch of the CDS market in In-dia will encourage foreign institu-tional investors to invest in domes-tic corporate bonds. The investmentin domestic corporate bonds willprovide much-needed funding forprojects, including infrastructuresector projects. Credit default swapsalso will investors to transfer andmanage credit risk in an effectivemanner through redistribution ofrisk. Such products are expected toincrease investors’ interest in cor-porate bonds and is likely to provebeneficial to the development of thecorporate bond market in India.

WWW.UPSCPORTAL.COM 45

UPSCPORTAL Current Affairs : http://upscportal.com/civilservices/current-affairs

Integrated Guidance Programme for IAS (Pre) - 2012http://upscportal.com/civilservices/online-course/integrated-free-guidance-programme

What is CDS?

A CDS is similar to a traditional in-surance policy where it obligesseller of the CDS to compensate thebuyer in the event of a loan default.The agreement is that in the eventof a default, buyer of the CDS re-ceives the money which is equiva-lent to the face value of the loan andseller of the CDS receives the de-faulted loan and with it the right torecover it at some later time.

ISDC for the Textile andApparel Sector

With the inauguration of the firststate-of-the-art ATDC-SMART (Skillfor Manufacturing Apparels throughResearch and Training) in Egmore,Chennai, the Integrated Skill Devel-opment Scheme (ISDC) for the tex-tile and apparel sector was launchedby the Ministry of Textiles. The Min-istry launched the project in asso-ciation with the Apparel Training &Design Centre (ATDC), which wasselected as a nodal agency for theproject. The project aims to imparttraining to a workforce of about256000 in the next two years. TheATDC-SMART project worth Rs 23billion focuses on the coreworkforce requirements of the gar-ment industry located across India.The importance of the Scheme canbe judged from the fact that the ap-parel industry is the second largestemployment provider in the countryafter agriculture.

Rural youth and women stand tobenefit the most from the ISDC. The rural youth would be able to findgainful employment in areas neartheir domicile with the help of theimparted training. India’s domestic

textile market is expected to growto US$ 60 billion and exports to US$50 billion by 2015. By 2015 the sec-tor is expected to create an addi-tional 12 million jobs, nearly 40 percent of which will be in the core pro-duction activities. The handicraftsindustry employs 12 million peoplein India currently.

GPON TechnologytransFerred to Telecom

Equipment Makers

The Centre for Development ofTelematics (C-DoT) on 5 December2011 transferred indigenously-de-veloped Gigabit Passive Optical Net-work (GPON) technology to seventelecom equipment manufacturers,including private players. The GPONtechnology was transferred to thetelecom equipment manufacturersto give the much-needed push tobroadband penetration in India. Thegovernment transferred this tech-nology to seven manufacturers inpublic and private sectors — ITI,Bharat Electronics, VMC Systems,United Telecoms, Sai InfoSystem (In-dia), SM Creative Electronics. Trans-fer of technology was also signedwith Tejas Networks for customiseddevelopment.

What isGPON Technology ?

The GPON technology is a pivotalcomponent required for broadbandconnectivity over optical fibre.

C-DOT indigenously designed anddeveloped GPON technology, whichcan be used to provide triple play(voice, video and data) through fi-bre-based networks. The presentGPON standards specify 2.5 Gbps

(Gigabit per second) downstreamand 1.25 Gbps upstream data capa-bility to customer premise. Apartfrom urban areas, the large datacarrying capability is important forIndian villages too where prevailinglow literacy levels will require bet-ter dissemination of informationwith greater graphic and audio con-tent. Besides, voice telephony, highspeed Internet access and IPTV, theC-DOT GPON has provision to carrycable TV signal too.The GPON tech-nology was tested, validated, field-evaluated and made operational inBSNL's network in Ajmer(Rajasthan). The technology willhelp fulfil requirements of majornational programmes like the Na-tional Optical Fibre Network and theState Wide Area Network.

WWW.UPSCPORTAL.COM46

UPSCPORTAL Current Affairs : http://upscportal.com/civilservices/current-affairs

Integrated Guidance Programme for IAS (Pre) - 2012http://upscportal.com/civilservices/online-course/integrated-free-guidance-programme

Buy Online at: www.upscportal.com/order-books

Buy Online at: www.upscportal.com/order-books

WWW.UPSCPORTAL.COM 47

UPSCPORTAL Current Affairs : http://upscportal.com/civilservices/current-affairs

Integrated Guidance Programme for IAS (Pre) - 2012http://upscportal.com/civilservices/online-course/integrated-free-guidance-programme

Indian – China

Indian diamond traders freedTwelve of the 22 Indian diamondtraders, detained in China for nearly2 years on charges of smuggling,reached India on 5 January 2012 af-ter they were let off following de-portation orders by a Chinese court.Twenty-two traders, hailing mostlyfrom Gujarat and Mumbai, were ar-rested in southern China's Shenzhencity in 2010 and were charged withsmuggling diamonds worth 50 mil-lion yuan or 7.3 million US Dollars.They were accused of sourcing dia-monds illegally from Hong Kong forsale in the Chinese market. TheShenzen Intermediate People'sCourt which heard the traders' caseon 7 December 2011, found 12 ofthem innocent and ordered theirdeportation. The rest of the tradershave been awarded varied prisonterms ranging from six years to threeand half years.

India to Develop Facilities NearChina Border

Considering the country's strategicand security requirements acrossthe China-India border, the Centrehas embarked on works to developinfrastructure, the government saidhere. Conscious of China develop-ing infrastructure in the border re-gions opposite India in Tibet andXinjiang Autonomous Regions there,the government was “giving carefuland special attention'' to the devel-

India & The Worldopment of infrastructure in the bor-der areas opposite China, “to meetour strategic and security require-ments and also to facilitate the eco-nomic development of these areas,”Defence Minister A.K. Antony toldthe Rajya Sabha. Infrastructure de-velopment included the Qinghai-Ti-bet railway line, with proposed ex-tension up to Xigze and Nyingchibesides roads and airport facilities.“The government keeps a constantwatch on all developments having abearing on India's security and takesall necessary measures to safeguardit,” he said. The States, where thegovernment is focussing on includeJammu and Kashmir, HimachalPradesh, Uttarakhand, Sikkim andArunachal Pradesh. Among the mea-sures was the development of thetwo airbases for the frontline airsuperiority fighters Su-30-MKI in theNortheast, one at Chabua and an-other at Tezpur.

Chinese-owned Ships EvadeInspection

Two merchant vessels of Chineseownership left anchorage midwaythrough an inspection by the Inter-national Transport Workers' Federa-tion (ITF), fearing action for not hav-ing given minimum wages to theircrew. The incident took place dur-ing the first day of the Flag of Con-venience (FOC) Week of Action be-ing observed by the ITF and its af-filiated unions. M.V. Joyful Spirit,owned by the Golden Ocean Group

Ltd., and m.v. J. Shine, owned by theJiangsu Ocean Shipping Company,were FOC ships that flew the flag ofHong Kong. They departed immedi-ately after unloading the cargo. AnFOC ship is one that flies the flag ofa nation other than the country ofownership to avail cheaper labour,evade taxes and pay a low registra-tion fee. The ITF, set up in 1896, hasmore than six million workers be-longing to 600 transport workers'unions in over 137 countries.

Since the launch of the FOC Weekof Action 10 years ago, 780 vesselshave been inspected and 35 agree-ments signed with ITF standards.Back wages to the tune of $1.02million have been realised. ACroatian ship, which flew a Panamaflag, was “arrested” by the ITF a yearago and was auctioned for Rs. 13crore through a court order .

India - Pakistan

India Rejects Pakistan Proposalto Shift Heavy Guns

India has rejected Pakistan's pro-posal to move heavy artillery andmortars away from the Line of Con-trol citing frequent ceasefire viola-tions. It asked Islamabad to comeclear on its nuclear policy, includingcommand and control over nuclearassets. This was conveyed to thePakistani officials by India during thetwo-day talks on nuclear and con-ventional confidence-building mea-

WWW.UPSCPORTAL.COM48

UPSCPORTAL Current Affairs : http://upscportal.com/civilservices/current-affairs

Integrated Guidance Programme for IAS (Pre) - 2012http://upscportal.com/civilservices/online-course/integrated-free-guidance-programme

sures (CBMs) held in Islamabad af-ter a gap of four years, sources saidhere on Friday. India conveyed toPakistan the need to demonstraterestraint and responsibility in thenuclear field and urged it to facili-tate talks on the Fissile Material Cut-off Treaty (FMCT). External AffairsMinister S.M. Krishna is slated tovisit Pakistan to review the progressin talks and both sides are keen tohave the meetings between HomeSecretaries, Water Resources Secre-taries, Defence Secretaries and For-eign Secretaries before the visit.

On the nuclear CBMs, India made itclear to Pakistan that views onnuclear doctrines could be ex-changed only when official docu-ments enunciating the policies wereavailable in the public domain. Offi-cials pointed out that India had an-nounced its nuclear doctrine of cred-ible minimum deterrence in 2003,which placed the command and con-trol of nuclear devices with the ci-vilian leadership. India stressed theneed to have transparency and pre-dictability in nuclear policies and theimportance of demonstrating, inpractical measure, restraint and re-sponsibility in the nuclear field. In-dian officials also asked their Paki-stani counterparts to allow discus-sion on the Fissile Material Cut-offTreaty at the Conference on Disar-mament. “This would be a CBM,”sources said.

6th Round of Talks BetweenIndia & Pakistan

India and Pakistan concluded theirSixth Round of Expert Level Talks onNuclear Confidence Building Mea-sures in Islamabad on 27 December2011. In this meeting, both the na-tions agreed to recommend to their

foreign secretaries to extend thevalidity of the agreement on reduc-ing the risk from accidents relatingto nuclear weapons. Pakistan sug-gested to India that heavy artilleryshould be removed from the Line ofControl in Jammu and Kashmir.Indiaand Pakistan also reviewed theimplementation and strengtheningof CBMs in the framework of LahoreMoU. The nations also agreed toexplore possibilities for mutuallyacceptable additional CBMs. Indiaand Pakistan discussed a proposalseeking a cooperative mechanism todeal with a crisis like Fukushima.

India-Russia

India-Russia Ties, An Anchor ofPeace & Stability

Global politics will be high on theagenda of Prime MinisterManmohan Singh's visit to Russia inthe context of shifting power equa-tions in the world, said India's en-voy to Russia. “The world is witness-ing a profound transformation anda shift in the global balance ofpower,” said Ambassador AjaiMalhotra. “Our relationship servesas an anchor of peace and stabilityduring this transitions phase.” Theinternational agenda of talks ac-quires special relevance in the lightof India taking the Chair of BRICS in2012 and hosting the group's nextsummit in late March, Mr. Malhotratold the media ahead of the summithere on December 15-17. It will bethe 12th bilateral summit since 2000when the two countries agreed tohold top-level meetings annually.Noting that India and Russia have“identical, close or common” per-spectives on a wide range of inter-national issues, the Indian envoyurged the two countries to step up

cooperation on the global stage.“Global challenges make it impera-tive that we further intensify our dia-logue on issues such as the need torevive the world economy, the situ-ation in West Asia and North Africa,challenges in neighbouring Afghani-stan, cooperation within BRICS, aswell as new threats posed by climatechange and piracy.”

Indo-Russian Cooperation CanPush Global Changes in The Right

Direction“Together India and Russia can helpshape global responses,” he said.“The changes under way offer anopportunity to bring about an inter-national order that is democratic,inclusive, based on the supremacyof international law and consistentwith the objectives and principlescontained in the U.N. Charter.” Dur-ing his talks with President DmitryMedvedev and Prime MinisterVladimir Putin, the Indian Premier isexpected to discuss bilateral coop-eration in defence, space, atomicenergy, hydrocarbons, and scienceand technology, he said. He pointedout that the ongoing protests thathave held up the commissioning ofthe first of two Russian-built nuclearreactors at Kudankulam will not im-pact on collaboration with Russia innuclear energy. “We will fulfill ourcommitments as regards Indo-Rus-sian cooperation in atomic powerenergy.” It is understood that a con-tract for the construction of two morereactors of the Russian design atKudankulam has been finalised butmay not be signed at the Moscowsummit. “Both sides understand thatwe need to proceed with due careand caution” in regard of theKudankulam protests, he said. Theoutgoing year has seen an encour-

WWW.UPSCPORTAL.COM 49

UPSCPORTAL Current Affairs : http://upscportal.com/civilservices/current-affairs

Integrated Guidance Programme for IAS (Pre) - 2012http://upscportal.com/civilservices/online-course/integrated-free-guidance-programme

aging growth in people-to-peoplecontact, with the Indian embassy inRussia issuing 25 per cent moretourist, business and conference-linked visas compared with last year,said the Indian envoy. India hasliberalised visa rules for Russianvisitors since June, and Russia re-ciprocated the move on December1.

5 Agreements Signed BetweenIndia & Russia

The Prime Minister of India visitedRussia on 15-17 December 2011 toparticipate in the 12th Indo-Russianannual summit. India and Russiasigned five major agreements dur-ing this summit, which includes li-censed production of an additional42 Sukhoi combat planes and tomanage fly ash residue at thermalplants in Siberia. A MoU was signedon increasing cooperation and ex-changes between anti-monopoliesagencies and another MoU wassigned to oversee the safety andquality issues by the drug standardsand safety organisations to increasetrade in the pharma sector. India andRussia also signed an educationalexchange programme to share ex-periences in the field of pedagogyand to form a joint working group tomonitor the agreement's implemen-tation. During this visit, India andRussia also issued a joint declara-tion.The main features of this joint dec-laration are as following: » India and Russia decided to com-memorate the 65th anniversary ofestablishment of diplomatic rela-tions between India and Russia in2012 in both the countries in a fit-ting manner. » The sides commended the suc-cessful launch, by India, of the jointly

built YouthSat for space sciencestudies; and expressed satisfactionon the progress in implementationof the joint lunar exploration project.» The nations welcomed the out-come of the eleventh meeting of theIndia-Russia Inter GovernmentalCommission on Military-TechnicalCooperation in addressing issues ofmutual concern. » As an important element of theirStrategic Partnership, both sides re-affirmed their commitment to worktogether to strengthen the centralcoordinating role of the United Na-tions in international affairs as wellas the crucial role of its SecurityCouncil in the maintenance of inter-national peace and security. » The sides expressed concern overnegative trends in the globaleconomy.» The nations identified successfulcompletion of the IMF reform underthe terms and conditions agreed inthe framework of the G-20 as a keyelement of the general reform of theinternational financial architecture.» In the context of the situation inthe Middle East and North Africa, thesides reaffirmed their commitmentto the principles of: searching theway to overcome crises in the regionin compliance with law, exclusivelythrough peaceful means, avoidingviolence and outside intervention,through broad, inclusive nationaldialogue on democratic reforms, tak-ing into account the legitimate rightsand aspirations of the peoples of theregion. » The sides agreed that the Shang-hai Cooperation Organization (SCO)had become an important factor forregional security and cooperation.» The sides attached importance tostrengthening cooperation among

BRICS countries.» India and the Russian Federation,as responsible states, possessingadvanced nuclear technologies,share the objective of preventiveproliferation of weapons of massdestruction and their means of de-livery, including preventing their pos-sible acquisition by terrorist groups. » The sides recalled the bilateralMoscow Declaration between Indiaand the Russian Federation on Inter-national Terrorism signed on Novem-ber 6, 2001 in Moscow.» Both sides reaffirmed the need forall states to combine efforts to van-quish terrorism.» The sides emphasized the impor-tance of joint bilateral and multilat-eral efforts in the fight against illicitdrug trafficking which poses a grow-ing threat to international peace andstability.» India and the Russian Federationexpressed concern over the situa-tion emerging around Iran’s nuclearprogramme. Both sides support acomprehensive and long-termsettlement of the situation throughexclusively political and diplomaticmeans by promoting dialogue.

India, Russia to Focus on Strate-gic Communications

India and Russia will focus ondeliverables in defence and strate-gic communications besides takingforward talks on partnerships innuclear energy and hydrocarbonsspheres during Prime MinisterManmohan Singh's visit to Moscow.As is the tradition during summitsin Moscow, Dr. Singh will also meetPrime Minister Vladimir Putin, archi-tect of restoring India-Russia tiesthat were in disarray during theYeltsin era. Briefing newspersons,

WWW.UPSCPORTAL.COM50

UPSCPORTAL Current Affairs : http://upscportal.com/civilservices/current-affairs

Integrated Guidance Programme for IAS (Pre) - 2012http://upscportal.com/civilservices/online-course/integrated-free-guidance-programme

Foreign Secretary Ranjan Mathaipainted a wide canvas of issues thatwould be taken up during the PrimeMinister's two substantial interac-tions with Mr. Medvedev and Mr.Putin scheduled on Friday. This is thetwelfth time the two countries willmaintain the ritual of holdingstandalone annual summits everyyear besides meeting regularly onthe sidelines of international meets.Having expressed disinterest in gasdeposits in Yamal Peninsula on ac-count of extreme weather condi-tions, given its energy woes, Indiais now keen on discussing modali-ties for teaming up with Novatechfor exploration. Talks will also beheld on other opportunities as Rus-sia opens up its strategic depositsthough there are stiff terms. The twosides will try sorting out the taxa-tion problems facing Imperial En-ergy, bought in haste by ONGC andan important vehicle for taking for-ward India's quest for fossil fuels inRussia.

An important event taking place ona day Russia joins the World TradeOrganisation, will be an importantevent taking place on that day willbe an India-Russia CEOs meet withTata Motors MD Ravi Kant andSistema chief V.P. Evtushenkov in thechair. Dr. Singh and Mr. Medvedevwill have a brief interaction with 20CEOs and ask them to prepare theground for a Comprehensive Eco-nomic Partnership Agreement. “En-ergetic efforts have been under wayto not only maintain the momentumbut also infuse greater content in ourbilateral ties. In the last few months,bilateral visits and talks have takenplace at the levels of Home, Exter-nal Affairs, Defence, Commerce,Health, Tourism, Steel and Power

Ministers,'' said Mr. Mathai. On thenext two units of Kudankulam, Mr.Mathai said discussions “are in ad-vanced stages and there were pros-pects for taking them forward verysoon”.

India and Nepal

India and Nepal on 27 November2011 signed a revised Double Taxa-tion Avoidance Agreement (DTAA). This will help prevent tax evasionand facilitate exchange of informa-tion on banking between the twocountries. DTAA, will allow Indiantraders and investors to enjoy taxrelaxation in India once they paytaxes in Nepal. The agreement isalso likely to increase confidence ofinvestors and help Nepal attractmore investment from India. The re-vised DTAA between India and Nepalwill replace an earlier agreementsigned between India and Nepal in1987.Important Information:» Indian firms are the biggest inves-tors in Nepal. They account forabout 47.5 per cent of total approvedFDIs (Foreign Direct Investments). » India is the biggest source of for-eign investments in Nepal and itslargest trading partner. However,Nepal accounts for only 0.44 per centof India's total trade.» The bilateral trade between Indiaand Nepal increased from USD 1.98billion US dollars in 2009-10 toaround 2.70 billion US dollars in2010-11, which is an increase of 37per cent.

India - Japan

Allaying apprehensions regardingnuclear cooperation in the wake ofthe Fukushima accident, Japan on

Wednesday expressed keenness toexpand ties in the sector after tak-ing into consideration the safetyaspects. Seeking to expand coopera-tion in the 60th year of establishingdiplomatic ties, India and Japan de-cided to expedite work on infrastruc-ture projects and agreed to enhancethe currency swap arrangementfrom $3 billion to $ 15 billion. Thiswas finalised during the talks be-tween Prime Minister ManmohanSingh and his visiting Japanesecounterpart Yoshihiko Noda. “Weshall proceed with the negotiationson the [civil nuclear] agreement af-ter giving due consideration tonuclear safety, nuclear disarmamentand non-proliferation,” Mr. Nodasaid at a joint press conference withDr. Singh after the talks.

Combating TerrorismThe two leaders resolved to developgreater cooperation in combatingterrorism through sharing informa-tion and utilising the India-JapanJoint Working Group on Counter-Ter-rorism, as well as multilateral forasuch as the Financial Action TaskForce and the Global Counter-Terror-ism Forum. Reaffirming the impor-tance of counter-terrorism coopera-tion at the UN, they recognised theurgent need to finalise and adopt theComprehensive Convention on Inter-national Terrorism and called uponall states to cooperate in resolvingthe outstanding issues expedi-tiously.

Nuclear TestsHe said Japan attached importanceto the voluntary commitments andactions India had expressed withregard to testing of nuclear devices.At the same time, Mr. Noda said itwas Japan's duty to share the expe-

WWW.UPSCPORTAL.COM 51

UPSCPORTAL Current Affairs : http://upscportal.com/civilservices/current-affairs

Integrated Guidance Programme for IAS (Pre) - 2012http://upscportal.com/civilservices/online-course/integrated-free-guidance-programme

riences and the lessons learnt fromthe nuclear power accident atFukushima and continue to contrib-ute to nuclear safety around theworld. Mr. Noda's comments as-sume significance as there havebeen questions on the fate of theproposed deal, which is under ne-gotiation, after the radiation scarein Fukushima when tsunami hit anuclear plant there on March 11.

Maritime SecurityDr. Singh and Mr. Noda agreed toexpand cooperation in maritime se-curity including safety and freedomof navigation and anti-piracy activi-ties, by promoting bilateral and mul-tilateral exercises, and through in-formation sharing, as well as dia-logues. They also welcomed thejoint exercises between the CoastGuards of the two countries pro-posed to be held in January 2012, ajoint statement issued after the talkssaid.

More Investment SoughtNoting that FDI from Japan ac-counted for just over one per centof its total outward FDI flows, Dr.Singh said “we would like to seemuch more Japanese investment andtechnology transfer in India, whichwe will facilitate.” Mr. Noda saidcooperation regarding peaceful usesof nuclear energy was conducive toJapan's efforts to address climatechange and to strengthen the glo-bal partnership with India.

Rail TechnologyDr. Singh welcomed the interestshown by Japan in sharing highspeed rail technology for passengernetworks in India.

India and Australia

India and Australia on 16 December2011 signed an agreement to amendDTAA (Double Taxation AvoidanceAgreement) with the objective ofensuring more effective exchange offinancial information between thetwo countries. DTAA was signed be-tween India and Australia in 1991.The revised tax agreement will fa-cilitate the exchange of informationon banking related information andtaxation policies. The revised agree-ment also has a provision that Indiaand Australia can assist each otherin the collection of revenue claims.

The agreement states that nation-als of one country will not be dis-criminated against the nationals ofthe other country, a feature absentin the earlier agreement. Moreover,there was no threshold limit for de-termining permanent establishment.In the revised agreement, thethreshold limit has been rationlisedto avail the exemption for serviceand exploration to boost cross bor-der movement of capital and ser-vices.

Uranium Exports to IndiaApproved

Australia’s ruling Labour Party on 4December 2011 in its annual policyconference approved plans to openup uranium sales to India. Thiscleared the way for talks on a bilat-eral nuclear agreement between In-dia and Australia. The conferenceheld in Sydney overturned its ownban, which prohibits Australia to selluranium to nations such as India thathave not signed Nuclear Non-Prolif-eration treaty. The policy changedoes not need to be approved byParliament. Australia possesses ap-

proximately 40 percent of theworld’s known uranium reserves.

WWW.UPSCPORTAL.COM52

UPSCPORTAL Current Affairs : http://upscportal.com/civilservices/current-affairs

Integrated Guidance Programme for IAS (Pre) - 2012http://upscportal.com/civilservices/online-course/integrated-free-guidance-programme

WWW.UPSCPORTAL.COM 53

UPSCPORTAL Current Affairs : http://upscportal.com/civilservices/current-affairs

Integrated Guidance Programme for IAS (Pre) - 2012http://upscportal.com/civilservices/online-course/integrated-free-guidance-programme

Science & TechnologyTop 10 Breakthroughs of

The year 2011

HPTN 052: The journal Science haslauded an eye-opening HIV study,known as HPTN 052, as the mostimportant scientific breakthrough of2011. This clinical trial demon-strated that people infected with HIVare 96 per cent less likely to trans-mit the virus to their partners if theytake antiretroviral drugs (ARVs).Some researchers consider HPTN052 a "game-changer" because of itsnear-100 percent efficacy in reduc-ing HIV transmission rates. And, in-deed, it has already sprung manyclinicians and policy-makers intoaction. For all thesereasons, Science spotlights theHPTN 052 study as the 2011 Break-through of the Year. Science's and itspublisher, AAAS, the non-profit sci-ence society, have identified nineother groundbreaking scientific ac-complishments during 2011.

The Hayabusa Mission: After somenear-disastrous technical difficultiesand a stunningly successful recov-ery, Japan's Hayabusa spacecraftreturned to Earth with dust from thesurface of a large, S-type asteroid.This asteroid dust represented thefirst direct sampling of a planetarybody in 35 years, and analysis of thegrains confirmed that the most com-mon meteorites found on Earth,known as ordinary chondrules, areborn from these much larger, S-type

asteroids.Unraveling Human Origins: Study-ing the genetic code of both ancientand modern human beings, research-ers discovered that many humansstill carry DNA variants inheritedfrom archaic humans, such as themysterious Denisovans in Asia andstill-unidentified ancestors in Africa.One study this year revealed how ar-chaic humans likely shaped our mod-ern immune systems, and an analy-sis of Australopithecus sediba fossilsin South Africa showed that the an-cient hominin possessed both primi-tive and Homo -like traits.Capturing a Photosynthetic rotein:In vivid detail, researchers in Japanhave mapped the structure of thePhotosystem II, or PSII, protein thatplants use to split water into hydro-gen and oxygen atoms. The crystal-clear image shows off the protein'scatalytic core and reveals the spe-cific orientation of atoms within.Now, scientists have access to thiscatalytic structure that is essentialfor life on Earth — one that may alsohold the key to a powerful source ofclean energy.Pristine Gas in Space: Astronomersusing the Keck telescope in Hawaiito probe the faraway universewound up discovering two clouds ofhydrogen gas that seem to havemaintained their original chemistryfor two billion years after the bigbang. Other researchers identifieda star that is almost completely de-void of metals, just as the universe'searliest stars must have been, but

that formed much later. The discov-eries show that pockets of matterpersisted unscathed amid eons ofcosmic violence.Getting to Know the Microbiome:Research into the countless mi-crobes that dwell in the human gutdemonstrated that everyone has adominant bacterium leading thegang in their digestivetract: Bacteroides , Prevotella orRuminococcus . Follow-up studiesrevealed that one of these bacteriathrives on a high-protein diet whileanother prefers vegetarian fare.These findings and more helped toclarify the interplay between dietand microbes in nutrition and dis-ease.A Promising Malaria Vaccine: Earlyresults of the clinical trial of a ma-laria vaccine, known as RTS,S, pro-vided a shot in the arm to malariavaccine research. The ongoing trial,which has enrolled more than 15,000children from seven African coun-tries, reassured malaria researchers,who are used to bitter disappoint-ment, that discovering a malariavaccine remains possible.Strange Solar Systems: This year,astronomers got their first goodviews of several distant planetarysystems and discovered that thingsare pretty weird out there. First,NASA's Kepler observatory helpedidentify a star system with planetsorbiting in ways that today's mod-els cannot explain. Then, research-ers discovered a gas giant caught ina rare "retrograde" orbit, a planet

WWW.UPSCPORTAL.COM54

UPSCPORTAL Current Affairs : http://upscportal.com/civilservices/current-affairs

Integrated Guidance Programme for IAS (Pre) - 2012http://upscportal.com/civilservices/online-course/integrated-free-guidance-programme

circling a binary star system and 10planets that seem to be freely float-ing in space — all unlike anythingfound in our own solar system.Designer Zeolites: Zeolites are po-rous minerals that are used as cata-lysts and molecular sieves to con-vert oil into gasoline, purify water,filter air and produce laundry deter-gents (to name a few uses). Thisyear, chemists really showed offtheir creativity by designing a rangeof new zeolites that are cheaper,thinner and better equipped to pro-cess larger organic molecules.Clearing Senescent Cells: Experi-ments have revealed that clearingsenescent cells (those that havestopped dividing) from the bodies ofmice can delay the onset of age-re-lated symptoms. Mice whose bod-ies were cleared of these loiteringcells didn't live longer than theiruntreated cage-mates — but theydid seem to live better, which pro-vided researchers with some hopethat banishing senescent cells mightalso prolong our golden years.

A new CandidateMalaria Vaccine

A new candidate malaria vaccinewith the potential to neutralise allstrains of the most deadly speciesof malaria parasite has been devel-oped by a team led by scientists atthe University of Oxford. The resultsof this new vaccine independentlyconfirm the utility of a key discoveryreported last month from scientistsat the Wellcome Trust Sanger Insti-tute who had identified this targetwithin the parasite as a potential‘Achilles' heel' that could hold sig-nificant promise for vaccine devel-opment. The most deadly form ofmosquito, Plasmodium falciparum ,

is responsible for nine out of tendeaths from malaria. Vaccinatingagainst malaria is likely to be themost cost-effective way of protect-ing populations against disease;however, no licensed vaccine is cur-rently available.Dr Sandy Douglas, a Wellcome TrustClinical Research Training Fellowfrom the University of Oxford andfirst author on the new study, adds:“We have created a vaccine thatconfirms the recent discovery relat-ing to the biology of RH5 antigen,given it can generate an immuneresponse in animal models capableof neutralising many — and poten-tially all — strains of the P.falciparum parasite, the deadliestspecies of malaria parasite.” Ournext step will be to begin safetytests of this vaccine. If these provesuccessful, clinical trials in patientsmay begin within the next two tothree years.

Fighting Iron Deficiency

Someone with an iron deficiencyshould substantially reduce con-sumption of coffee and black tea.This is according to Petra Renner-Weber, a member of Germany'sHome Economics and Nutrition Sci-ence Association. She says that tan-nin in the drinks inhibits the absorp-tion of iron. “About two hours shouldpass after a meal before drinkingcoffee or tea,” she added. Renner-Weber said the body absorbed ironfrom animal products such as meatand sausage best. When its ironstores are depleted, the body ab-sorbs a lot of iron from food. Whenstores are well supplied, it takes inlittle of the mineral.She advises vegetarians to eat iron-rich vegetables such as beets as

well as legumes and whole grains.She said that vitamin C and organicacids enhanced the absorption ofiron from plant foods and so she rec-ommends drinking a glass of orangejuice at meals or having fruit saladfor dessert. Iron deficiency, which adoctor can detect with a blood test,is caused by an unbalanced diet or,in women, heavy menstrual periods,Renner-Weber said. Symptoms in-cluded tiredness, weakness, in-creased susceptibility to infections,problems with hair and nail growth,and dry, chapped skin. Men requireabout 10 mg of iron daily, andwomen 15.

Supernova of aGeneration

It was the brightest and closest stel-lar explosion seen from Earth in 25years, dazzling professional andbackyard astronomers alike. Now,thanks to this rare discovery—whichsome have called the “supernova ofa generation” — astronomers havethe most detailed picture yet of howthis kind of explosion happens.Known as a Type I a supernova, thistype of blast is an essential tool thatallows scientists to measure theexpansion of the universe and un-derstand the very nature of the cos-mos. On August 24, the PalomarTransient Factory (PTF) team discov-ered the supernova in one of thearms of the Pinwheel Galaxy (alsocalled M101), 21 million light yearsaway. They caught the supernovajust 11 hours after it exploded.

27 Club' Hypothesis isIncorrect

The list of well known musicianswho have died at age 27 may look

WWW.UPSCPORTAL.COM 55

UPSCPORTAL Current Affairs : http://upscportal.com/civilservices/current-affairs

Integrated Guidance Programme for IAS (Pre) - 2012http://upscportal.com/civilservices/online-course/integrated-free-guidance-programme

like more than a coincidence — AmyWinehouse, Jim Morrison, JimiHendrix, Janis Joplin, Kurt Cobain,and Brian Jones to name a few —but their age is unlikely to have beenthe cause of their demise, accord-ing to research published inthe British Medical Journal. Whilefame may increase the risk of deathfor musicians, probably due to theirrock and roll lifestyle, this risk is notlimited to age 27, say the authorsled by Adrian Barnett fromQueensland University of Technol-ogy in Australia. To test the “27 club”hypothesis, the authors comparedthe mortality of famous musicianswith that of the UK population. Theyincluded 1,046 musicians (solo art-ists and band members) who had anumber one album in the UK chartsbetween 1956 and 2007.

During this period (1956-2007) it wasfound that 71 (7 per cent) of themusicians died. The authors usedmathematical analysis to determinethe significance of age 27. Theyfound no peak in the risk of death atthis age, however musicians in their20s and 30s were two to three timesmore likely to die prematurely thanthe general UK population. The re-search team found some evidenceof a cluster of deaths in those aged20 to 40 in the 1970s and early1980s. But there were no deaths inthis age group in the late 80s. Theauthors conclude that the “27 club”is based on myth, but warn thatmusicians have a generally in-creased risk of dying throughouttheir 20s and 30s.Protecting an anti-malarial drugfrom developing resistance: It is adrug that has its roots in ancientChinese medicine. In the fightagainst malaria, a disease that over

200 million people are estimated tohave caught in 2010, some 655,000of whom died of it, protecting theeffectiveness of artemisinin-baseddrugs has become vitally important.A number of Indian pharmaceuticalcompanies have been among thosemanufacturing and marketing drugsthat are likely to foster resistance toartemisinin in the malaria parasite,according to the latest World Ma-laria Report that was recently re-leased. Artemisin and its derivativeshave saved countless lives after thesingle-celled parasite, Plasmodiumfalciparum , that causes the mostdangerous forms of the disease be-came resistant to the drug chloro-quine. However, strains that are re-sistant to even artemisinin haveemerged in parts of South-East Asiaand could potentially spread, as hashappened with earlier antimalarialdrugs.

To prevent that from happening, theglobal health agency recommendsthat artemisinin be given in combi-nation with another drug. Suchartemisinin-based combinationtherapy (ACT) should, it says, befirst-line treatment for uncompli-cated malaria caused by P.falciparum . The two-drug combina-tion reduced the chances of theparasite developing resistance.Moreover, a three-day course of arecommended ACT generally clearedthe parasites from the body. The useof ACT has grown rapidly. Globally,the number of ACT courses procuredby the public sector jumped nearlyseven-fold between 2005 and 2006,and then more than doubled, reach-ing 181 million, in 2010, accordingto the World Malaria Report. Thedemand for these drugs was ex-pected to reach 287 million treat-

ments this year and touch 295 mil-lion courses in 2012. Ten of the 28manufacturers of monotherapieswere in India, according to a spokes-person for the WHO Global MalariaProgramme. “Oral artemisininmonotherapy is banned in India,”according to the ‘Guidelines for Di-agnosis and Treatment of Malaria inIndia' published in 2010 by the Na-tional Institute of Malaria Researchin Delhi and the Union HealthMinistry's National Vector Borne Dis-ease Control Programme.

Nuclear DisasterResponse Failed During

Tsunami

Japan's response to the nuclear cri-sis that followed the March 11 tsu-nami was confused and riddled withproblems, including an erroneousassumption an emergency coolingsystem was working and a delay indisclosing dangerous radiationleaks, an interim report revealed onDecember 26. The disturbing pictureof harried and bumbling workers andgovernment officials scrambling torespond to the problems atFukushima Dai-ichi nuclear powerplant was depicted in the report de-tailing a government investigation.The 507-page interim report, com-piled by interviewing more than 400people, including utility workers andgovernment officials, found authori-ties had grossly underestimated tsu-nami risks, assuming the highestwave would be 6 meters (20 feet).The tsunami hit at more than doublethose levels. The report criticized theuse of the term “soteigai,” meaning“outside our imagination,” which itsaid implied authorities were shirk-ing responsibility for what had hap-pened. It said by labeling the events

WWW.UPSCPORTAL.COM56

UPSCPORTAL Current Affairs : http://upscportal.com/civilservices/current-affairs

Integrated Guidance Programme for IAS (Pre) - 2012http://upscportal.com/civilservices/online-course/integrated-free-guidance-programme

as beyond what could have beenexpected, officials had invited pub-lic distrust.

The report, set to be finished by mid-2012, found workers at Tokyo Elec-tric Power Co., the utility that ranFukushima Dai-ichi, were untrainedto handle emergencies like thepower shutdown that struck whenthe tsunami destroyed backup gen-erators setting off the world's worstnuclear disaster since Chernobyl. Abetter response might have reducedthe core damage, radiation leaks andthe hydrogen explosions that fol-lowed at two reactors and sentplumes of radiation into the air, ac-cording to the report. The govern-ment also delayed disclosure of ra-diation data in the area, unneces-sarily exposing entire towns to ra-diation when they could have evacu-ated, the report found. The govern-ment recommended changes soutilities will respond properly to se-rious accidents. It recommendedseparating the nuclear regulatorsfrom the unit that promotes atomicenergy, echoing frequent criticismsince the disaster. Japan's nuclearregulators were in the same minis-try that promotes the industry, butthey are being moved to the envi-ronment ministry next year to ensuremore independence.

Microalgae

To its votaries, tiny organismsknown as microalgae could hold an-swers to some intractable problems.That includes curbing carbon diox-ide emissions that are contributingto global warming and reducing theburden of industrial effluents. “Wecan change trash into gold” andsafeguard human existence on the

planet, exclaimed Ji-Won Yang ofthe Department of Chemical andBiomolecular Engineering at the Ko-rea Advanced Institute of Scienceand Technology in South Korea.Microalgae, like plants, are capableof photosynthesis, using the energyfrom sunlight to turn carbon dioxideand water into sugar. Prof. Yangspoke of his team's efforts at treat-ing municipal wastewater using thesingle-celled green alga,Chlorellavulgaris. Such wastewater containedplenty of organic compounds andnutrients to sustain the organism.The oils that then accumulated in thecells could be harvested and turnedinto biodiesel. It was found that un-treated wastewater gave the bestalgal growth. Nitrogen-fixing bacte-ria found in the wastewater could beacting as growth enhancers, henoted.

A pilot plant based on the alga hadbeen established at a plant of theChina Steel Corporation. To meetpollution control norms, the effluentwas earlier being neutralised withsodium hydroxide. The large quan-tities of sludge that resulted hadthen to be disposed in a securedlandfill. It had been possible to en-gineer a treatment process to effi-ciently carry out the neutralisationby providing suitable conditions fora blue green alga, Chroococcusturgidus, to grow in the effluent. Asthe alga grew, it reduced the acidityof the effluent. This algal strain hadbeen isolated from the effluent it-self. The remediation plant had beenworking well for over five years, pro-ducing negligible quantities ofsludge and saving money that hadearlier been spent on sodium hy-droxide.

Ordinary TissueTransFormed into Heart

Muscle Cells

Scientists discovered a way to trans-form ordinary tissue into beatingheart muscle cells. It could pave theway for new therapeutic approachesfor making a damaged heart to re-pair itself. Scientists used a zebrafishsystem to develop a small and ro-bust molecule, which can transformstem cells into beating heart musclecells. The scientists in their experi-ment found that cardionogen treat-ment enlarged the zebrafish heart bystimulating production of new car-diac muscle cells from stem cells.The scientists discovered threestructurally related molecules(Cardiongen-1, 2 and3) after screen-ing 4000 compounds. It could pro-mote or inhibit heart formation de-pending on when they were admin-istered during development.

New theoryExplains How Fishes

Grew into Amphibians

A small fish crawling out of a dryingdesert pond underlines a theory thatties up the fishes with the amphib-ians, reveals a study. “Such a pluckyhypothetical ancestor of ours prob-ably could not have survived theoverwhelming odds of perishing ina trek to another shrinking pond,”said Gregory J Retallack, professorof geological sciences at Universityof Oregon, who led the study. Thisscenario comes from the late Devo-nian period, roughly 390 million to360 million years ago, which lateHarvard palaeontologist AlfredRomer propounded, the Journal ofGeology reported. Challenging

WWW.UPSCPORTAL.COM 57

UPSCPORTAL Current Affairs : http://upscportal.com/civilservices/current-affairs

Integrated Guidance Programme for IAS (Pre) - 2012http://upscportal.com/civilservices/online-course/integrated-free-guidance-programme

Romer's theory on fish survival,Retallack said that the transitionalfossils were not associated with dry-ing ponds or deserts, but were foundconsistently with humid woodlandsoils, according to a university state-ment.

Papaya Mealy Bug

In India mealybug infests a wholerange of crops nd numerous weeds.Infested growing points becomestunted and swollen which may varydepending upon the susceptibility ofeach host. Heavy clustering of mea-lybugs can be seen under leaf sur-face giving the appearance of a thickmat with waxy secretion. Severe in-festations resemble patches of cot-ton all over the plant. They excretecopius amount of honey dew thatattracts ants that leads to the de-velopment of black sooty mould,which inhibits the plants' ability tomanufacture food. Both nymphs andadults suck the sap from leavescausing withering and yellowing ofleaves. Fruit may drop prematurelyon crop plants. Heavy infestationcan cause defoliation and evendeath of the plant. When fruits areinfested, they may be entirely cov-ered with the white, waxy coatingof the mealy bug. Infestation canlead to fruit drop, or fruit may remainon the host in dried and shrivelledcondition.

Biological control by release of natu-ral enemies has proved to be verysuccessful. The National bureau ofagriculturally important insects(NBAII) has successfully importedthree parasitoids to suppress themealybugs effectively. The larvaeof Spalgius apius commonly calledas blue butterflies feed on all stage

of the mealybug. As the young lar-vae are similar in appearance tomealybugs, it is difficult to recognizethem amidst the host population.Adult is a small butterfly with upperside of the wings dark brown incolour and ash coloured lower sideswith dark striations. Krishi VigyanKendra at Karur has mass multipliedthe parasitoids successfully in thelaboratory as well as field condi-tions.

Solar-Powered SprayerThat Solves Many

Problems

One need not be a genius to developany commonly used devices. Takethe case of the humble yet impor-tant knapsack manual sprayer (hungon the shoulders). Though manymodels are available on the shelves,priced at different rates, most ofthem last for only a few years.“Regularly filling them with water,pesticides and fuel (in case of fueloperated ones) makes it difficult tolift and hang them on the shoulder.After an hour of spraying, the weightof the machine creates a backacheand shoulder pain for farmers,” saysMr. David Raja Beleau, AssistantDirector of Horticulture Kadayam,Tamil Nadu, who developed a solarpowered battery operated sprayerfor farmers.

High-SpeedBullet Train in China

China on 23 December 2011launched a high-speed bullet trainin Quingdao, Shandong province. Itsspeed can reach up to 500 kilometreper hour. The train was launched byChina’s largest rail vehicle maker,

CSR Corp. Ltd. The six-car train hasa maximum tractive power of 22800kilowatts, compared with 9600 kilo-watts for the CRH380 trains. Thetrain is made from plastic materialsreinforced with carbon fibre. It isdesigned to resemble an ancientChinese sword. CRH 380 trains arecurrently in service on the Beijing-Shanghai High-speed Railway. Itholds the world speed record of 300kilometres per hour.China has thelargest network of bullet-train trackin the world, with 8000 miles oftrack.

Memory Gene Npas4

A team of neuroscientists found agene that turns on when memoriesare stored in the brain. This discov-ery could help trace the exact loca-tions of memories in the brain. Itcould help in creating and alteringmemory. The gene is called Npas4 ,which is very active in the hippoc-ampus. The animal is known to havea brain structure critical in forminglong-term memories. During micestudies Scientists found that by tak-ing out Npas4 from test subjects,neuroscientists were able to preventnew memories from forming.

Tube Light Flicker

Both conventional fluorescent lampand compact fluorescent lamp (CFL)produce white light by the processof fluorescence underlying the con-version of invisible ultra-violet (UV)generated from the electrical dis-charge (passage of electricity) alongthe column of the tube to visible lightoccurring on the white coating madeof powdery luminescent materialcoating inside the walls of the tube.The process of generating white

WWW.UPSCPORTAL.COM58

UPSCPORTAL Current Affairs : http://upscportal.com/civilservices/current-affairs

Integrated Guidance Programme for IAS (Pre) - 2012http://upscportal.com/civilservices/online-course/integrated-free-guidance-programme

light from these lamps is the samealthough chemical composition ofthe luminescent materials (phos-phors) used in these lamps are dif-ferent which eventually determinethe light output produced. There isslight difference in the process ofinitiating the electrical dischargebetween CFL and conventional fluo-rescent lamp.

In a conventional fluorescent lamp,the electrical discharge along thelengthy column of the tube (usually4 feet long) is struck through a highvoltage with the help of a choke(voltage converter) and bi-metallicstrip starter combination. Uponswitching on, the lamp circuit isclosed due to the deflection of bi-metallic strips establishing contact.Also the electrical discharge is ini-tially built along the medium argon/neon-mercury mixture followed byregular discharge through mercuryvapours to yield stabilized genera-tion of UV light. These two pro-cesses take some time to stabilize.On the other hand in the case of CFLhaving built-in integrated electronicballast comprising array of AC-DC-AC converters and oscillators, withthe instant generation of high fre-quency (50 kHz) in the CFL column,the electric discharge is struck in-stantly without any delay and startsfluorescing facilitating a quick startas high frequency helps to strikeelectrical discharge faster.

Higgs Boson

The results announced from CERNon December 13 by the two teams— ATLAS and CMC — do not providedefinite and conclusive evidence ofthe presence of the elusive Higgsboson, but they have collected suf-

ficient data that is consistent withthe possible appearance of the Godparticle. According to theoristssome subatomic particles gain massby interacting with the Higgs boson.The Higgs boson is the only undis-covered part of the Standard Modelof physics, which describes the ba-sic building blocks of matter andtheir interactions.

The combined results presented lastmonth by the two teams provided noroom for the Higgs boson to hide in.The results had narrowed down themass region where the Higgs par-ticle is most likely to be — between114 GeV and 141 GeV (gigaelectoronvolt). The latest results have furthernarrowed the mass range — 115-130GeV range in the case of ATLAS, and117-127 GeV by CMC. “We have notcollected enough evidence for a dis-covery. There is an excess of eventscompatible with the hypothesis thatit could be a Higgs,” Guido Tonelli,spokesman for CMC was quoted assaying in Nature . But more confir-matory data is required before itspresence can be proved. Accordingto the journal, if supported by fur-ther data, the results suggest aHiggs particle with a mass of about125 GeV. This is because the ATLASresults are “consistent with a 125-126 GeV Higgs at a statistical levelof at most 3.6 standard deviation,while the CMC team found 124 GeVsignal of at most 2.6 standarddeviation,” Nature states.

Oyster ‘language'

Scientists in Japan have begunstudying the ‘language' of oysters inan effort to find out what they aresaying about their environment. Re-searchers are monitoring the open-

ing and closing of the molluscs inresponse to changes in seawater,such as reduced oxygen or red tide,a suffocating algal bloom, that canlead to mass die-offs. Using a de-vice they have nicknamed the ‘kai-lingual,' scientists from KagawaUniversity want to see if they candecode oyster movements that mightwarn of possible problems. The kai-lingual uses a series of sensors andmagnets to send information on theopening and closing of shells in re-sponse to environmental changes.The technique has never before beenused on oysters farmed for food, buthas been employed by pearl oysterfarmers.

Kepler-20e andKepler-20f

NASA's Kepler mission on 20 Decem-ber 2011 discovered the first Earth-size planets orbiting a sun-like staroutside our solar system. The plan-ets, calledKepler-20e and Kepler-20f, are too close to their star to bein the so-called habitable zonewhere liquid water could exist on aplanet's surface, but they are thesmallest exoplanets ever confirmedaround a star like our sun. The dis-covery marks the next importantmilestone in the ultimate search forplanets like Earth. The new planetsare thought to be rocky. Kepler-20eis slightly smaller than Venus, mea-suring 0.87 times the radius of Earth.Kepler-20f is a bit larger than Earth,measuring 1.03 times its radius.Both planets reside in a five-planetsystem called Kepler-20, approxi-mately 1000 light-years away in theconstellation Lyra.To validate Kepler-20e and Kepler-20f, astronomers used a computerprogram called Blender, which runs

WWW.UPSCPORTAL.COM 59

UPSCPORTAL Current Affairs : http://upscportal.com/civilservices/current-affairs

Integrated Guidance Programme for IAS (Pre) - 2012http://upscportal.com/civilservices/online-course/integrated-free-guidance-programme

simulations to help rule out otherastrophysical phenomena masquer-ading as a planet. Kepler-20e orbitsits parent star every 6.1 days andKepler-20f every 19.6 days. Theseshort orbital periods mean very hot,inhospitable worlds. Kepler-20f, at800 degrees Fahrenheit, is similar toan average day on the planet Mer-cury. The surface temperature ofKepler-20e, at more than 1400 de-grees Fahrenheit, would melt glass.The Kepler-20 system includes threeother planets that are larger thanEarth but smaller than Neptune.Kepler-20b, the closest planet,Kepler-20c, the third planet, andKepler-20d, the fifth planet, orbittheir star every 3.7, 10.9 and 77.6days. All five planets have orbits ly-ing roughly within Mercury's orbit inour solar system. The host star be-longs to the same G-type class asour sun, although it is slightlysmaller and cooler.

Measuring Radiation onJourney to Mars

The Radiation Assessment Detector,the first instrument on NASA's nextrover mission to Mars to begin sci-ence operations, was powered upand began collecting data Decem-ber 6, almost two weeks ahead ofschedule. RAD is the only instrumentscheduled to collect science data onthe journey to Mars.The instrumentis measuring the energetic particlesinside the spacecraft to character-ize the radiation environment anastronaut would experience on afuture human mission to the RedPlanet. “We are seeing a strong fluxin space, even inside the spacecraft,about four times higher doses of ra-diation than the baseline we mea-sured on the launch pad from the

RTG, or radioisotope thermoelectricgenerator, used to power the rover.It's very exciting to begin the sciencemission.” RAD will measure the rel-evant energetic particle speciesoriginating from galactic cosmicrays, the Sun and other sources. Ofparticular interest are the particlesaccelerated by coronal mass ejec-tions on the surface of the Sun,which spew fast-moving clouds ofradiation across the solar system.RAD was designed to characterizeradiation levels on the surface ofMars, but an important secondaryobjective is measuring the radiationon the almost nine-month journeythrough interplanetary space, to pre-pare for future human exploration.

NASA DevelopedSpace Harpoon

NASA(National Aeronautics andSpace Administration) built a proto-type capable of launching test har-poon tips across a distance of a mile(1.6km). It would be safer to collectcomet material using the space har-poon before landing on the celestialbodies. The samples thus collectedwill help reveal the origins of theplanets and how life was created onEarth. NASA’s Stardust mission hadrecovered particle samples in 2002.

These samples included an aminoacid, glycine, which is used by liv-ing organisms to create proteins. Itendorsed the fact that some of life’singredients had formed in space andwere delivered to Earth by meteor-ite and comet impacts. A comet isan icy small Solar System body that,when close enough to the Sun, dis-plays a visible coma and sometimesa tail.

India Successfully Test-Fired N-capable

Agni-I Missile

India successfully test-fired itsnuclear capable Agni-I strategic bal-listic missile on 1 December 2011from the test range at Wheeler Is-land off Odisha coast, as part of theIndian Army's user trial. It was test-fired from a mobile launcher, fromlaunch pad-4 of the Integrated TestRange (ITR). The Strategic ForceCommand (SFC) of the Army, as partof their training exercise, executedthe trial with logistic support pro-vided by Defence Research Devel-opment Organisation (DRDO) at theITR. The trajectory of the missile,which had an operational strikerange of 700 km, was tracked by so-phisticated radars and electro-optictelemetry stations located along thesea coast and ships positioned nearthe impact point in the downrangearea.

Agni-I missile is equipped with aspecialised navigation system whichensures it reaches the target with ahigh degree of accuracy. Agni-I wasdeveloped by Advanced SystemsLaboratory (ASL), the premier mis-sile development laboratory of theDRDO in collaboration with DefenceResearch Development Laboratory(DRDL) and Research Centre Imarat(RCI) and integrated by Bharat Dy-namics Limited (BDL), Hyderabad. The last trial of the Agni-I missilewas successfully carried out on 25November 2010 from the samebase. Details of Agni-I:» Agni-I strategic ballistic missile isan indigenously developed surface-to-surface missile

WWW.UPSCPORTAL.COM60

UPSCPORTAL Current Affairs : http://upscportal.com/civilservices/current-affairs

Integrated Guidance Programme for IAS (Pre) - 2012http://upscportal.com/civilservices/online-course/integrated-free-guidance-programme

» It is a single-stage missile» It is powered by solid propellants» Agni-I can carry payloads up to1000 kg» It Weighed 12 tonnes» It was 15-metre-long

Why Mobile PhonesSwitched When

Travel in a Flight?

Airplanes, soon after takeoff andmuch before landing, fly at an alti-tude of higher than a kilometre dur-ing their flight. They have to ascendto these high altitudes for the aerialflight to avoid turbulence due toclouds and the clear air turbulence(CAT) present at low and normal al-titudes. At such high altitudes, thepilots have to largely depend on theavionics and computer-assistedcoded signal communications. Thepilots know the flight location anddestination besides other geo-graphical details by guided com-mands from the Global PositioningSystem (GPS) which links the controltower, airplane cockpit and geosta-tionary satellites. Further, they canknow the weather and other meteo-rological conditions and directionsof landing and takeoff at the air-ports, only through wireless commu-nications from the control towers atthe airports. In poor weather, thepilots use instrument landing system(ILS) to find the runway. Most ofthese avionics are remote opera-tions and all the communicationsare in wireless radio and microwavesemionics. All the wireless signaltrafficking is accomplished by modu-lated radio and microwave transmis-sions which type the cell phonesalso owe their functioning to. If thereare any mobile phones on board in

the cabin of the flight in ‘ on' modeduring takeoff, flight and landing, themicrowave transmission betweenthe mobile phone towers and themobile phones might interfere withthe communications the flights arebusily engaged in (between thecockpit and the control towers).

YETI DemystifiesMathematical Modeling

Young Ecologists Talk and Interact(YETI) 2011, at the Indian Instituteof Techonlogy-Guwahati, witnesseda unique activity on the second day.Instead of interaction on ecology orconservation, participants in one ofthe workshops were asked to do anarithmetic exercise using calcula-tors. They were amazed at the re-sults they got as they had no ideathat these could be related to stud-ies in ecological sciences. It wasVishwesha Guttal, assistant profes-sor at the Centre for Ecological Sci-ences (CES), Indian Institute of Sci-ences (IISc), Bangalore, whodemystified mathematical modellingof ecology for the young ecologists.Mr. Guttal helped them understandthat through proper application ofsimple mathematical modelling, in-teresting results of ecological stud-ies could be found.

Innovative TechniqueShowcased at

National Conference

“A single person combines in him-self the work of a weatherman, anengineer, a marketing executive, aveterinarian, and above all a states-man endowed with the responsibil-ity of feeding his people, says Mr G.R. Sakthivel an enterprising and in-

novative farmer fromSathyamangalam, Erode, TamilNadu. A member of the scientificadvisory committee of MYRADA-KVKand Erode district organic farmers'federation, Mr. Sakthivel developeda simple yet effective mechanism tofilter cattle waste and use the samein sugarcane cultivation. The fourcompartment system includes thefiltration technique, ensuring that anenriched solution gets collected atthe end, mixed with water, and sentby drip irrigation system to the field.The first section is meant for collec-tion of cow dung and urine mixing.After thorough mixing, the solutionis sent to the second compartmentfor first filtration. The solid matteris used for biogas production and theupper part of the solution is thenallowed to flow into a third compart-ment where jaggery is added for fer-mentation. The clear enriched fil-trated medium is collected and usedfor irrigation. By adopting this tech-nology the farmer can saveRs.27,000 per acre as it reduceslabour and fertilizer cost, accordingto Mr. Sakthivel. This technologyaids water holding capacity in thesoil and presence of earth worms isconsiderably increased in the fields.

Land MineTechnology to Help

Detect Breast Cancer

Scientists claimed that now a simpletest could diagnose breast cancer inwomen in eight seconds. They claimto have developed it from a technol-ogy that is used to detect landmines. The new screening tool,called MARIA, is safer and morecomfortable than traditional mam-mogram X-rays. It can be used on

WWW.UPSCPORTAL.COM 61

UPSCPORTAL Current Affairs : http://upscportal.com/civilservices/current-affairs

Integrated Guidance Programme for IAS (Pre) - 2012http://upscportal.com/civilservices/online-course/integrated-free-guidance-programme

women of any age, unlike currenttechnology. Scientists already car-ried out three successful trials with300 women at Frenchay andSouthmead hospitals. They alsoclaimed that results from the trialsshowed a diagnostic success rate of80 per cent. MARIA stands forMultistatic Array processing for Ra-dio wave Image Acquisition, is madefrom 60 antennas, which create acomplete scan of the breast in eightseconds.

Genetic Tests toDetect Breast Cancer

Scientists recently developed a ge-netic test to detect breast cancer. Itwill help those women who are inthe early stages of breast cancer. Itwill save them the trouble of under-going chemotherapy. Scientists in-vented the Oncotype DX test, whichcan help doctors determine the like-lihood of breast cancer returning andif further intervention is needed.The test uses a small sample ofbreast tumour tissue and focuses ongroups of genes, which can influencehow a cancer is likely to grow andrespond to treatment. At present,the test is available in the US andAmerican Society for Clinical Oncol-ogy and the National. Comprehen-sive Cancer Network include it intheir treatment guidelines for theearly stage of breast cancer.

Why is it Hardto Pull an Empty Mug

Completely ImmersedUpside Down in Water?

Not only is it hard to pull out anyempty mug completely immersedupside down in water but also it is

harder to push an empty mug upsidedown into water.The latter difficultyis mainly due to the buoyancy, theair sac, captured inside the mug,experiences during such operation.However, when we try to pull out thesame empty mug or the mug alreadydrowned, in an upside manner, weexperience certain difficulty of pull-ing it out. This difficulty stems froma different cause and owes to thesurface tension phenomenon of liq-uids. The atoms or molecules of aliquid in its bulk regions have theirchemical valences fully satisfied inall the six directions (front and rear,left and right and up and down ormathematically, x and x', y and y'and z and z' directions of space)whereas those present on the sur-face of the liquid have only five oftheir directions (front and rear, leftand right and down) chemically sat-isfied while leaving the upward con-nectivity missing (say, z direction).This deficiency has the effect ofkeeping all the atoms or moleculesof the liquids, on their surface, morefirmly coherent on their lateral sidesenabling the surface of such liquidact like an elastic skin. Hence, itwould demand certain amount offorce to tear of the surface of theliquids.

The amount of force required to openapart one unit length of the liquid'ssurface is defined as the surfacetension of the liquid. Its value isunique to the liquid and is highlysensitive to the temperature, pres-sure and purity of the liquid. Forwater, its value is about 72 dynesper cm (or 72 milli newtons permeter). When a mug is pulled outupside down, its brim has to tearopen the surface of water layer by alength (perimeter) equal to the

mug's brim to detach itself from thesurface of the water. Say, the mughas a perimeter (brim) of about 12inches (i.e., about 30 cms) and amass of about half kilogram withnegligible thickness. Then, it is heldon its brim by the water surface bya surface force of about 0.02 new-tons when the mug is about to de-tach from the surface. This means,we have to apply this much addi-tional force (besides the force re-quired against gravity) when themug's brim is pulling out from theliquid's surface besides the usualforce of about 5 newtons against thegravity. The 5 newtons against thegravity would have been the onlyforce required had there been nosurface tension phenomenon.Hence, we feel it hard to pull out anempty mug, upside down, from wa-ter.

Technology to ProducePocket TV

British scientists recently developeda technology, which could be usedto produce pocket TV. Scinetists de-veloped a new form of light-emittingcrystals, known as quantum dots.These dots can be used to produceultra-thin televisions. The tiny crys-tals are 100000 times smaller thanthe width of human hair. This can beprinted onto flexible plastic sheets(which can be rolled up) to producea paper-thin display.

Blue Brain Project

Science has advanced in the SecondMillennium in ways that we nowchallenge ourselves into doing whatwe could not have earlier. We haveembarked on an experiment to de-termine the ultimate particle of

WWW.UPSCPORTAL.COM62

UPSCPORTAL Current Affairs : http://upscportal.com/civilservices/current-affairs

Integrated Guidance Programme for IAS (Pre) - 2012http://upscportal.com/civilservices/online-course/integrated-free-guidance-programme

which all nuclei, atoms, moleculesand materials are made anywhereon earth or in the vast sky. We lookfor the “God particle”. We have sentman-made crafts to other planets,and have made machines and toolsthat enquire whether life exists else-where in the sky, and whether thereare other planets similar to ours thatmay supports life- “second earths”.We have read the “book of humanlife”, the 3.2 billion- letter-long codeof DNA that makes us what we are.There are two ways to approach thisgrand challenge. One is to try andunderstand the neurons (nerve cells)of “lower” organisms – worms, flies,fish, rats and such, and build on thisknowledge. This involves experi-ments on the “normal” organism andon its “mutants” – its cousins whoare born (or tampered with in the lab)with one or more neural problem.Many biologists are involved in suchexperiments, and several more di-rectly study humans with neurologi-cal problems and try to make senseout of the basis behind such errorsin the brain.

By the mid-1970s, information tech-nology had advanced to such a levelthat companies, notably IBM, hadthought of modelling the “thought”behind chess games that we humansplay. The advanced computers pro-gramming that they did at that timewas christened “Deep Thought” (aterm coined by researchers atCarnegie Mellon University, includ-ing Dr. Thomas Anantharaman). It isthese advances in computers thatled Dr. Henry Markram of EcolePolytechnique Federal de Lausanne,Switzerland, to think of creatingsupercomputer models of the brainthat would be accurate to the lastbiological details. To this end, he

has put together what he calls theBlue Brain Project (the blue heresymbolizing supercomputers).The approach of Blue Brain is binary.It uses the information availablefrom the hundreds of thousands ofpublications of neuroscientists onone hand, and ability of computerprogrammers to createconnectivities between the millionsof “neurons” in silico on the other.Combining the two, he expects tobuild a facility that would aim atdata integration and help build brainmodels.Gesture-based communica-tions are quite common in other pri-mates such as orangutans and chim-panzees, which cannot speak butuse a variety of hand, feet and limbgestures to communicate amongthemselves and also with their hu-man care-takers. Dr. Amy Follick ofEmory University, Atlanta has beenable to distinguish 31 manual ges-tures and is 18 facial/vocal gestures.

Bone Marrow CancerGenes Identified

Scientists identified genes that canincrease a person’s risk of develop-ing multiple myeloma by 30 percent.Multiple myeloma is an aggressiveform of bone marrow. A team at theInstitute of Cancer Research used atechnique known as genome wideassociation study to scan the DNAof 1675 patients with multiple my-eloma. When results were com-pared, scientists found that two re-gions of the DNA that were morecommon in people with multiplemyeloma and were therefore linkedto a higher chance of developing thedisease. Multiple myeloma is alsoknown as plasma cell myeloma orKahler's disease. It is a cancer ofplasma cells, a type of white blood

cell responsible for the productionof antibodies.

More Potent Ways toDesign HIV Drugs found

In perhaps the most comprehensivesurvey of the inner workings of HIV,an international team of scientistsled by researchers at the Universityof California, San Francisco hasmapped every apparent physical in-teraction the virus makes with com-ponents of the human cells it infects.This work may reveal new ways todesign future HIV/AIDS drugs. Inback-to-back papers published inthe journal Nature , the survey re-veals a pathogenic landscape inwhich HIV's handful of proteinsmakes hundreds of physical connec-tions with human proteins and othercomponents inside the cell. In onepaper, the team details 497 suchconnections, only a handful of whichhad been previously recognized byscientists.

Disrupting these connections mayinterfere with HIV's lifecycle, and theexistence of so many new connec-tions suggests there may be severalnovel ways to target the virus. In acompanion paper, Krogan and col-laborating labs investigated onesuch connection in detail. They dis-covered that an HIV protein calledVif makes a physical connection witha human protein called CBF-â, hi-jacking its function. The UCSF-ledstudy has provided the most com-prehensive and detailed picture todate of all the interactions HIV haswith the human cells it infects, andidentifying these interactions maylead to the development of newdrugs to treat the disease. Interfer-ing with this association may be a

WWW.UPSCPORTAL.COM 63

UPSCPORTAL Current Affairs : http://upscportal.com/civilservices/current-affairs

Integrated Guidance Programme for IAS (Pre) - 2012http://upscportal.com/civilservices/online-course/integrated-free-guidance-programme

way to block the virus. Ultimately, ifscientists can design compounds todo this safely and effectively, thosecompounds could form the basis fora new type of HIV/AIDS treatment.

Mechanism Controllingthe Fat in Human Body

Scientists made a key discoveryabout the mechanism controlling thefat in human body. It sheds new lighton how proteins regulate appetitecontrol and insulin secretion.This isthe first time such a mechanism wasdescribed and it's unique, showingthe importance of this protein tocellular function.CPT1 is the protein responsible forregulating fatty acid oxidation in theliver and is critical for metabolism.Its activity determines whether aperson suffers from fatty liver in onecase or ketosis in the other. The dis-covery will help in better under-standing of why some people has aspeedy metabolism and othersstruggle to control their appetite.Thiswould be a major breakthrough intackling the obesity crisis faced byhumans.

Diamond-StuddedPlanets

Recent findings have revealed thatsome stars in the Milky Way couldbe harbouring carbon super earths— gigantic planets completely be-reft of life but potentially holdingbillions of tonnes of diamond. Thefinding comes from a lab experimentat Ohio State University, where sci-entists recreated the temperaturesand pressures of earth's lowermantle to study how diamonds formthere. The goal was to understandwhat happens to carbon inside plan-

ets in other solar systems, andwhether solar systems that are richin carbon could produce planets thatare mostly made of diamond. WendyPanero, researcher in the School ofEarth Sciences at Ohio State alongwith doctoral student CaymanUnterborn used what they learnedfrom the experiments to constructcomputer models of the mineralsthat form in planets composed withmore carbon than earth. “It is pos-sible for planets that are as big as15 times the mass of the Earth to behalf made of diamond,” saidUnterborn, according to a universitystatement. Our results suggest car-bon-rich planets can form with acore and a mantle, just as Earth did,”said Panero. “However, the coreswould likely be very carbon-richmuch like steel and the mantle wouldalso be dominated by carbon, muchin the form of diamond,” he added.

National Seminar onEco-Friendly

Crop Protection

In recent years, there has been re-newed interest in popularizing or-ganic and consumer-safe pest man-agement technologies, to cater toorganic and export-focus cultivationof crops in India. A national seminaron Biotechnological approaches inorganic and eco-friendly crop pro-tection towards promoting knowl-edge transfer and technology aware-ness was organised recently by theSun Agro biotech research centre(SABRC) and the Department of Zo-ology, University of Madras inChennai. He also emphasized theneed to adequately support the or-ganic farming communities with ap-propriate technologies, especiallyeco-friendly pest control options to

be available at farm level. He alsorecalled the past initiatives ofSABRC in convening two such na-tional seminars earlier in the last fiveyears, with focus on organic andeco-friendly pest management tech-nologies. He explained that this suc-cessful campaign for mass produc-tion and field release of the importedbioagent was an outstanding suc-cess story in biological pest controlat national level. About 60 expertsfrom several ICAR institutes andother Universities participated.

Adult Stem Cellsin the Heart

An international team of scientistsled by Prof Richard Harvey of theUniversity of New South Wales andcolleagues at Victor Chang CardiacResearch Institute discovered a newpopulation of adult stem cells in theheart. The findings were piblishedfollowing recent reports that stemcells harvested from human heartsduring surgery show promise for re-versing heart attack damage. Scien-tists claimed in their findings, pub-lished in the latest edition of the 'CellStem Cell' journal that the adultstem cells can augment the devel-opment of new regeneration andrepair therapies for people who havesuffered cardiac failure. This is thefirst time this new population ofstem cells was formally described,and its origins clearly defined. Amouse was used as a model system.The first part of the study includedthe discovery and characterisationof a new population of multi-potent,adult stem cells that live in the heart. The stem cells were found to bepowerful in dividing, and respond-ing to their native environment toform whatever tissue is needed for

WWW.UPSCPORTAL.COM64

UPSCPORTAL Current Affairs : http://upscportal.com/civilservices/current-affairs

Integrated Guidance Programme for IAS (Pre) - 2012http://upscportal.com/civilservices/online-course/integrated-free-guidance-programme

repair. The new group of cells aremulti-potent, and highly specific tothe heart. These cells when trans-lated into the human setting is ex-pected to work well at regeneratingand repairing a broken heart or aheart that has suffered injurythrough heart attack or heartfailure. The scientists are of theopinion that population of cells arevery high up in the stem cell hierar-chy, and can generate a number ofprogenitor cells that would exist ina healthy heart, ready for action.

Hydro Gel Developedby Researchers

Scientists developed a hydro gel thatregenerated healthy and scar-freetissue on skin damaged by severeburns. The hydro gel helps in for-matting of new blood vessels andskin including hair follicles. The in-jured soldiers, fire victims andpeople with third degree burns canavail of the gel. Third-degree burnstypically destroy the top layers ofskin down to the muscle. The treat-ment involves, dressing up thewound, which includes hydro geland 3D framework of polymers.

Saguna Baugh

Saguna Baug is a farming outfit atMalegaon-Neral of Raigad District.It was set up as a role model to in-spire those who move away fromfarming. Started by a Gandhian free-dom fighter, popularly known asHarikaka Bhadsavle it is presentlybeing taken care by his son. Aftertwenty years of tedious and persis-tent efforts this beautiful agricul-tural project is wide-open for visi-tors.

Oxidative StressDamages DNA

Researchers have now decoded themechanism that repairs DNA dam-aged in this way. This repair mecha-nism could lead to less invasive ap-proaches in cancer therapy.

Pattern of HydrogenBonds

A pattern of hydrogen bonds be-tween odorant and receptor, ac-counting for the specificity of theolfactory sensors has been found.Computer simulations helped pre-dict if odorants activate a certainreceptor or not.

Bees, Flowering PlantsAdjust to Earlier Spring

An analysis of bee collection dataover the past 130 years shows thatspring arrives about 10 days earlierthan in the 1880s, and bees andflowering plants have kept pace byarriving earlier.

Maturation Clock

An increase in fruit yield might beachieved by manipulating a molecu-lar timer that determines the num-ber of branches that make flowers.

Small Spider’s BrainsOverflow into Their Legs

Smithsonian researchers report thatthe brains of tiny spiders are so largethat they fill their body cavities andoverflow into their legs. As the spi-ders get smaller, their brains get pro-portionally bigger.

New horned dinosaur announced A new species of horned dinosaurwas just announced by scientists,nearly 100 years after the initial dis-covery of the fossil.

Stradivarius Violin

Using computed tomography imag-ing and advanced manufacturingtechniques, a team of experts hascreated a reproduction of a 1704Stradivarius violin.

Fire & Wind

Firstly, consider the presence of alow velocity wind. ( say, virtually, stillair). At any moment of time, thiswind is just sufficient to sustain thefire. That is, it functions just as an‘element.' Obviously, it cannot pro-vide any motive force (kinetic en-ergy) for the fire (flame). Hence, thefire cannot spread. Ultimately, itblows out. Secondly, consider thepresence of a high velocity wind. Atany moment of time, there is not onlysufficient supply of air to sustain thefire but also there is adequate air toprovide motive force. Thus, its func-tion is two-fold: as an element andas a motive force. Hence, the firespreads.

Bacteria Live Inside Cells

Bacteria build camouflaged homesfor themselves inside healthy cells,and cause disease, by manipulatinga natural cellular process.

Vanishing Elephants,Rise of Modern Humans

With the disappearance of el-ephants in the Middle East, newhominids who had to be more agile

WWW.UPSCPORTAL.COM 65

UPSCPORTAL Current Affairs : http://upscportal.com/civilservices/current-affairs

Integrated Guidance Programme for IAS (Pre) - 2012http://upscportal.com/civilservices/online-course/integrated-free-guidance-programme

and skillfull to hunt smaller andfaster prey took over the MiddleEastern landscape and eventuallyreplaced Homo erectus.

300 Year OldJACK Tree

The tree is presently in the field offarmer Mr. K. Narasimhaiah and isnearly 300 years old and the treebears nearly 300 fruits annually,which the farmer sells for Rs 20,000-Rs.30,000.

Evolution & Skull Shape

Skull shape did not occur indepen-dently through separate evolution-ary events, but by actually precipi-tating each other.

Cassini DeliversHoliday Treats

Radio signals flying clear across thesolar system from NASA's Cassinispacecraft have delivered a holidaypackage of glorious images , includ-ing those of Saturn's moon Titan andother icy baubles orbiting Saturn.

New Kind of Metal

New experiments and computationshave revealed that iron oxide under-goes a new kind of transition underdeep-Earth conditions due to highpressures and temperatures (as at-oms, electrons are squeezed to-gether).

Darwin About Emotions

Contrary to what many psychologi-cal scientists think, people do not allhave the same set of biologically‘basic' emotions.

Genes of ThyroidCancer Found

Three genes that increase the riskof thyroid cancer, which has the larg-est incidence increase in cancersamong both men and women, havebeen discovered, raising importantdetails relevant to diagnosis andtreatment.

Why Alcohol MakesSome Aggressive

If people who focus on the here andnow, without thinking about the im-pact on the future, tend to be ag-gressive even when sober, the ag-gressive effect generally getsgreatly magnified when they aredrunk.

Foetal Arsenic Exposure

A study advances our understandingof the sources of exposure to ar-senic and the potential for consum-ing harmful levels of arsenic via rice.Arsenic exposure during pregnancyhas health risks to the foetus.

Steroids to Retina

Steroids hitching a ride into theretina on nanoparticles calleddendrimers offer a new way to treatage-related macular degenerationand retinitis pigmentosa. The ste-roids target damage-causing cells.

Cornea GeneDiscovered

Since a transparent cornea is essen-tial for vision, the eye has evolvedto nourish the cornea without bloodvessels. A gene plays a major role

in maintaining clarity of the corneain humans.

Elderly as Fast as Youngin Some Brain Tasks

Healthy older people can be trainedto respond faster in some decision-making tasks without hurting theiraccuracy — meaning their cognitiveskills in this area aren't so differentfrom younger adults.

Sauropoda Dinosaur

For the first time, the fossil remainsof sauropoda has been recorded inAntarctica. Until now, remains of theherbivorous dinosaurs had been re-covered from all other continentallandmasses.

Prehistoric Wood Found

Under the cold clear waters of LakeHuron, researchers have found afive-and-a-half-foot-long, pole-shaped piece of wood that is 8,900years old.

Brighter Future ForSolar Energy

The efficiency of conventional solarcells can be increased up to 66 percent by using an organic plasticsemiconductor material. It is basedon harvesting double the number ofelectrons from one photon.

Left-Handedness is aNormal Variant

Most of the time, left-handednessis simply a naturally occurring, nor-mal variant. In some cases, however,it is disadvantageous and may well

WWW.UPSCPORTAL.COM66

UPSCPORTAL Current Affairs : http://upscportal.com/civilservices/current-affairs

Integrated Guidance Programme for IAS (Pre) - 2012http://upscportal.com/civilservices/online-course/integrated-free-guidance-programme

reflect a genetic defect or early de-velopmental disturbance.

Microbes inMars-like Conditions

Microbes from ice within a lava tubehave been found to thrive in cold,Mars-like conditions. They can tol-erate temperatures near freezing,low oxygen levels, and can grow inthe absence of organic food.

New Vaccine AttacksBreast Cancer in Mice

A new vaccine developed at the Uni-versity of Georgia and the MayoClinic in Arizona dramatically re-duces tumours in a mouse modelthat mimics 90 per cent of humanbreast and pancreatic cancer cases.

Frogs Find Mates WithMatching Chromosomes

When it comes to love songs, femaletree frogs are pretty picky. Certainfemale tree frogs may be remarkablyattuned to the songs of mates whoshare the same number of chromo-somes as they do, says a new study.

Double Destruction Dueto ‘Double Tsunami'

The destructive tsunami generatedby the March 2011 Japan earthquakewas a ‘merging tsunami' thatdoubled in intensity over ruggedocean ridges, amplifying its destruc-tive power before reaching shore.

Record Massive BlackHoles Discovered

Astronomers have discovered thelargest black holes to date — twomonsters with masses equivalent to10 billion suns that are threateningto consume anything, even light,within a region five times the sizeof our solar system.

Solar Storms CouldSandblast the Moon

Solar storms and associated Coro-nal Mass Ejections (CMEs) can sig-nificantly erode the lunar surface ac-cording to a new set of computersimulations by NASA scientists. Thiscould also cause atmospheric lossfor Mars.

Comet LovejoySnapped

The recently discovered CometLovejoy has been captured in stun-ning photos and time-lapse videotaken from the European SouthernObservatory's Paranal Observatoryin Chile.

Global CO {-2}Emissions

Global carbon dioxide emissionsfrom burning fossil fuels have in-creased by 49 per cent in the lasttwo decades, according to the lat-est figures by an international team.

WWW.UPSCPORTAL.COM 67

UPSCPORTAL Current Affairs : http://upscportal.com/civilservices/current-affairs

Integrated Guidance Programme for IAS (Pre) - 2012http://upscportal.com/civilservices/online-course/integrated-free-guidance-programme

Cricket

India - West IndiesOne Day Series

India beat West Indies by 34 runs towin the fifth and final cricket one-day match and recorded an emphatic4-1 series triumph in Chennai on 11December 2011. Electing to bat, In-dia scored 267 runs with the loss ofsix wickets in the stipulated overs. Manoj Tiwary 104 and Virat Kohli 80were the top scorers for India. In re-ply, West Indies reached as close as233 in 44.1 overs. K. Pollard 119 andA Russel 53 were top scorers forWest Indies. Darren Sammy was thecaptain of West Indies whileVirender Sehwag was the captain ofIndian side for this series. GautamGambhir was the stand–in captainfor the last one–dayer. India’s ManojTiwary was declared Man of TheMatch and Indian batsman RohitSharma got Man of the Series Award.Manoj Tiwary and K. Pollard scoredtheir international one day career’sfirst centuries in this match. Indiaand West Indies played this five-oneday match series from 29 No-vember 2011 to 11 December 2011.

Science & TechnologyPakistan - Bangladesh

Test SeriesPakistan defeated Bangladesh byseven wickets in the second and fi-nal cricket Test in Dhaka on 21 De-cember 2011. Pakistan by defeatingBangladesh in the second test wasable to to sweep the series 2-0. Pa-kistan was set 103 to win after bowl-ing Bangladesh out for 234 in itssecond innings and achieved thetarget in the final session. Pakistancaptain Misbah-ul-Haq secured thevictory by hitting a six afterMohammad Hafeez made 47 andAzhar Ali chipped in with 34. Earlier,left-arm spinner Abdur Rehman took4-51 to restrict Bangladesh.

Tennis

Davis CupSpain won its fifth Davis Cup titleafter Rafael Nadal defeatedArgentina’s Juan Martin del Potro 1-6 6-4 6-1 7-6 (0) on 4 December2011. The victory earned Nadal hisfirst title since winning his sixthFrench Open in June 2011. Nadalwith the victory stretched his recordon clay to 16-0. Nadal earned his20th straight victory since debuting

in the competition in 2004. Nadalcaught Del Potro out with one of 35winners to take the second set andcruise through the third set. HoweverPotro’s late fightback broke Nadalfour times in the fourth set to sendit to a tiebreaker, where Nadal racedahead without losing a point beforesecuring victory with a forehand win-ner. Argentina remained the onlynation to play in four Davis Cup fi-nals and never win one.

Barclay’s ATPWorld Tour Finals

Swiss tennis star Roger Federer de-feated Jo-Wilfried Tsonga of Francein their singles final tennis match atthe Barclay’s ATP World Tour Finals,in the O2 arena in London on 27November 2011. The victory earnedhim a record sixth ATP World TourFinals title.

The victory also helped him moveahead of Pete Sampras and IvanLendl on the list of Tour Finals cham-pions in the 100th final of his career.

Federer has won 70 trophies in hiscareer and has also equalled Lendl'srecord of 39 match wins in the end-of-season event.

Federer received the winners'cheque worth $770,000. For the firsttime since 2002 he ended the sea-son without a Grand Slam title.

WWW.UPSCPORTAL.COM68

UPSCPORTAL Current Affairs : http://upscportal.com/civilservices/current-affairs

Integrated Guidance Programme for IAS (Pre) - 2012http://upscportal.com/civilservices/online-course/integrated-free-guidance-programme

Badminton

BWF World Super SeriesBadminton Championship

Indian ace badminton player andWorld No. 4 Saina Nehwal of Indiawas defeated by World No. 1 WangYihan 18-21, 21-13, 21-13 in thewomen’s singles final of the$500000 BWF World Super Seriesbadminton championship in Chinaon 18 December 2011. In 2011 Sainawon only the Swiss Open Super Se-ries title. Wang proved to be too goodfor the Indian shuttler in the crucialphases of the 72-minute final. Thescores were levelled twice, four-alland 11-all, before the Chinese pickedseven points in a row to take a huge18-11 lead. With this win, Yihan kepther 100 per cent record against Sainaintact, having now beaten her in alltheir four faceoffs so far. SainaNehwal had created history by be-coming the first ever Indian to enterthe singles final of the World SuperSeries. Gutta Jwala and V. Diju werethe only other Indians who made itto the final in mixed doubles in the2009 edition.

India Open Grand PrixIndonesia’s experienced shuttler,Taufik Hidayat, won his first Men’sSingles title of 2011 at the 2011$120,000 Syed Modi India OpenGrand Prix Gold badminton champi-onship when he defeated SourabhVerma of India. Taufik is currentlyWorld Number 12 in Men’s Singles

world rankings. Hidayat who endedhis title drought of 2011 with a 21-15, 21-18 triumph over Saurabh re-ceived $9,000 as the prize money.In the first set of this final battle,Taufik played with precision and wonit with a comfortable score. in thesecond game, Sourabh elevated hislevel of play and matched the speedof Taufik but lost the set with a de-cent margin. Ranked 72 in the worldand on his debut season on the se-nior international circuit, Sourabhhad won the Bahrain InternationalChallenge in 2011 and had alsoreached the quarterfinals of TATAOpen India International Challenge,Thailand Grand Prix Gold and IndianSuper series.

The women's singles final was wonby third seed Inthanon Ratchanok ofThailand. Naoki Kawamae & ShojiSato (Jpn) emerged winners in Men'sdoubles while in women's doubles,Shinta Mulia Sari & Lei Yao ofSingapore defeated Miyuki Maeda &Satako Suetsuna of Japan to emergechampions.

Motor Racing/F1

JK Tyre-FMSCI National RacingChampionship

Ameya Walavalkar of Mars Racingregistered his sixth win of 2011 inthe Formula LGB Swift class, therebyclinching his maiden drivers' cham-pionship title in the JK Tyre-FMSCINational Racing Championship at theMMRT in Irungattukottai on 27 No-vember 2011. Walavalkar finished

22 points ahead of teammate R.Deepak, who came fifth in the race.Sarosh Hataria of Meco Motorsportsstood second. Karthik Krishna fin-ished third but was penalised forovertaking when the yellow flag wasbeing waved and thus conceded hispodium place to Chitesh Mandody.Rahil Noorani, a championship con-tender in the Polo R Cup series wasexcluded from the competition aftera post-race incident in the pits afterhe vent his frustration at Ajay Kini.

Golf

Goodricke East India AmateurGolf Championship

Delhi's Honey Baisoya became theyoungest winner of the 22ndGoodricke East India amateur golfchampionship at RCGC on 25 De-cember 2011. The 15-year-old de-feated the country's No.1 amateur,S. Chikkarangappa to win 3 and 2.Both had to play two rounds of 36holes on the final day. Baisoya se-cured a winning start after makingbirdies on first, fourth, fifth, ninth,12th and 15th in the first round of18 holes where both Baisoya andChikka halved 11 holes. Though,Chikka won on seventh, 11th and14th, Baisoya was 2-up after the firstround. In the second round, Baisoyaagain took an early lead after mak-ing a birdie on the first to go 3-upthough Chikka kept giving a toughcompetition. Baisoya in total had sixbirdies in the first round of 18 andthree more in 16 holes in the sec-

WWW.UPSCPORTAL.COM 69

UPSCPORTAL Current Affairs : http://upscportal.com/civilservices/current-affairs

Integrated Guidance Programme for IAS (Pre) - 2012http://upscportal.com/civilservices/online-course/integrated-free-guidance-programme

ond round while Chikka had threebirdies in both rounds. Baisoya alsodropped fewer bogeys.

Football

Indian Player of the YearIndia striker Sunil Chhetri was on 20December 2011 voted Player of theYear by the All India Football Fed-eration. He was chosen by I-Leaguecoaches from a five-player shortlist.He will receive a cash award of Rs 2lakh and a silver plaque. The crite-ria of selection were Performance forIndia and general conduct on and offthe field. Chhetri scored 13 goals in17 internationals in 2011, whichwas the highest the by an Indian.Chettri has also scored 11 goals in20 appearances for I-League clubMohun Bagan. He won the man ofthe championship in the SAFF Cham-pionship in New Delhi in 2011. Hehas also been awarded the presti-gious Arjuna Award in the past.I.M.Vijayan was the first recipient ofthe award in 1992. In 2010 the awardwas bestowed on Gouramangi Singh

6th SAFF ChampionshipIndia won its 6th SAFF Championshiptitle against the newcomer, Afghani-stan on 11 December 2011. India'sstar striker Sunil Chhetri won all theindividual awards in the tournament.India was a five time SAFF championbefore it won the 2011 SAFF. Theother participating nations in the2011 South Asian Football Federa-

tion Championship (SAFF) were Af-ghanistan, Sri Lanka, Bhutan,Bangladesh, Maldives, Nepal andPakistan. Winner India received aprize purse of USD 50,000 while Af-ghanistan took home USD 25,000. Inaddition, All India Football Federa-tion announced a cash award of Rs.1 lakh for each player for winningthe tournament.

Hockey

Champions Challenge HockeyTournament

Belgium defeated India 4-3 in the final of the Champions Challengehockey tournament in Johannesburg,South Africa on 4 December 2011.The victory enabled Belgium, whoare already through to the 2012 Lon-don Games, to qualify for the nextChampions Trophy in Australia. TheBelgium sise was coached by ColinBatch. Tom Boon scored the winninggoal for Belgium and in the processbecame the joint top scorer of thetournament along with Justin ReidRoss of South Africa with eight goals.VR Raghunath, Sandeep Singh andShivender Singh scored for India.Sandeep Singh finished the tourna-ment with seven goals, all throughpenalty corners, and won the Playerof the Tournament award.

Four-nation TournamentThe Indian women's hockey teamdefeated Ireland 4-1 win to clinchthe bronze medal in the four-nationtournament held in Parana, Argen-

tina in December 2011. Both theteams were locked 1-1 at half-timebut India came back strongly in thesecond session to score three goals.Jaspreet Kaur, Anuradha Thokchom,Vandana Katariya and Preety SunilaKiro 69thscored for India India thuswon bronze medals at both the four-nation tournaments held at BuenosAires and Parana on the current tour.In the final World champion Argen-tina defeted South Africa 2-0 to winthe tournament.

Archery/Shooting

32nd Sahara Senior NationalArchery Championship

The Archery Association of India(AAI) announced in late December2011 that the 32nd Sahara seniorNational archery championship(recurve and compound) will be heldat the JRD Tata Sports Complex,Jamshedpur from 24 to 29 January2012. The meet is to serve as theselection trials for aspirants of aplace in the Indian archery team forthe first Asian Grand Prix at Bangkok scheduled to be held 14 to 20 Feb-ruary 2012. The best five finishersin the men's recurve section will viefor three places in the Indian team,and the best five women will vie forone spot in the women's recurveteam at a selection trials to be heldat SAI, Eastern Centre, from 3 to 6February 2012. The AAI retainedJayanta Talukdar and the threewomen archers, Chekrovolu Swuro,Deepika Kumari and L. Bombayla

WWW.UPSCPORTAL.COM70

UPSCPORTAL Current Affairs : http://upscportal.com/civilservices/current-affairs

Integrated Guidance Programme for IAS (Pre) - 2012http://upscportal.com/civilservices/online-course/integrated-free-guidance-programme

Devi, in the Indian team for all in-ternational competition till the Lon-don Olympics since they earned thecountry four Olympic quota places.

Athletics

Fukuoka Men's MarathonJosphat Ndambiri of Kenya won theFukuoka men's marathon in Japanwinning the 42.195km race in 2hrs7mins and 36secs on 4 December2011. Mwangi stood second. BothNdambiri and Mwangi currently re-side in Japan. Mwangi crossed theline one minute and two secondsafter Ndambiri who had opened upa sizeable lead following the 35kmmark. Yuki Kawauchi of Japan fin-ished in third place with a time oftwo hours, nine minutes and 57 sec-onds. The Fukuoka marathon, nowin its 64th year, is widely regardedas a prestigious title. In 2011 theocassion was treated as valuablepreparation for next year's Olympicsin London.

Chess

World Women's Team ChessChampionship

The Indian team finished at thefourth position in the WorldWomen's Team Chess Champion-

ship at Mardin,Turkey on 27 Decem-ber 2011. India's chances of winninga medal was blown away followinga loss to Georgia and then China.The losses by Tania Sachdev andSoumya Swaminathan of India pavedthe way for an easy Georgian vic-tory in the end.

China won the gold medal with 16points while the silver was claimedby the Russians who finished with13 points in all. Georgia ended withthe bronze while India and Ukrainewere tied for the fourth spot. Ukrainecaused the biggest surprise of thechampionship with a sensational2.5-1.5 victory over leaders China.Though China was not much af-fected, the helped the Ukrainianeves come into reckoning after arather subdued performance in thefirst half.

London Chess Classic’s OpenIndian Grandmaster Abhijeet Guptawon the London Chess Classic'sopen section, with a hard-earnedvictory over Grandmaster KeithArkell of England. Abhijeet, a formerworld junior champion and reigningnational champion, scored eightpoints in all. The chess tournamentwas held from 3 December 2011 to12 December 2011.

Another Indian, International Mas-ter Sahaj Grover pulled himself to thesecond spot after defeating EnglishGrandmaster Peter Wells in the fi-nal round of the event. Sahaj, whoscored 7.5 points in all.

Volleyball

Vajpayee CupKerala defeated Andhra Pradesh instraight sets to lift the men's title inthe 10th All-India Invitational volley-ball tournament for the VajpayeeCup on 25 December 2011. Keralawon the game 25-20, 25-18, 25-18.Karnataka outdid Tamil Nadu to oc-cupy the third position in the men’scategory.

The Kerala on the other hand de-feated Central Railway (Mumbai)and to claim the titlee. Tiji Raju andN. Bijina led the turnaround as it won20-25, 18-25, 25-23, 26-24, 15-13.The women had to come back fromtwo sets down to take the title. TheKerala women lost the first set 20-25 before finding themselves indeep trouble after losing the secondset 18-25. In the deciding set Keralaoutdid Central Railway 15-13 toclinch the title. Southern Railway(Chennai) finished third in thewomen's sections.

WWW.UPSCPORTAL.COM 71

UPSCPORTAL Current Affairs : http://upscportal.com/civilservices/current-affairs

Integrated Guidance Programme for IAS (Pre) - 2012http://upscportal.com/civilservices/online-course/integrated-free-guidance-programme

UPSCPORTAL

BANK POPRACTICE SET

UPSCPORTAL’S

fgUnh lkfgR;izSfDVl lsV (eq[; ijh{kk)

vkbZ- ,- ,l@ih- ,l lh ijh{kkvksa ds fy,mi;ksxh

CSAT PAPER-1GENERAL STUDIESQUESTION BANK

UPSCPORTAL’S

UPSCPORTAL

GENERALSTUDIESIAS (MAINS)

Solved Paper

Buy Online at:www.upscportal.com/order-books

WWW.UPSCPORTAL.COM72

UPSCPORTAL Current Affairs : http://upscportal.com/civilservices/current-affairs

Integrated Guidance Programme for IAS (Pre) - 2012http://upscportal.com/civilservices/online-course/integrated-free-guidance-programme

Sahitya AkademiAwards

Twenty-two writers selected on 21December for the prestigious SahityaAkademi awards for2011. Eightbooks of poetry, seven novels, threeessays and one each short stories,narrative history, biography and playwere selected for the awards. Writ-ers in 23 languages are recongisedevery year by the Akademi.The bookswere selected on the basis of rec-ommendations made by a jury ofthree members in each language.The awards that are given every yearand contain an engraved cooperplaque, a shawl and a cheque of Rs1 lakh.

Sahitya Akademi awards for Po-etry Poets chosen for the awardincludes: Late Kabin Phukan(Assamese), Manindra Gupta(Bengali), Premananda Mosahari(Bodo), Naseem Shafaie (Kashmiri),Melvyn Rodrigues (Konkani),Harekrishna Satapathy (Sanskrit),Aditya Kumar Mandi (Santali) andKhaleel Mamoon (Urdu).

Sahitya Akademi awards for Nov-els Novelists who won the awardinclude: Kashinath Singh (Hindi),Kshetri Bira (Manipuri),Kalpanakumari Devi (Odia), BaldevSingh (Punjabi) and Atul Kanakk(Rajasthani).

Award & PrizesTamil novelist Su Venkatesan's , de-but novel Kaval Kottam earned himthe coveted Sahitya Akademi Awardin Tamil section. The book is aboutthe 600-year old history of the cityof Madurai and a security systemthat prevailed there

Awardees in Essays CategoryLalit Magotra (Dogri), Grace(Marathi) and Samala Sadasiva(Telugu) received the award for theirbooks on essays.

Awardees in Other SectionsHistorian Ramachandra Guha waschosen for the award for his book ofnarrative history - India AfterGandhi. Mohan Parmar (Gujarati)was chosen for his short stories, M.K.Sanu (Malayalam) for his biographyand Mohan Gehani (Sindhi) for hisbook on plays.

Bharatendu HarishChandra Awards

Fourteen persons (Leading writersand journalists) were givenBharatendu Harish Chandra Awardsfor 2009 and 2010 by Minister ofState for Information and Broadcast-ing, S Jagathrakshakan on 28 De-cember 2011.

2009 WinnersThe first prize in the Journalism andMass Communication category forthe year 2009 was awarded to DilipChandra Mandal for his

manuscript Corporate, Loktantra aurPaid News. Kumud Sharma’sbook Samachar Bazar kiNaitikta bagged the second prize.The third prize went jointly toShivananda Kamde for his book Car-toon Patrakarita and Dr. Akela Bhaifor his book Radio Sahitya aurPatrakarita. In the Women’s issuescategory, the first prize for 2009 wasgiven to Lata Kot for hermanuscript Adha Asmaan Hamara.The first prize in Children’s Litera-ture category for 2009 was given toGhamandi Lal Agrawal for his book‘Geet Gyan Vigyan Ke’ and the sec-ond prize to Renu Saini for her manu-script ‘Bachpan ka Safar’.

2010 WinnersFor 2010 the second prize in Jour-nalism and Mass Communicationcategory was awarded to PranjalDhar’s manuscript SamkaleenVaishwik Patrakarita Mei Akhbaar.No one was awarded first prize for2010 in this category. In Women’sissues category for 2010, SumanRai’s book Gharelu Hinsa meinMahila Sanrakshan Adhiniyam 2005,2006 got the first prize. SanjeevJaiswal Sanjay’s book Dooba HuaQila and the book Cycle Par ThaaKavva by Prabhat received the firstand second prize respectively in theChildren’s Literature category. In theNational Integration category for2010, Shiv Kumar Rai’s manuscript‘Meri Jati Bhartiya’ was given thefirst prize.

WWW.UPSCPORTAL.COM 73

UPSCPORTAL Current Affairs : http://upscportal.com/civilservices/current-affairs

Integrated Guidance Programme for IAS (Pre) - 2012http://upscportal.com/civilservices/online-course/integrated-free-guidance-programme

About Bharatendu Harish ChandraAwards: The BhartenduHarishchandra Awards are given topublished or unpublished books infour categories- Journalism andMass Communication, Women’s Is-sues, Children’s Literature and Na-tional Integration. The awards wereinstituted in 1983 to encourage origi-nal Hindi writings on topics relatedto journalism and mass communica-tion. Awards for writings on women’sissues, children’s literature and na-tional integration were included infrom 1992-93. The Awards schemeis coordinated by Publications Divi-sion, Ministry of Information andBroadcasting, Government of India.The Ministry of Information & Broad-casting, increased the BhartenduHarishchandra award money fromthe year 2009.

For journalism and mass communi-cation categories, the first prize nowcarries an award of Rs 75000, upfrom Rs 35000 earlier. The amountfor the second prize was increasedfrom Rs 25000 to Rs.50000 and thethird from Rs.20000 to Rs.40000. Forbooks in category of Women’s Issues,Children’s Literature and NationalIntegration, the first prize carries anaward of Rs 40000 instead ofRs.15000 and the second prizeRs.20000 instead of Rs.10000.

Mahatma GandhiInternational Award

Tibetan spiritual leader, the DalaiLama was presented the MahatmaGandhi International Award forPeace and Reconciliation on 4 Janu-ary 2011, the fourth day of the ten-day Kalachakra initiations. The granddaughter of Mahatma Gandhi, IlaGandhi, presented the award, insti-

tuted in 2003, to the Dalai Lama atthe Kalachakra ground. Ila Gandhi,who is based in South Africa, ischairman of the Gandhi Develop-ment Trust that gives the award. Ti-betan Nobel Peace Laureate on theoccasion encouraged everyone touse human intelligence in challeng-ing and overcoming negative emo-tions with positive emotions. Hementioned in his speech that disar-mament is needed to attain com-plete non-violence. He also men-tioned that mental peace results innon-violent actions and mental un-rest causest he opposite. He con-cluded by stating that the real pathto a happy life is not through moneyn power but through honesty andcompassion. The Dalai Lama wassupposed to have received theaward in October 2011 as part of amuch anticipated visit to South Af-rica. The visit however had to becalled off following a five-week waitfor a visa to be issued. Former re-cipients of the Mahatma Gandhi In-ternational Award include Burmesepro-democracy leader Aung San SuuKyi, South African leader NelsonMandela and Tanzania’s JuliusNyerere.

Justice P.N. BhagwatiAward

Former president A.P.J. Abdul Kalampresented the Justice P.N. BhagwatiAward to O.P. Jindal Global Univer-sity chancellor and managing direc-tor of Jindal Steel and Power Ltd,Naveen Jindal on 6 December 2011.The award was conferred upon Jindalfor his outstanding contribution tolegal education and corporate phi-lanthropy. Justice P.N. BhagwatiAward has been instituted by theCapital Foundation. Naveen Jindal is

the Founding Chancellor of the O.P.Jindal Global University. He is alsoa Member of Parliament, the Chair-man and Managing Director ofJindal Steel and Power Ltd (JSPL),and a sports enthusiast. The O.P.Jindal Global University has fourschools: Jindal Global Law School(JGLS), Jindal Global Business School(JGBS), Jindal School of Interna-tional Affairs (JSIA) and Jindal Schoolof Government and Public Policy (JGSP). The Bhagwati Award wasalso given to Arun Mohan, a senioradvocate in the Supreme Court, inrecognition of his outstanding con-tribution to access to justice. HumanResource Development MinisterKapil Sibal and Law and Justice Min-ister Salman Khurshid were con-ferred the Justice V.R. Krishna IyerAward on the occassion.

Life Time AchievementAward

Lord Swraj Paul, leading NRI indus-trialist was conferred with the LifeTime Achievement award for hisoutstanding contributions in variousfields. The award, instituted by anIndian company Powerbrands, waspresented to Lord Paul on 12 De-cember 2011 by Subrata Roy, chiefof the Sahara Group.

Lord Paul, Chairman of the CaparoGroup, dedicated the award to theordinary people of India who he fellshave little, but never complains. Phi-lanthropist and Labour politician,Lord Paul is the chancellor of theUniversity of Wolverhampton andthe University of Westminster. Paulwas awarded the Padma Bhushan bythe Government of India in 1983. Hewas elevated to the British Peeragein 1996. Raghupati Singhania of J K

WWW.UPSCPORTAL.COM74

UPSCPORTAL Current Affairs : http://upscportal.com/civilservices/current-affairs

Integrated Guidance Programme for IAS (Pre) - 2012http://upscportal.com/civilservices/online-course/integrated-free-guidance-programme

Tyres was earlier given thePowerbrands Hall of Fame Award.

42nd International FilmFestival of India (IFFI)

The 42nd edition of InternationalFilm Festival of India (IFFI) was heldin Panaji, Goa from 23 November to3 December 2011. Film star ShahRukh Khan inaugurated the 42ndedition of IFFI. The inaugural cer-emony was marked by presentationof Life Time Achievement award,which was revived after almost 10years to French film maker BertrandTavernier by Shah Rukh Khan. Theopening film of the festival was TheConsul of Bordeaux, directed byFrancisco Manso and Joao Correa.The 42nd IFFI came to an end withthe Asia premiere of the film TheLady directed by Luc Besson. For thefirst time, the inauguration ceremonywas held 30 km away from the mainaction point. There was a first-everopen screen on a football ground forfree display of soccer films. The fes-tival was the first to feature the newsections like 3D, Animation(Sketches on Screen) and FestivalKaleidoscope films.

Awards presented during the Fes-tival: The 42nd International FilmFestival of India come to an end atthe Kala Academy in Panaji with thepresentation of awards in five dif-ferent segments. The Golden Pea-cock Award for the Best Film wasgiven to a Columbian Film Porfirio.The Chief Guest of the closing cer-emony, Southern Superstar, Suriyapresented the award to the DirectorAlejandro Landes and Producer,Francisco Aljure. The Silver PeacockAward for Best Actor (Female) cat-egory was given to the actor

Nadezhda Markina for her role in theRussian film Elena. Whereas, theSilver Peacock Award for Best Actor(Male) category was won by SassonGabey for his role in an Israeli filmRestoration. The Special Jury Awardwas presented to Salim Ahamed fordirecting Malayalam film AdaminteMakan Abu meaning Abu, Son ofAdam. The Special Jury Award car-ries a Silver Peacock Trophy, a cer-tificate and Rs. 15 lakh. The fivemembers Jury under the chairman-ship of Adoor Gopalakrishnan hadscreened 14 films from as manycountries and unanimously decidedthe awards.

Gomant Vibhushan

World renowned architect and Goa'sson-of-the-soil Charles Correa wasbestowed with Goa's highest civil-ian honour, the Gomant Vibhushan,on the eve of the Golden Jubiliee ofGoa's Liberation on 18 December2011. The award was presented tohim by Goa governor KShankaranarayanan. Goa governorpresented Correa with the over half-metre high, glass-encased brass rep-lica of Goa's state symbol along witha cash award of Rs 5 lakh and a ci-tation in one of Correa's own archi-tectural creations, the Kala Academyat Panaji. Correa's architectural ge-nius and projects dotsnot only Indiancities but also cities around theworld, including Boston and Lisbon.Correa is the second recipient of theGomant Vibhushan, the first beingeminent nuclear scientist AnilKakodkar, who was awarded theGomant Vibhushan in 2010. Correahas earlier been conferred withIndia's Padma Shri and PadmaVibhushan.

Clean DevelopmentMechanism Project

Award

Delhi Metro Rail Corporation(DMRC)won a green award for reducingemissions of polluting gases in thenational capital by 6.3 lakh tons ev-ery year. The best Clean Develop-ment Mechanism (CDM) projectaward was presented at Urban Mo-bility Conference, 2011 which con-cluded in New Delhi on 7 December2011. The best CDM award wasgiven to Delhi Metro for for its ModalShift Project under which it becamethe first Metro rail and rail-basedsystem in the world to be certifiedby United Nations for getting carboncredits for reducing green housegas(GHG) emissions. Delhi Metro,which ferries around 18 lakh peopledaily, helped to reduce pollution lev-els in the city by 6.3 lakh tonnes ev-ery year, thus helping in reducingglobal warming. DMRC director(Works) Mangu Singh received theaward on the organisation's behalf.This is the second CDM project fromDMRC to be registered with the UNin the last three years. DMRC's firstCDM project on regenerative brak-ing had also achieved many inter-national firsts apart from earningvaluable foreign exchange for thecountry.

Grammy Award

The Academy of Recording Arts andSciences on 21 December 2011chose Apple co-founder Steve Jobsto be posthumously honoured witha Grammy award for his contributionto music technology. Jobs is to behonoured as a creative visionary. Heis one of 11 people who will be

WWW.UPSCPORTAL.COM 75

UPSCPORTAL Current Affairs : http://upscportal.com/civilservices/current-affairs

Integrated Guidance Programme for IAS (Pre) - 2012http://upscportal.com/civilservices/online-course/integrated-free-guidance-programme

honoured with special awards. Jobs,who died on 5 October 2011 of pan-creatic cancer, will receive a Trust-ees Award on 13 February 2012 forhelping create products that trans-formed the way music, TV, moviesand books are consumed. Apple Com-puter Inc first received a TechnicalGrammy Award in 2002 for contri-butions of outstanding technical sig-nificance to the recording field.Bandleader and composer DaveBartholomew, recording engineerRudy Van Gelder will also receive theaward along with Jobs. The academyalso picked the Allman Brothers,Glen Campbell, Diana Ross and Bra-zilian pianist/singer/guitarist Anto-nio Carlos Jobim as recipients of Life-time Achievement Awards. Jobimwas known for composing The Girlfrom Ipanema, a worldwide hit in themid-1960s which won a Grammy forRecord of the Year in 1965. Trumpeter Wayne Jackson, saxo-phonist Andrew Love of the Mem-phis Horns, country legend GeorgeJones, and the late Gil Scott-Heronwere also named as recipients. Ger-man sound-technology firmCelemony and the late audio engi-neer Roger Nichols, who worked withartists including Ross, PlacidoDomingo, James Taylor and StevieWonder, were also recognised withGrammy Technical Awards. TheTrustees Awards are given to peoplewho make a contribution to musicbeyond performance. The Academyof Recording Arts and Sciences alsochose seven performers to receiveLifetime Achievement Awards: TheAllman Brothers Band, GlenCampbell, Antonio Carlos Jobim,George Jones, The Memphis Horns,Diana Ross, Gil Scott-Heron.

Shakti BhattFirst Book Prize

Jamil Ahmad was declared the win-ner of the Shakti Bhatt First BookPrize for his book The WanderingFalcon at the British Council in NewDelhi on 21 December 2011. TheWandering Falcon, is a narrativeabout the lives of tribal people alongthe borders of Afghanistan, Pakistanand Iran. The book contained per-sonal sights, sounds, actual facesand conversations. The book thathas won critical acclaim worldwidewas written decades ago and thenwent into hibernation for 40 years.He started writing the book in 1971,it was completed by 1973 and thenit hibernated for 40 years.

Born in Jalandhar in 1933, JamilAhmad is a Pakistani national. Hehad not agreed to the demands ofthe publishers, which eventually ledto the book being locked away forfour decades. The book was pub-lished by Hamish Hamilton — animprint of Penguin earlier in 2011.Ahmad was posted in Quetta in 1959where he first came into contactwith tribals and was fascinated bytheir lives and their stories.The former civil servant from Paki-stan, Ahmad gave away the prizemoney of Rs. 1 lakh to St. Columba'sSchool, where he was enrolled from1941 to 1944. The prize is set up inmemory of writer and editor ShaktiBhatt. A shortlist of six books wasannounced ahead of the prize cer-emony. The shortlist included TheCollaborator by Mirza Waheed, TheTruth About Me by A. Revathi,Chinaman by Shehan Karunatilaka,A Free Man by Aman Sethi and R.D.Burman: The Man, The Music byAnirudha Bhattacharjee and Balaji

Vittal. The winner was chosen by ajury comprising graphic novelist andillustrator Sarnath Bannerjee, writerand blogger Jai Arjun Singh, and nov-elist Palash Krishna Mehrotra.

Sangita Kalanidhi M.S.Subbulakshmi Award

S.R. Nathan, former President of theRepublic of Singaporepresented theSangita Kalanidhi M.S.Subbulakshmi Award to SangitaKalanidhi-elect Trichy Sankaran atthe inauguration of the 85th annualconference of the Music Academy inChennai on 15 December 2011. S.R.Nathan also inaugurated the 85thannual conference of the MusicAcademy. On the occassion the sec-ond volume of the SangitaSampradaya Pradarsini waslaunched. The Music Academy andthe Singapore Indian Fine Arts Soci-ety (SIFAS) exchanged a memoran-dum of understanding on educationand training in Carnatic music, draft-ing of syllabus and training of teach-ers during the inaugral ceremony.The Music Academy pioneered theMargazhi music festival in Chennai,which had evolved into a wondrousphenomenon that greatly enrichedthe cultural underpinning of the cityover the last eight decades. The fes-tival remains unparalleled for itssheer size, aesthetics, quality andundiluted classicism.

Trichy Sankaran:Trichy Sankaran isthe mridangam maestro and flag-bearer of the Pudukottai' percussivetradition. Trichy is 2011’s designatefor the Academy's Sangita Kalanidhi.Trichy Sankaran, who is based inToronto became the first recipient ofthe Sangita Kalanidhi title in thePudukkottai parampara of his guru

WWW.UPSCPORTAL.COM76

UPSCPORTAL Current Affairs : http://upscportal.com/civilservices/current-affairs

Integrated Guidance Programme for IAS (Pre) - 2012http://upscportal.com/civilservices/online-course/integrated-free-guidance-programme

Palani Subramania Pillai. SangitaKalanidhi M.S. Subbulakshmi Award has been instituted by The Hindu.The award carries a cash prize of Rs.1 lakh.

WWW.UPSCPORTAL.COM 77

UPSCPORTAL Current Affairs : http://upscportal.com/civilservices/current-affairs

Integrated Guidance Programme for IAS (Pre) - 2012http://upscportal.com/civilservices/online-course/integrated-free-guidance-programme

In The NewsAppointed

Kim Jong-unNorth Korea declared the young un-tested heir Kim Jong-un as supremehead of the country, as tens of thou-sands of people rallied in Pyongyangone day after the funeral of his fa-ther, Kim Jong-il, to swear their alle-giance to the dynastic transfer ofpower. Kim Jongun, believed to bein his late 20s, was unveiled as suc-cessor in September last year, fol-lowing his father's 2008 stroke. Af-ter his father's sudden death Dec.17, he was rapidly elevated to thetop military and party posts, al-though he has yet to assume thoseofficial titles. How much he has con-solidated his grip on power beforehis father's death and whether hewould have to depend on caretak-ers or even regents remain topics ofintense speculation and contentionamong outside analysts. All indica-tions from the North, however, sug-gest that at least in the public eye,he will not share power with anyone.North Korea said the "great succes-sor" will faithfully follow his father's

songun, or "military-first," policy,which has raised tensions withWashington and Seoul.

Vinod RaiThe Comptroller and Auditor General(CAG) of India, Vinod Rai was on 8December 2011 elected chairman ofthe United Nations panel of exter-nal auditors. Rai succeeded NorbertHauser, vice-president of Germany'sFederal Court of Auditors as chair-man of the United Nations panel ofexternal auditors. As head of thepanel, Rai will shoulder the respon-sibility of overseeing the externalaudit works and periodical reportson accounts and management of theUN and its specialised agencies. Aschairman Rai will oversee the auditof the International Atomic EnergyAgency (IAEA), besides setting au-dit agenda for all UN specializedagencies such as the World HealthOrganization (WHO), World FoodProgramme and the World Intellec-tual Property Organization. JagbansSingh, director general in the officeof the CAG, was elected chairmanof the Technical Group of the UNpanel. Much of the external auditors'work is determined by the technicalgroup that is mandated to set spe-

cific topics of interest to be takenup for audit.

The accounts and management op-erations of the UN, the specializedagencies (WHO, WFP, WIPO etc.)and the IAEA are required to be au-dited and reported periodically byindependent external auditors, whoare supreme audit institutions ofvarious countries selected by theUN. Rai will give direction to theexternal auditors and will have a fi-nal say in preparation of the reportsbefore they are placed to the UNGeneral Assembly. The findings andrecommendations of United Nationspanel of external auditors are takenseriously, and the reports are closelymonitored for effective implementa-tion.

Ajit SinghRashtriya Lok Dal (RLD) leader AjitSingh was inducted into the UnionCabinet as civil aviation minister on18 December 2011. PresidentPratibha Patil administered the oathof office and secrecy to Singh at aceremony in Rashtrapati Bhavan.The Jat leader will face the challengeof reviving the aviation sector ingeneral and the government-owned

WWW.UPSCPORTAL.COM78

UPSCPORTAL Current Affairs : http://upscportal.com/civilservices/current-affairs

Integrated Guidance Programme for IAS (Pre) - 2012http://upscportal.com/civilservices/online-course/integrated-free-guidance-programme

Air India in particular as the new CivilAviation minister. The Civil AviationMinistry was held by Overseas In-dian Affairs Minister Vayalar Raviafter the elevation of Praful Patel asCabinet Minister.

Prof. Parvin SinclairNoted mathematician and the Pro-Vice Chancellor, IGNOU Prof. ParvinSinclair was appointed the Directorof National Council of EducationalResearch & Training (NCERT) on 25December 2011. Parvin will hold thepost for five years or until furtherorders, whichever is earlier. A Ph. .Din Mathematics from the Universityof Mumbai , Prof. Parvin Sinclair hasworked in the field the EducationalResearch and Training since 1979.Parvin Sinclair is currently a Profes-sor with the Indira Gandhi NationalOpen University. Prof parvin Sinclairis known for her work in the field ofmathematics and designing ofcourse material to suit learners ofthe subject on the distance mode.With a MSc in Mathematics from IIT-Delhi, Sinclair worked with the TIFR,Mumbai, till 1987 after which shejoined the IGNOU. She has been en-gaged in research work on ways ofcommunicating mathematics to chil-dren of all age groups. Sinclair wasalso Chief Advisor, Mathematics Syl-labus Design Committee, NCERT, in2005 and 2006.

Moncef MarzoukiThe National Constituent Assembly

(ANC) of Tunisia elected the leaderof the party, Council for the Repub-lic (CPR), Moncef Marzouki as thenew President of Tunisia on 12 De-cember 2011. He became Tunisia'sfirst elected president since theNorth African country's revolutionsparked the Arab Spring. He was aleader of the centre-left ‘Congressfor the Republic’ party. Marzoukibecame interim president with 153out of 217 votes in the assembly, withthree voting against, two absten-tions and 44 blank ballots cast as aprotest vote by some members.Marzouki’s appointment followed ananimated debate within the Con-stituent Assembly focused on theformation of Tunisia’s provisionalconstitution. He had the support ofhis group and the other two majorforces in the Tunisian Assembly, theNahda Islamic Movement and theAtakatol party.

Rita MenonFormer Textiles Secretary, RitaMenon on 3 January 2011 took overas the Chairman-cum-Managing Di-rector of the India Trade PromotionOrganisation (ITPO), whichorganises exhibitions and tradefairs. She succeeded Rajeev Kher,Additional Secretary in the Com-merce Ministry, who was officiatingas the CMD of the organisationsince August 2011. Rita Menon is a1975 batch IAS officer of UttarPradesh cadre who retired as Tex-tiles Secretary on 31 December

2011. ITPO, among other fairs, isknown for its annual flagship eventIndia International Trade Fair.

M.P. MuralidharanThe Union government on 5 January2012 cleared two key military ap-pointments. Vice Admiral M.P.Muralidharan took charge as the19th Director-General of the IndianCoast Guard. The position of CoastGuard Chief lay vacant since Vice-Admiral Anil Chopra was promotedas the Eastern Naval Command chiefin October 2011. Lt-General SanjeevChachra was appointed the nextArmy military secretary. Prior to tak-ing over as the Director-General, ViceAdmiral Muralidharan was the Chiefof Personnel at the Integrated Head-quarters (IHQ), Ministry of Defence.Earlier he had served as the firstCommandant of the Indian NavalAcademy at Ezhimala. An alumnusof the National Defence Academy,he was commissioned into the Navyin 1975. During the last three andhalf decades, he held several signifi-cant operational and staff appoint-ments. The Admiral is an alumnusof National Defence Academy andwas commissioned into the IndianNavy on 1 Jan 1975. A Navigationand Direction Specialist, he is alsoa graduate of Defence Services StaffCollege, Willington.

Lt-General Sanjeev Chachra

Lt-Gen Chachra is to take over as thenew military secretary when Lt-GenG M Nair retires on 31 January 2012.The military secretary's post gainedsome prominence after a previousincumbent Avadhesh Prakash faceda court martial in the infamousSukna land scam case as also dueto the ongoing controversy over

WWW.UPSCPORTAL.COM 79

UPSCPORTAL Current Affairs : http://upscportal.com/civilservices/current-affairs

Integrated Guidance Programme for IAS (Pre) - 2012http://upscportal.com/civilservices/online-course/integrated-free-guidance-programme

Army chief General V K Singh's ac-tual date of birth.

Death

Dev AnandAn icon of Indian cinema and ever-green star Dev Anand died at the ageof 88 in London on 3 December 2011(4 December according to IST) aftersuffering a massive cardiac arrest.Dharam Dev Anand was born on 26September 1923 and is more popu-larly known as Dev Anand. DevAnand was an Indian film actor, di-rector and producer known for hiswork in Hindi cinema. Anand cameBombay in the early 1940s and be-gan his career in the militarycensor's office at Churchgate. Hejoined his older brother, Chetan, asa member of the Indian People'sTheatre Association (IPTA).Film Career: He was offered a star-ring role in Prabhat Films' Hum EkHain (1946) soon after he joinedIPTA. Anand was offered his first bigbreak by Ashok Kumar. He was pickedas the hero for the Bombay Talkiesproduction, Ziddi (1948), co-starringKamini Kaushal. Ziddi went on tobecome a huge success. Anandstarted producing films after thesuccess of Ziddi. he launched hisown company Navketan in 1949which, by 2011, had produced 31films. Dev chose Guru Dutt as direc-tor for the crimethriller, Baazi (1951). His acting

in Kala Pani (1958), as the son whois willing to go to any lengths to clearhis framed father's name won himhis first Filmfare award for Best Ac-tor for the film. He attempted filmsof tragic genre occasionallylike Pocketmaar (1956), Kala Pani(1958), Bombai Ka Baboo (1960) and Sharabi (1964) and tasted suc-cess in them as well. Dev alsoplayed a few characters with a nega-tive shade, like in Jaal(1952). Hisfirst colour film, Guide withWaheeda Rehman was based on thenovel of the same name by R. K.Narayan. Guide, directed by youngerbrother Vijay Anand, was an ac-claimed movie. His directorial debut,the espionage drama Prem Pujari,was a flop. He however tasted suc-cess with his 1971 directorialeffort, Hare Rama HareKrishna which talked about theprevalent hippie culture. Hare RamaHare Krishna that launched ZeenatAman became a cult movie. Dev alsobecame known as a filmmaker oftrenchantly topical themes.Political activity: Dev Anand hadbeen politically active. He led agroup of film personalities whostood up against the Internal Emer-gency imposed by the then PrimeMinister of India, Indira Gandhi. Heactively campaigned against herwith his supporters in Indian parlia-mentary elections in 1977. He alsoformed a party called the NationalParty of India, which he later dis-banded.Awards: The Government of Indiahonoured him with the PadmaBhushan in 2001 and the DadasahebPhalke Award in 2002 for his contri-bution to Indian cinema. He won thefilmfare award thrice- Best actor forkalapani (1958), Best Film for Guide(1966), Filmfare Lifetime Achieve-

ment Award (1991). LifetimeAchievement Award for OutstandingAchievement in Indian Cinema wasbestowed on him at IIFA Award inJohannesburg, South Africa in 2003.In 2004, he was given the Living Leg-end Award by the Federation of In-dian Chamber of Commerce and In-dustry (FICCI) in recognition of hiscontribution to the Indian entertain-ment industry.He received several internationalaward as well: He was honoured byan Award at the hands of the thenFirst Lady of the United States ofAmerica, Hillary Rodham Clinton inJuly 2000. He was also awarded theIndo-American Association Star ofthe Millennium Award. Donna Ferrar,Member New York State Assembly,honoured him with a New York StateAssembly Citation for his Outstand-ing Contribution to the CinematicArts Worthy of the Esteem and Grati-tude of the Great State of New Yorkon 1 May 2001.

He was honoured with a SpecialNational Film Award by the Govern-ment of Nepal at Nepal’s firstNationIndian film festival inStockholm in 2005.

Kim Jong-ilThe enigmatic leader of North Ko-rea Kim Jong-il passed away on 17December 2011 after he suffered amassive heart attack on a train. Thenews of his death was announcedby the state television from theNorth Korean capital, Pyongyang on19 December 2011. An autopsy con-ducted mentioned that the deathwas caused by physical and mentalover-work. Kim Jong-il led his nationfor 17 years and through a devas-tating famine while frustrating theUnited States and other global pow-

WWW.UPSCPORTAL.COM80

UPSCPORTAL Current Affairs : http://upscportal.com/civilservices/current-affairs

Integrated Guidance Programme for IAS (Pre) - 2012http://upscportal.com/civilservices/online-course/integrated-free-guidance-programme

ers with approach to talks on givingup nuclear arms in return for foodand other assistance. Kim took overafter his father and founding Presi-dent Kim Il-sung died in 1994, com-ing to power with a reputation as aplayboy. In the mid- to late-1990s hepresided over a famine which killedhundreds of thousands of his people.Kim Jong-il was born as YuriIrsenovich Kim on 16 February 1941/2. He was the General Secretary ofthe Workers' Party of Korea, the rul-ing party since 1948, Chairman of theNational Defence Commission ofNorth Korea, and the supreme com-mander of the Korean People's Army,the fourth-largest standing army inthe world. In April 2009, NorthKorea's constitution was amendedto refer to him implicitly as the su-preme leader. On 24 December 1991,Kim was also named supreme com-mander of the North Korean armedforces.

Kiro GligorovKiro Gligorov, the first democraticallyelected president of Macedoniadied in Skopje on 1 January 2012. Hehad led his nation through a blood-less secession from the former Yu-goslavia. Gligorov had becamepresident of Macedonia in January1991 when it was still a Yugoslavrepublic. He led his countrymenthrough a referendum in which theyvoted for independence, and the ter-ritory of 2.1 million people becamethe only republic to secede from Yu-

goslavia without a war. Gligorovserved two consecutive presidentialterms, leading the nation from Janu-ary 1991 to November 1999.

S.BangarappaVeteran politician and former ChiefMinister of Karnataka S.Bangarappawho was the Chief Minister ofKarnataka from 1990-92 died on 26December 2011. He was born on 26October 1932 in Kubatur village ofShimoga District, he was a highlyrespected leader among backwardclasses. He represented Shimogaconstituency in the 14th Lok Sabha.S Bangarappa changed parties andconstructed parties but finallylanded in Janata Dal S Party.Bangarappa, who had tested hispolitical fortunes in the BharatiyaJanata Party (BJP) after a long stintin Congress, had recently joined theH D Devegowda-led Janata Dal(Secular). The three time Lok Sabhamember represented Shimoga con-stituency and has been a legislatorseveral times. S Bangarappa was akeen enthusiast in culture of thestate. Bangarappa was in to stageplays, singing and was part of oneor two films in Kannada. SBangarappa was instrumental inholding the first Indian internationalfilm festival in Bengaluru in 1993.

Vaclav HavelFormer Czech president VaclavHavel who had been suffering fromchronic respiratory problems died on18 December 2011 at the age of 75.He was one of the leading anti-Com-munist dissidents of the 1970s and1980s. He was the tenth and lastpresident of Czechoslovakia (1989–92) and the first President of theCzech Republic (1993–2003). Heplayed a seminal role in the Velvet

Revolution that won his people theirfreedom. The peaceful resistancepursued by him exposed the empti-ness of a repressive ideology. Hewas his country's first democrati-cally elected president after the Vel-vet Revolution. The non-violent Vel-vet Revolution ended four decadesof repression by a regime whichHavel ridiculed as Absurdistan.Vaclav Havel, the dissident play-wright wove theatre into politics topeacefully bring down communismin Czechoslovakia and thus becamea hero of the epic struggle thatended the Cold War.

Vaclav Havel was born on 5 October1936 in Prague. His family’s assetswere confiscated by the communistsin 1948. Havel who was denied aformal education had to earn a de-gree at night school.

His political activism began in Janu-ary 1977, when he co-authored thehuman rights manifesto Charter 77,which drew wide attention in theWest. Havel first made a name forhimself after the 1968 Soviet-led in-vasion that crushed the PragueSpring reforms of Alexander Dubcek.He wrote a series of undergroundessays that stand with the work ofSoviet dissident Andrei Sakharov.Havel first came to internationalfame as a dissident playwright in the1970s. The playwright turned politi-cal activist spent four-and-a-halfyears in prison for opposingCzechslovakia's Communist govern-ment before emerging as a leaderof the non-violent Velvet Revolutionin 1989. During his presidency,Czechoslovakia broke up into CzechRepublic and Slovakia in 1993. Hewas credited with laying the ground-work that brought his Czech Repub-

WWW.UPSCPORTAL.COM 81

UPSCPORTAL Current Affairs : http://upscportal.com/civilservices/current-affairs

Integrated Guidance Programme for IAS (Pre) - 2012http://upscportal.com/civilservices/online-course/integrated-free-guidance-programme

lic into the 27-nation EuropeanUnion. Czech Republic became amamber of the European Union in2004. During his regime his countryalso joined NATO in 1999. Havel leftoffice in 2003.

P. K. IyengarThe former Chairman of the AtomicEnergy Commission (AEC), P.K.Iyengar, 80, who passed away inMumbai on Wednesday, was a greatscientist and a role model to manypeople, according to AEC ChairmanSrikumar Banerjee. He excited theminds of youngsters in severalfields. Dr. Iyengar was Chairman,AEC, and Secretary, Department ofAtomic Energy (DAE) from 1990-93.He was Director, Bhabha AtomicResearch Centre (BARC), from 1984to 1990. Prior to this, he headed thePhysics group in BARC. Dr. Iyengar,who belonged to Tirunelveli districtin Tamil Nadu, did hispostgraduation in physics in 1952from the University of Travancoreand obtained his Ph.D. from the Uni-versity of Bombay in 1963. He joinedthe Tata Institute of FundamentalResearch (TIFR) of the DAE in 1952.He was deputed to Chalk River Labo-ratories of the Canadian Atomic En-ergy Establishment. He was trainedin Canada under Dr. B.N.Brockhouse, a Nobel Laureate inPhysics.After he returned to India, Dr. Iyengarpioneered research in neutron phys-

ics and built an internationallyrecognised team in that field in theBARC. He led a team that indig-enously designed and developed thePURNIMA reactor, which was com-missioned in 1972. He made a sig-nificant contribution to the indig-enous building of the Dhruva reac-tor, which attained criticality whenhe was Director, BARC. Dhruva con-tinues to be a world-class facilityeven today, and Dr. Iyengar's contri-bution during the final stages of itscommissioning was significant.Starting his research in nuclear phys-ics in the TIFR in 1952, Dr. Iyengarforesaw the advent of nuclear reac-tors and the opportunities they of-fered in both basic research andnuclear technology. He startedbuilding neutron spectrometers in1958 around Apsara, Asia's firstnuclear reactor, even before the highflux reactor CIRUS was operational,and gave India an early start in neu-tron beam research. He encouragedresearch and development in mate-rial science, radiation physics, mo-lecular biology, lasers and accelera-tors. He was primarily responsiblefor the establishment of the RajaRamanna Centre for Advanced Tech-nology in Indore.

As AEC Chairman, Dr. Iyengar vigor-ously pursued the nuclear powerprogramme with the commissioningof reactors at Narora in UttarPradesh and Kakrapara in Gujarat.He emphasised the importance ofthe development of the Fast BreederTest Reactor at the After his retire-ment from the DAE, Dr. Iyengar heldseveral positions. He received sev-eral awards, including ShantiSwarup Bhatnagar award andPadma Bhushan. He is survived byhis wife, a son and a daughter.

Amichand RajbansiCharismatic and controversial SouthAfrican Indian politician AmichandRajbansi, 69, died in a hospital hereon Thursday. He was admitted twomonths ago for a bronchial condi-tion. He died just a fortnight shortof his 70th birthday. Nicknamed“The Bengal Tiger” because of hisfiery nature, Rajbansi courted politi-cal controversy throughout his ca-reer but remained popular with asection of the Indian community,which repeatedly voted him backinto positions through his MinorityFront party. Rajbansi and his wife,pharmacist Shameen Thakur, werethe only two representatives of theparty in the KwaZulu-Natal provin-cial legislature. Rajbansi claimed tohave honed his political skillsthrough working with leaders of theAfrican National Congress (ANC)such as Albert Luthuli, MosesMabhida and Nelson Mandela. InJanuary 2009, Rajbansi received aLifetime Achievement Award fromthe India International FriendshipSociety in New Delhi.

Ram Nath SinghPoet Ram Nath Singh, popularlyknown as Adam Gondvi, breathed hislast in Lucknow on 18 December2011. Gondvi lived like a villagerthroughout his life and wrote revo-lutionary poetry focusing on the pa-thetic state of dalits and the poor.He himself died in poor financialcondition and cultural and socialactivists had to appeal for donationsfor his treatment. Adam Gondvi wasborn on 22 October 1947 in AataaParaspur village of Gonda district inUttar Pradesh. His couplet, Kaajubhuni plate mein, whisky bharigilaas mein, utra hai raamraajvidhayak niwas mein was a great hit

WWW.UPSCPORTAL.COM82

UPSCPORTAL Current Affairs : http://upscportal.com/civilservices/current-affairs

Integrated Guidance Programme for IAS (Pre) - 2012http://upscportal.com/civilservices/online-course/integrated-free-guidance-programme

among the masses. Gondvi alsocriticised politicians who soughtvotes on caste and communal basisin his gazal collections. His collec-tion Dharti ki satah par and Samayse Muthbhed made him an ac-claimed gazal writer. He was felici-tated with Dushyant Kumar prize in1998 by Madhya Pradesh govern-ment.

Socrates Brasileiro de SouzaViera de Oliveira

Brazil's former football captain,Socrates who led Brazil in two WorldCups died of septic shock on 4 De-cember 2011 in Sao Paulo. Socrateswas also known as the Golden Heeand the Big Thin One. He was re-nowned as one of the greatplaymakers of his generation. Bornon 19 February 1954, Socrates waspart of a golden Brazilian generationwho included midfielder Zico, Jun-ior, Falcao and Eder.

Socrates Brasileiro de Souza Viera deOliveira made his international de-but in 1979 and captained Brazil atthe 1982 and 1986 World Cups. How-ever Brazil did not manage to winthe cup under the captaincy of theformer attacking midfielder. Socrateswon 60 caps and scored 21 goals inhis seven-year Brazil career. He hadscored 25 goals in 63 games for Bra-zil. Socrates was a also a culturalicon and a political activist who usedhis celebrity to fight for the end ofBrazil's 1964-1984 military govern-ment. Socrates was a physician andhad practiced medicine after retir-ing as a player in 1989.

Jacob E. GoldmanJacob E. Goldman, a founder of thePalo Alto Research Center (PARC)that developed breakthrough com-puting innovations died on 20 De-cember 2011. The Palo Alto Re-search Center developed computinginnovations such as the graphicaluser interface and ethernet net-works. Physicist Jacob E. Goldman,as Xerox's chief scientist foundedthe company's vaunted Palo AltoResearch Center, which invented themodern personal computer. In 1970s,the laboratory created a string ofinnovations from laser printing toobject-oriented programming to theworld's first WYSIWYG (What-You-See-Is-What-You-Get) editor.Goldman played an important roleboth at the Ford Motor Co., duringthe 1950s, and later at Xerox in the1960s and 1970s, in financing basicscientific research in an effort tospark corporate innovation. He be-came a private investor after retire-ment, according to the book Deal-ers of Lightning: Xerox PARC and theDawn of the Computer Age byMichael Hiltzik.

Mario MirandaThe world renowned cartoonist,Mario Miranda died in Goa on 11December 2011. He was 85. He fa-mously captured vignettes of Goanlife on canvas in his trademark stylefor over two decades. Miranda wasthe recipient of the Padma Bhushan

(2002) and Padma Shri (1988)awards. He started his career as acartoonist for the Times of IndiaGroup in 1953 and later moved intoillustration and fine art. His cre-ations such as Miss Nimbupani andMiss Fonseca which appeared on aregular basis in Femina, EconomicTimes, and The Illustrated Weeklyof India brought him fame. In his stintwith the Times of India, Mirandaworked with the noted cartoonist,R.K. Laxman. Miranda’s own booksare Goa with Love, A little World ofHumor, Sketch book, Germany inWintertime, Impression of Paris andMario de Miranda.

Vasant RanjaneFormer Indian Test (medium-pace)cricketer Vasant Ranjane passedaway in Pune on 22 December2011. Born in Pune on 22 July 1937,Ranjane grabbed 19 wickets in Testsat 34.15 per innings and 175 wick-ets from 64 first class ties. Ranjanethus made a successful first-classdebut in 1956-57, when he took 9for 35 and 4 for 36 for Maharashtraagainst Saurashtra. Ranjane playedseven Tests for India between 1958and 1964 and representedMaharashtra and Railways in first-class cricket from 1956 to 1971. Hemade his Test debut against WestIndies in 1958-59. Next he was cho-sen for his second game in1961-62,when England toured India. Heplayed two more Tests in that seriesand was taken on the 1962 tour ofWest Indies, where he played onegame, the final Test at Kingston. Heplayed two more Tests, against En-gland in Madras in 1963-64 andagainst Australia at the same venuea year later and finished with 19 Testwickets at an average of 34. Ranjaneworked for Indian Railways as a fit-

WWW.UPSCPORTAL.COM 83

UPSCPORTAL Current Affairs : http://upscportal.com/civilservices/current-affairs

Integrated Guidance Programme for IAS (Pre) - 2012http://upscportal.com/civilservices/online-course/integrated-free-guidance-programme

ter and retired in 1994.

George WhitmanGeorge Whitman, the American-bornowner of Shakespeare & Company,a fabled English-language bookstoreon the Left Bank in Paris died in Parison 14 December 2011. His bookstoreserved as a magnet for writers , po-ets and tourists for close to 60 years.Born in East Orange, New Jersey,United States he was awarded theOfficier des Arts et Lettres medal bythe French government for his con-tribution to the arts over the previ-ous fifty years.

Hamilton BobbyFormer Indian footballer HamiltonBobby passed away following a car-diac arreston 17 December 2011. Bobby, an assistant manager of In-dian Bank who fell unconscious athis place of work could not be savedthough he was immediately rushedto the hospital. He had twicecaptained Tamil Nadu in the SantoshTrophy and also represented thestate for seven years from 1993 to2000. The footballer had also playedin the under-16 and under-23 na-tional teams and was a member ofthe senior India national team atPresident's Cup in Colombo.

Satyadev DubeyTheatre director, screenplay writerand actor Satyadev Dubey, creditedwith introducing existential and ab-surd theatre, passed away on 25

December 2011. Dubey was a mav-erick who went on to become a the-atre guru. Born in Bilaspur,Chhattisgarh, in 1936, Dubey's firstambition was to become a cricketer.However after coming to Mumbai,he gave up the idea of becoming acricketer. Dubey came into promi-nence with Dharamvir Bharati’s ra-dio-play Andha Yug that brought tothe fore the pervasive criminal andhomicidal tendencies during thetimes of war. He penned down thescreenplay for Shyam Benegal’s filmBhumika as well. He penned screen-plays/dialogue of some acclaimedfilms in the 1970s, including ShyamBenegal's Nishant, Ankur, Kalyugand Bhumika, and also acquired adistinct identity as theatre director-teacher. He won the Filmfare Awardfor Best Dialogue in 1980 for the filmJunoon. He also won the Silver Lo-tus Award in 1978 for Best Screen-play for the fim Bhimika. He sharedthis award with Shyam Benegal andGirish Karnad

KV VaradarajKV Varadaraj, one of the few Indianfootballers who played in two Olym-pics in 1948 and 1952, passed awayin Bangalore on 20 December 2011.Varadaraj was nicknamed 6-footerby the English. Born on 7 May 1924,Varadaraj came up quickly throughthe ranks representing ChallengeUnion Mysore, Mysore Railways andBangalore Blues.

Varadaraj was the goalkeeper of theIndian team led by Mohun Bagandefender Talimeren Ao that took thefield in the 1948 London Games.Despite Varadaraj also known asMysore Wall guarding the goal, In-dia lost to France 1-2 in the firstround. Varadaraj won the gold medal

at the 1951 inaugural Asian Gamesin New Delhi as he helped India keepa clean slate in all the matches,beating Iran 1-0 in the final. Laterhe also worked a coach for theHMT. Varadaraj was one among the sev-eral Olympians to have emergedfrom Karnataka, then known asMysore. Among his illustrious con-temporaries were T Shanmugam, BNVajravelu, Ahmed Khan, SA Basheerand S Raman.

Accused/Resigned

Jacques ChiracA French court sentenced formerFrench President Jacques Chirac toa two-year suspended jail term fol-lowing his conviction on graftcharges. He was held guilty for mis-appropriation of public funds andembezzlement. This is the first timea French Head of State has beenconvicted since the trial in 1945 ofMarshal Philippe Petain who col-laborated with the Nazis. He is thefirst former or serving president ofmodern France to be tried, althoughNazi-era collaborationist leaderPhilippe Petain was convicted oftreason and king Louis XVI was sentto the guillotine by revolutionariesin 1793. Chirac who suffers fromneurological problems and is in poorhealth was not present in the court-room. served two consecutive termsas President from 1995 to 2007.Chirac's crimes date back to the1990s when, as Mayor of Paris, heplaced several of his party men andwomen on the public payroll in whathas been described as phantom jobs.Chirac's criminal conduct had costParis taxpayers the equivalent of 1.4million euros ($1.8 million). Hebreached the duty of trust that

WWW.UPSCPORTAL.COM84

UPSCPORTAL Current Affairs : http://upscportal.com/civilservices/current-affairs

Integrated Guidance Programme for IAS (Pre) - 2012http://upscportal.com/civilservices/online-course/integrated-free-guidance-programme

weighs on public officials chargedwith caring for public funds or prop-erty, in contempt of the general in-terest of Parisians.

Ogyen Trinley Dorje The 17th Karmapa Ogyen TrinleyDorje was named in a chargesheetfiled by the police in a HimachalPradesh court on the seizure of for-eign currency worth Rs.70 millionfrom a monastery in January.

Ilker BasbugTurkey's former Army chief IlkerBasbug was arrested on 6 January2012 over an alleged attempt on partof the chief to topple the Islamist-rooted government in the country.Ilker Basbug, the 26th chief of staffof the Turkish republic was placedin preventive detention for settingup and leading a terrorist group andof attempting to overthrow the gov-ernment. General Basbug, whoserved as Army chief from 2008 to2010, was sent to a prison atIstanbul's Silivri prison. Basbug is thefirst such high-ranking military com-mander to be arrested as a suspectsince a former chief of staff in the1960s.

Rod BlagojevichFormer Illinois Governor RodBlagojevich was sentenced to 14years in federal prison on 7 Decem-ber 2011. He was arrested andcharged with a wide-ranging corrup-tion spree that included trying to

auction off President BarackObama's vacated Senate seat.Blagojevich, a onetime rising star inthe Democratic Party whose publicdefense strategy included stints onreality TV, a media tour and a book.He was convicted over the courseof two trials on 18 counts of corrup-tion. His crimes included lying to theFBI, attempting to shake down achildren's hospital for campaign con-tributions and seeking to securecampaign funds, a cabinet post oran endowment for a new foundationin exchange for the Senate seat va-cated by Obama. Under federalguidelines, he is to serve 85% of hissentence, or around 12 years.

Honour

‘Time’ Person of the YearThe Protester was named Timemagazine’s 2011 Person of the Yearon 14 December 2011. Time definesthe Person of the Year as someonewho, for better or for worse,influenthat individual action canbring collective, colossal change.2011 witnessed unprecedented risein both peaceful and sometimes vio-lent unrest and dissent from the ArabSpring to the Occupy Wall Streetmovement. People across the globein Tunisia, in Egypt, in Libya dis-sented and demanded even whenthey were answered with tear gasor a hail of bullets. The Mideast wasthe centerpiece of the year’s pro-tests.

US Special Operations Commandand overall commander of the secretUS mission into Pakistan in May2011 that killed Osama bin Laden,came in at second place on the Timelist. Chinese dissident artist AiWeiwei came in at No 3. Rep. PaulRyan, who put forth a plan to tackleAmerica’s burgeoning national debtstoof fourth. Britain’s Duchess ofCambridge, Kate Middleton, whomarried Prince William finshed fifth.In 2010, Facebook founder and CEOMark Zuckerberg got wa named thePerson of the Year.Time Magazine mentioned that Turk-ish Prime Minister Recep TayyipErdogan was the People's Choice forthe Person of the Year. In an onlinepoll, Erdogan received 123000 votesnominating him as the most influ-ential person of the year significantlymore than any other nominee.

Hema MaliniVeteran Bollywood actress HemaMalini was named the person of theyear by People for the Ethical Treat-ment of Animals (PETA). PETA namedher the Person of the Year for rais-ing her voice to oppose cruelty toanimals and her staunch commit-ment to a vegetarian diet. TheDream Girl on behalf of PETA hadrecently appealed to the minister ofEnvironment and Forests to take im-mediate steps to end the cruel tra-dition of jallikattu. Jallikattu is asport event in Tamil Nadu in whichterrified bulls are kicked, punched,

WWW.UPSCPORTAL.COM 85

UPSCPORTAL Current Affairs : http://upscportal.com/civilservices/current-affairs

Integrated Guidance Programme for IAS (Pre) - 2012http://upscportal.com/civilservices/online-course/integrated-free-guidance-programme

jumped on, dragged to the groundand otherwise tormented. In re-sponse to her plea, the Ministry is-sued a notification in The Gazetteof India stating that bulls could nolonger be used as performing ani-mals.

UdaipurReaders of international travelmagazine Condé Nast Travellervoted the lake city of Udaipur inRajasthan as the best leisure desti-nation in India. Rajasthan State Tour-ism Minister Bina Kak received theCondé Nast Traveller India Award-2011 from Union Tourism MinisterSubodh Kant Sahay in New Delhi on3-4 December 2011 weekend. Theaward opened up Rajasthan’s tour-ism sector to the globetrotters andpackage tour operators in a big way.Condé Nast Traveller is one of theworld's most renowned travel maga-zines. Its readers vote for variouscategories of destinations, accom-modations and transport every year.

Alastair CookEngland batsman Alastair Cook on6 December 2011 received Memberof the Order of the British Empire(MBE) medal from Britain's QueenElizabeth II at Buckingham Palace.The left-handed opener's prolificform in Australia during the 2010/11 Ashes series helped England toa 3-1 series win - their first Test cam-paign triumph Down Under in 24years. Cook scored a mammoth 766runs in the series to play a major partin that series win. Then series win

helped propel England to the top ofthe Test rankings. Cook began hisTest career with a second-inningscentury in Nagpur, India, in March2006. He has till date made 19 Testtons in all. In the second innings ofthe first Ashes Test in November2010, Cook made 235 not out. Hethen scored 148 in the second Test,82 in the fourth and 189 in the fifthand final Ashes encounter in Sydney.

Dam999Controversial film, Dam999 directedby Sohan Roy made it to a shortlistfor the Oscars under the Best Pic-ture category. The film, based on acatastrophe caused by a 100-year-old dam in India giving way, is amongthe 265 films in contention for theBest Picture. Tamil Nadu governmentimposed a ban citing that it maycause animosity between Kerala andTN people as the film was dealingwith the subject of the 116-year-oldMullaperiyar dam on the Kerala-TNborder. The film also fulfilled othercriteria of being exhibited theatri-cally on 35 mm, 70 mm or in digitalformat and having a running time ofover 40 minutes. Three of the songsfrom Dam999, The film`s songs -Dam 999 (Theme song ), DakkangaDugu Dugu and Mujhe Chodke wereamong the 39 songs shortlisted un-der the Best Original Song category.All three songs were written by Royhimself, and set to tune byOusephachan. National award win-ner Hariharan lent his voice to theHindi song Mujhe Chod Ke.

Under 30 AchieversThe Forbes' 30 under 30 list was pre-pared by selecting young turks from12 diverse fields including energy,finance, media, law, entertainment,science, design and technology.

Facebook's Mark Zuckerberg, humanrights activist Ronan Farrow and popstars Lady Gaga and Justin Bieberare also included in Forbesmagazine's list of top-achievingpeople under age 30. The list re-leased in December 2011 included10 people of Indian origin.

Among the Indian-origin people onthe list is 17 year old Param Jaggi, astudent and inventor at Austin Col-lege. Jaggi created algae-filled de-vice that fits over a car's tailpipe andturns carbon dioxide into oxygen.Vivek Nair, Chief Executive of Dam-ascus Fortune, who is developing atechnology that transforms indus-trial carbon emissions into carbonnanotubes was also named in thelist. Manvir Nijhar, Co-Head of Eu-ropean Equity Derivatives Sales atCitigroup was named as well. KunalShah, the youngest Managing Direc-tor at Goldman Sachs was also in-cluded in the list. Making a mark inthe field of science was Raj Krishnan,Chief Executive of Biological Dynam-ics who is developing blood teststhat use electric fields to detect keysignals that a patient has cancerfrom the blood. Sidhant Gupta, agraduate student at the Universityof Washington who has been listedas well, is developing new sensorsand software for the home that con-serve electricity, heat and gas.

Accidents

Sukhoi Su-30 MKI Fighter Air-craft Crashed

A Sukhoi Su-30 MKI fighter aircraftfrom the Lohegaon Air Force Stationcrashed on 13 December 2011 at aplace near Wadebholai, 25 km awayfrom the Pune city. The accident wasexplained as a mechanical failure.

WWW.UPSCPORTAL.COM86

UPSCPORTAL Current Affairs : http://upscportal.com/civilservices/current-affairs

Integrated Guidance Programme for IAS (Pre) - 2012http://upscportal.com/civilservices/online-course/integrated-free-guidance-programme

The plane lost contact with itsbase's Air Traffic Control. A Court ofInquiry was ordered into the inci-dent. However, no casualties tookplace due to the accident. It was aroutine training sortie. The Sukhoibelonged to the No 20 'Lightening'squadron and was the 8th IAF planeto crash in 2011. The IAF lost 30fighter aircraft's, of which 10 werelost from 2009 to 2011.

Commity/Commission/Conference

Conference on AutismNon-government organisation Asso-ciation for Behaviour Analysis (ABA-India) organised the 2nd ABA-Indiaconference Autism: From Early Child-hood to Adulthood Successful DataBased Intervention at India HabitatCentre, New Delhi on 17 and 18 De-cember 2011. Association forBehaviour Analysis is working onincreasing the limited resource poolof behaviour analysts and educatingparents of children with autism.Theconference is focussed on the childwith autism and successful evidencebased interventions for these chil-dren. Besides the keynote speakersand presentations, papers were alsobe presented by parents teachingtheir children using ABA principles.The conference offered a first oneof a kind learning opportunity.ABA-India is an affiliate chapter of ABAInternational, U.S., and is pioneer-ing the introduction of ABA in India.ABA is a subject dealing with thescientific study of human behaviourand its applications in various fields,including mental health, develop-mental disabilities, child develop-ment and education. ABA methodsare used to shape behaviour forbringing positive changes in society.

The Bonn ConferenceThe Global Conference onAfghanistan’s future concluded here.Bonn is the capital of Germany. Theone-day global conference in Bonnon Afghanistan’s future ended on 5December 2011. During the summit,Afghanistan pledged to intensify thedrive against corruption in exchangefor international support after NATOforces withdraw in 2014. About1000 delegates from 100 countriesand international organisations par-ticipated in the organisation. Paki-stan didn’t attend the Bonn summit.F irst Bonn summit was held in2001.

Meeting on Citizen Engagementthrough Social Media

Communications and IT MinisterKapil Sibal on 15 December 2011sought help from social networkingwebsites and Internet companies inpropagating government's variouse-governance initiatives. The objec-tive is to make people more awareof the online environment. Sibalchaired a meeting Citizen engage-ment and use of social media at-tended by functionaries of all lead-ing Internet companies includingGoogle, Facebook and Twitter be-sides other stakeholders. The meet-ing was organised on 15 Decemberto discuss how the social mediacould empower the government,since under the normal processes ofgovernance only limited dialoguewith the representatives of societyis possible. The Internet companieswere asked to give their opinions onhow to use social media and e-gov-ernance to empower citizens. Theobjective of using the internet toimprove citizen engagement inpolicy formulation and implementa-tion of its projects and programmeswas highlighted.

Committee to increase theProductivity of Agriculture

SectorPresident Pratibha Devisingh Patilconstituted a 12-member committeeto study and recommend measuresfor increasing productivity, profitabil-ity and sustainability of the agricul-ture sector in India. The committeewill work under the Chairmanship ofGovernor of Punjab and Rajasthan,Shivraj Patil. The committee will de-liberate on the need for formulatingrequisite policy initiatives and leg-islative measures and prospects forinvestment in the agriculture andallied sectors among others. TheCommittee will also discuss on mea-sures for transfer of innovations andcutting- edge technologies to thefarm and allied sectors.

Various

People’s National Party won inJamaica

Jamaica's main opposition party haswon the country's parliamentaryelections, while the ruling JamaicaLabour Party concededdefeat.Jamaica is an island nation.It is situated in the Caribbean Sea.Preliminary results showed that with50 percent of the votes counted, theopposition People's National Partyis expected to win 42 out of the to-tal 63 parliamentary seats.

ECU to deal with Black Moneyissue in IBSA Countries

The Indian government in December2011 set up an electronic centre unitin the Finance Ministry to deal withthe unaccounted money issue inIBSA countries- India, Brazil andSouth Africa. The new unit is to dealwith governments in exchanging in-formation on tax-evaders and black

WWW.UPSCPORTAL.COM 87

UPSCPORTAL Current Affairs : http://upscportal.com/civilservices/current-affairs

Integrated Guidance Programme for IAS (Pre) - 2012http://upscportal.com/civilservices/online-course/integrated-free-guidance-programme

money in IBSA nations. The govern-ment received information from 10countries on the issue. The IBSAelectronic centre unit is aimed attackling the black money menace inthe IBSA nations. The information iscurrently being disseminated by vari-ous government agencies. Prosecu-tion has also been launched in 17cases pertaining to LTG bank in Ger-many. The unit, in close associationwith the revenue intelligence agen-cies of other countries, will identifyand share information on tax evad-ers, abusive tax schemes as well asof the people who are involved inpromotion of such activities. Abu-sive tax schemes are transactionsdesigned to reduce or defer taxation.India has Double Taxation AvoidanceAgreements (DTAAs) is currently inplace with Brazil and South Africa.Representatives of all the countriesare to have an exchange of informa-tion and hold regular meetings in thethree countries by rotation to workout a joint strategy on how to tacklethis growing menace of stashingfunds abroad and prevent ways oftax evasion.

EMB-145I AircraftThe first fully modified aircraft, EMB-145I for the indigenously developedIndian Airborne Early Warning andControl System (AEW&C) undertookits maiden flight from Sao Jose dosCampos in Brazil on 6 December2011. The modified Embraer aircraftundertook its maiden flight withabout 1000 Mission System Compo-nents provided by the Centre for Air-borne Systems (CABS) of the De-fence Research DevelopmentOrganisation (DRDO). The compo-nents include the critical item – AESA(Active Electronic Scanning Antenna)Radar Antenna developed by DRDO

and certified from ANAC, Interna-tional FAR Certification Agency. Theindigenous system, AEW&C esti-mated to cost around Rs 1800 crorewould detect, identify and classifythreats present in the surveillancearea and act as a command and con-trol centre to support air operations,according to DRDO officials. Thesystems for the EMB-145I aircraft iscurrently being developed by Ban-galore-based Centre for Air BorneSystems (CABS), a Defence Re-search and DevelopmentOrganisation (DRDO) facility. Theinclusion of this aircraft will enableIndia to join the league of countriescapable of developing and deliver-ing such complex Airborne Systemof Systems to its user.

World's First Test Tube Eld's DeerThe first Eld’s deer was born via invitro fertilization in Thailand. Theresearchers at the Smithsonian Con-servation Biology Institute (SCBI)collected eggs, inseminated in vitrowith thawed semen to produce em-bryos and transferred the embryosto a surrogate mother. As a result, afawn was born on 17 October 2011at the Khao Kheow Open Zoo inThailand. The Smithsonian Conser-vation Biology Institute plays a keyrole in the Smithsonian’s global ef-forts to conserve species and trainfuture generations of conservation-ists. Eld's Deer is also known as theThamin or Brow-antlered Deer. It isan endangered species of deer in-digenous to southeastern Asia.

SibsagarThe country's first cooperative uni-versity will be set up at Sibsagar inAssam. To be known as the RajivGandhi University of CooperativeManagement, it will offer courses on

cooperative management and trainexisting manpower in both the gov-ernment and the private sectors, tomanage the cooperative sector.

Deadline for Social NetworkingA Delhi court set 6 February as adeadline for 22 social networkingsites including the Facebook, theGoogle, the Yahoo and the Microsoftto remove all anti-religious or anti-social content.The court directed thesocial networking sites to file com-pliance reports by 6 February 2012.Additional Civil Judge Mukesh Kumarhad on 20 December in a ex-parteorder issued summons to the socialnetworking websites, grantedaround one and half month for com-pliance of his order. The court aftergoing through the several printoutsof the objectionable contents, CDand other documents had foundthem defamatory and derogatoryand ordered to take them off fromthe sites. The social networking siteswere issued summons by the court,on grounds of carrying objectionablecontent. The companies werecharged with section 292 (sale ofobscene books etc), 293 (sale ofobscene objects to young personetc) and 120-B (criminal conspiracy)of the IPC.

AhmedabadIn an exclusive opinion poll con-ducted for The Times of India,Ahmedabad emerged as the bestamong India’s mega-cities, edgingout Pune, Mumbai and Delhi. Thisfinding was done by an exclusiveopinion poll conducted for TheTimes of India by leading marketresearch firm, IMRB. The survey wason quality of life in India. Ahmedabadbagged the national award on 13December 2011, in the category of

WWW.UPSCPORTAL.COM88

UPSCPORTAL Current Affairs : http://upscportal.com/civilservices/current-affairs

Integrated Guidance Programme for IAS (Pre) - 2012http://upscportal.com/civilservices/online-course/integrated-free-guidance-programme

Best City in the Implementation ofBasic Services to Urban Poor (BSUP)under the Jawaharlal Nehru NationalUrban Renewal Mission (JNNURM)scheme.

Indira BhavanThe West Bengal Government on 28December 2011 declared its decisionto take over Indira Bhavan, whichwas the official residence of formerChief Minister Jyoti Basu at Salt Lake.West Bengal CM Mamata Banerjeedeclared that Indira Bhavan wouldbe turned into a museum and a re-search centre devoted to Kavi NazrulIslam, a contemporary ofRabindranath Tagore. After thedeath of Jyoti Basu in 2010, the pre-vious government (CPIM) had mulledmany options including converting itinto a government guest house. Thedecision to convert Indira Bhavaninto Nazrul Bhavan was taken in pub-lic interest and to promote researchwork on the great rebel poet. TheNazrul Bhavan would house a re-search centre and museum on thegreat rebel poet who made a memo-rable contribution to the country'sfreedom movement. The buildingwill house primarily the poet'smemorabilia, his books and re-search on the poet's works. The bun-galow was named Indira Bhavanduring the Congress Session in 1972when the then Prime Minister IndiraGandhi stayed there for the durationof the session. Indira Bhavan wasbuilt in 1972 primarily to serve as aguest house for then Prime MinisterIndira Gandhi for her visits to the city.

The Left Front government later de-cided to allot Indira Bhavan for Basuto live in and he had moved into thebungalow in August 1989. Sincethen, he continued to live there tillhis death in 2010.

Malappuram (Kerala) Off-campusof the AMU

Union Minister for Human ResourceDevelopment Kapil Sibal dedicatedthe Malappuram (Kerala) off-campusof the Aligarh Muslim University(AMU) to the nation at a functionheld in Perinthalmanna on 24 De-cember 2011. The Aligarh Muslim University spe-cial centre in Malappuram is one ofthe five such centres being set up inthe country. The new campus willhouse five hostels for men and threewomen hostels. The constructionalso includes an administrativeblock, two academic blocks and 20-odd staff quarters. The Union gov-ernment sanctioned Rs 200cr for theAMU campus which is expected tobe ready in five years. It will havefacilities from pre-school to a full-fledged medical college. The stategovernment will spend Rs 100cr onthe development of the campus.

India 3rd Amongst the LargestPurchasers of US Arms

According to the Defence SecurityCooperation Agency (DSCA) report,India emerged as the third largestpurchaser of US arms with contractsworth $4.5 billion in the year ended30 September 2011 as US weaponssales rose to $34.8 billion in fiscal2011. Defence Security CooperationAgency is the Pentagon agency thatoversees foreign arms sales. Eventhough the US firms lost the bid foran estimated USD 10 billion contractfor sale of 126 jets to Indian Air Force

(IAF), India was listed as the thirdlargest purchaser of US armsthrough the government to govern-ment channel Foreign Military Sales(FMS). American arms sales to Indiainclude Naval Reconnaissanceplanes, giant transporters, sea-skimming missiles and otherweapon systems. According to thereport government-to-governmentforeign military sales totalled $28.3billion in fiscal 2011, while othersales managed by various govern-ment agencies reached $6.5 billion.The Pentagon had projected govern-ment-to-government sales wouldreach $36.3 billion for the year end-ing 30 September 2011. Topping thelist for the fiscal year 2011 ending30 September 2011 were- the Af-ghan Security Forces ($5.4 billion);Taiwan (US $4.9 billion); India ($4.5billion); Australia ($3.9 billion); SaudiArabia ($3.5 billion); Iraq ($2.0 bil-lion); the United Arab Emirates ($1.5billion); Israel ($1.4 billion); Japan($0.5 billion); and Sweden ($0.5 bil-lion).

Green Film FestivalGreen Film Festival was held in Delhifrom 6 December to 10 December2011. Leading environment and wild-life films from across the world werescreened as part of the CMSVatavaran film festival. BiodiversityConservation was the theme of thefestival. A total of 114 films werescreened, taking viewers into thediverse geographical realms ofplanet Earth through the lens of thefinest environment and wildlife film-makers. It was a summit for Asianfilm-makers, special programmes forchildren throughout the event anddiscussions on the environment withmore than 50 speakers and 300 del-egates from 15 different countries.

WWW.UPSCPORTAL.COM 89

UPSCPORTAL Current Affairs : http://upscportal.com/civilservices/current-affairs

Integrated Guidance Programme for IAS (Pre) - 2012http://upscportal.com/civilservices/online-course/integrated-free-guidance-programme

It held an award ceremony for youngenvironment journalists and film-makers also.

Six Agrochemical MultinationalCorporations

A panel of jurors of the PermanentPeople's Tribunal, an internationalopinion tribunal, presented its find-ings following three days of deposi-tions by citizens from around theworld. The findings of the panel in-dicted parent states of six agro-chemical multinational corporations,host countries such as India, andinternational bodies (such as theUnited National, Food and Agricul-tural Organisation and the WorldTrade Organisation). The panel high-lighted that the failure of states toregulate, monitor and discipline theactivities of the companies madeculpable along with the host states.Witnesses from around the worldhad testified in the four-day Perma-nent People's Tribunal regarding thehealth impact, human rights viola-tions and unethical practices by sixtransnational corporations — Bayer,BASF, Dupont, Monsanto, Syngentaand Dow Chemical.

German economist Elmar Altvaterpresented the facets of the findingsand also mentioned on the occassionthat economic liberalisation, deregu-lation and privatisation of formerlypublic goods have led to humanrights violations and unethical prac-tices by six transnational corpora-tions. The conclusion drawn main-tained that transnational have notbeen responsible in their pursuit ofhigher profits, and for this peoplehave had to pay a hefty price.

Books

Nahjul BalaghaIran's former Foreign Minister AliAkbar Wilayati on 21 December2011 released a 1000-year-oldmanuscript, Nahjul Balagha believedto be a collection of letters,speeches and aphorisms of thefourth Caliphate of Islam Hazrat Aliat Aligarh Muslim University. Themanuscript was released for publi-cation at Aligarh Muslim University.The Urdu, Hindi and English versionsof this rare manuscript is to be pub-lished by the AMU with financialsupport of the Iran Government.Bahr-e-Zakhkhar, another book be-lieved to contain a rare collection ofTazkira (a sort of biography mostlyin the form of poetry) in Persian wasalso released. AMU's Maulana AzadLibrary was also gifted 3000 Persianmanuscripts and CDs by the IranCultural House.

Izhar-e-KhayalVice President and Rajya SabhaChairman M Hamid Ansari releasedtwo booksExpression ofThoughts (English) and Izhar-e-Khayal (Urdu) authored by K.RahmanKhan, Deputy Chairman of RajyaSabha in New Delhi on 19 Decem-ber 2011. The books in English andUrdu are compilation of his selectedspeeches on various subjects likeParliament, democracy and gover-nance, economy and business, be-sides issues of equity, justice andsecularism, women empowermentand development. The books - Ex-pression of Thoughts and Izhar-e-Khayal were released in the pres-ence of Lok Sabha Speaker MeiraKumar and Leader of Opposition inRajya Sabha Arun Jaitley. Former LokSabha Speaker Somnath Chatterjee

wrote the foreword to the Englishversion of the book, while HazratMaulana Rabe Hasan Nadwi Sahabhas written for the Urdu version. Onthe occasion,the Vice Presidenthighlighted Khan’s seminal contribu-tion to the field of education andminority uplifting especially matterspertaining to Wakf. The Wakf Com-mittee Report presented under hisChairmanship was a remarkablepiece of diligent work.

Non-Stop IndiaThe book Non-Stop India by journal-ist-author Mark Tully was launchedin Chennai by Editor-in-Chief of TheHindu, N Ram on 13 December 2011.The book was published jointly byPenguin Books India and MadrasBook Club. Tully, who also authoredNo Full Stops in India mentioned onthe occasion of the launch that hehad extensively covered India overthe last 20 years. The book talked ofthe visit of the writer who has trav-elled across the length and breadthof the counrty which include visit-ing Maoists-hit areas, problemsfaced by Dalits. The book had alsomentioned about the growth of TataGroup and its Chairman Ratan Tata.Mark Tully cautioned in his book thatexcessive secularism was a dangerto a country like India and added thatboth secularism and communalismwere used as political weapons.The Official Journey to Makkah OpusA book titled The Official Journey to

WWW.UPSCPORTAL.COM90

UPSCPORTAL Current Affairs : http://upscportal.com/civilservices/current-affairs

Integrated Guidance Programme for IAS (Pre) - 2012http://upscportal.com/civilservices/online-course/integrated-free-guidance-programme

Makkah Opus was unveiled on 14 December 2011 by Prince Turki Bin Faisal Al Saud of Saudi Arab at pre-launchfunction in New Delhi. The book is scheduled to be formally launched in 2012. Prince Turki Bin Faisal Al Saud isa member of the Saudi royal family and founder of the King Faisal Centre for Research and Islamic Studies, whichhas provided active support and advice for the publication of the Makkah Opus. The book is bound by hand in thefinest leather and contained in a silk covered clamshell case.

The book is being published by the Opus Media Group, luxury publisher of premium large format books. The Indiaedition, priced at a whopiing $40000-50000, would be limited to 100 copies and will be exhibited in differentcities of the country in March 2012.

The book has been described as a compendium of finest scholarly writing, and both historical and speciallycommissioned art and photography. It is believed to faithfully represent the incredible journey of Hajj pilgrimageworld-over. The book is a limited edition volume which contains 100000 words both in Arabic and English withthe support of 2000 photographs, most of which have been specially commissioned. A striking feature of thebook is the two meters long gatefold pictures. The book has also been compiled with a view to make it aneducational tool for those who follow and are interested in the Islamic faith. The book includes interviews,profile, photography and features, including contributions from eminent Islamic scholars and Islamic VVIPs acrossthe world.

Useful LinksUPSC Papers: http://upscportal.com/civilservices/PAPERSUPSC Tips: http://upscportal.com/civilservices/TIPSFREE CHAT: http://upscportal.com/civilservices/CHATUPSC Toppers: http://upscportal.com/civilservices/TOPPERSUPSC Interviews: http://upscportal.com/civilservices/interviewsUPSC QUIZ: http://upscportal.com/civilservices/QUIZCurrent Affairs: http://upscportal.com/civilservices/current-affairsOnline Forum: http://upscportal.com/civilservices/forumUPSC Books: http://upscportal.com/civilservices/BOOKSFree NCERT Books: http://upscportal.com/civilservices/ncert_booksUPSC Syllabus: http://upscportal.com/civilservices/SyllabusUPSC Results: http://upscportal.com/civilservices/ResultsUPSC Coaching: http://upscportal.com/civilservices/coaching/civil-services-coaching-center-listGOVT Jobs: http://upscportal.com/civilservices/JOBSNotification: http://upscportal.com/civilservices/tag/ias-civil-services-india/notifications

Civil ServicesIAS: http://upscportal.com/civilservices/IASIES: http://upscportal.com/civilservices/IESIPS: http://upscportal.com/civilservices/IPSIFS: http://upscportal.com/civilservices/tag/upsc-services/IFSCDS: http://upscportal.com/civilservices/tag/upsc-services/cds-combined-defence-services-examinationSSC: http://upscportal.com/civilservices/tag/upsc-services/sscSCRA: http://upscportal.com/civilservices/tag/upsc-services/scra-special-class-railway-apprentices-examinationRRB: http://upscportal.com/civilservices/tag/upsc-services/rrb

WWW.UPSCPORTAL.COM 91

UPSCPORTAL Current Affairs : http://upscportal.com/civilservices/current-affairs

Integrated Guidance Programme for IAS (Pre) - 2012http://upscportal.com/civilservices/online-course/integrated-free-guidance-programme

SCRASpecial Class Railway

ApprenticesPractice Paper

WWW.UPSCPORTAL.COM 93

UPSCPORTAL Current Affairs : http://upscportal.com/civilservices/current-affairs

Integrated Guidance Programme for IAS (Pre) - 2012http://upscportal.com/civilservices/online-course/integrated-free-guidance-programme

GENERAL ABILITY TEST

SYNONYMS

Directions (Q. No. 1 to 15): Each of thefollowing 15 items consists of a word incapital letters, followed by four words.Select the word that is most similar inmeaning to the word written in capitalletters.

1. INSPIRE(a) Compel (b) Persuade(c) Infuse (d) Encourage

2. PROSCRIBE(a) Recommend (b) Allow(c) Advance (d) Banish

3. ABBREVIATE(a) Shorten (b) Enlarge(c) Decrease (d) Change

4. MULL(a) Recommend (b) Think(c) Punish (d) Provoke

5. VINDICTIVE(a) Strategic (b) Triumphant(c) Revengeful (d) Demonstrative

6. SURFEIT(a) Satiate (b) Pass(c) Confiscate (d) Delegate

7. ZANY(a) Magician (b) Pet(c) Thief (d) Clown

8. REGALE(a) Harass (b) Entertain(c) Express (d) Suppress

9. APPURTENANCE(a) Privilege (b) Journey(c) Concept (d) Necessity

10. SYCOPHANT(a) Suppliant (b) Follower(c) Flatterer (d) Admirer

11. PERNICKETY(a) Spiteful (b) Careless(c) Fussy (d) Ignorant

12. QUAINT(a) Old (b) Quiet(c) Haunted (d) Unusual

13. INVIDIOUS(a) Irritable (b) Harsh(c) Sinful (d) Unpopular

14. PUNCTILIOUS(a) Stupid (b) Wasteful(c) Meticulous (d) Timid

15. IMPOSTURE

SCRASpecial Class Railway Apprentices

Practice Paper

WWW.UPSCPORTAL.COM94

UPSCPORTAL Current Affairs : http://upscportal.com/civilservices/current-affairs

Integrated Guidance Programme for IAS (Pre) - 2012http://upscportal.com/civilservices/online-course/integrated-free-guidance-programme

(a) Claim (b) Status(c) Destruction (d) Deception

ANTONYMS

Directions (Q. No. 16 to 30) Each of thefollowing 15 items consists of a won incapital letters, followed by four words.Select the word which is closest to theopposite in meaning of the word written incapital letters.

16. BRIDGE(a) Divide (b) Blind(c) Release (d) Open

17. PLATONIC(a) Lunatic (b) Sublime(c) Major (d) Sensual

18. NADIR(a) Progress (b) Liberty(c) Zenith (d) Modernity

19. AVOIDANCE(a) Possession (b) Passion(c) Pursuit (d) Power

20. COMPLY(a) Refuse (b) Agree(c) Disobey (d) Deny

21. STRINGENT(a) Magnanimous (b) Lenient(c) Vehement (d) General

22. MITIGATE(a) Stagnate (b) Suffer(c) Aggravate (d) Instigate

23. GOBLIN(a) Angel (b) Hermit(c) Traveller (d) Pilot

24. LISSOME(a) Ungainly (b) Huge(c) Pungent (d) Crud

25. SPURIOUS(a) False (b) Genuine(c) Simple (d) Systematic

26. ZOOM(a) Soothe (b) Plurmet(c) Subjugate (d) Recite

27. TANGIBLE(a) Gentle (b) Eluive(c) Refined (d) Pabable

28. REPRIMAND(a) Reward (b) Appreciate(c) Encourage (d) Praise

29. SMOTHER(a) Repress (b) Cheer(c) Nurture (d) Irritate

30. BOISTEROUS(a) Serene (b) Tumultous(c) Brazen (d) Opaque

COMPREHENSION

Directions: This section you have fourshort passages. After each passage you willfind several questions based on thepassage.-First, read passage-I and answerthe questions based on it. Then go on tothe other passages.Examples ‘I’ and ‘J’ have been solved foryou.

PassageI our approach to life, be it pragmaticor other wise, a basic fact thatconfronts us squarely and unmatch-able is the desire for peace, securityand happiess. Different forms of lifeat different levels of existence makeup the teeming denizens of this earthof out And, no matter whether theybelong to the higher groups such ashuman beings or to the lower groupsuch as animals, all beings primarilyseek peace, comfort and security. Lifeis as dear to a mute craturas it is toa man. Even the lowliest insect strives4 protection against dangers thatthreaten its life. Justs each one of uswants to live and not to die, so do allother creatures.

I. The author’s main point is that(a) different forms of life are found on

earth

WWW.UPSCPORTAL.COM 95

UPSCPORTAL Current Affairs : http://upscportal.com/civilservices/current-affairs

Integrated Guidance Programme for IAS (Pre) - 2012http://upscportal.com/civilservices/online-course/integrated-free-guidance-programme

(b) differed levels of existence arepossible in nature

(c) peace ad security are the chiefgoals of all living beings

(d) even the weakest creaturestruggles to preserve its life

J. Which one f the following assumptionsor steps is essential in eveloping theauthor’s position ?(a) All form of life have a single

overriding goal(b) The will survive of a creature is

identified with a desire for peace(c) All being, are divided into higher

and lower groups(d) A paral lel; drawn between

happiness and life, and pain a3death

ExplanationI. The idea which represents the

author’s main point is ‘peace andseccurity are the chief goals of allliving beings’, which is response (c).So (c) is the correct answer.

J. The best assurtion underlying thepassage is ‘The will to survive of acreature is-identified with a desire forpeace, which is response (b). So (b)is the correct answer.

Passage – I

That artificial intelligence quotientshould seek to replace the time-testedI.Q. as a measure of mental ability isperfectly in consonance with thepresent day standards in a plasticsociety. However, the battle over greycells whether in human or mechanicalminds, whose latest round has foundUncle Sam shedding crocodile tearsover Japan’s failure to deliver on itspromise to produce a fifth generationcomputer, may find the Asian TigerCubs-The under-35 Japanese

researchers-having the last laugh. For,though all the boastful Tokyo talk adecade ago to build 1,000 processorcomputers to process knowledge-andnot merely numbers which is all theSilicon Valley Chips supposedly do-hasremained just talk, the 180 youngscientists in the 10-year venture havenevertheless made the big brains atSilicon Valley look rather silly with theirproduct which has a yen for logicalprogramming. The jubilation in theValley may turn to depression whenthe inexorable logic of thisdevelopment pulls down Washingtonfrom its pedestal of supercomputersupremacy.

31. Asian Tiger Cubs are(a) young Japanese researchers(b) mechanical minds(c) the big brains at Silicon Valley(d) fifth generation computers

32. Uncle Sam reacts to their failure with(a) sorrow (b) depression(c) jubilation (d) insincere sorrow

33. What have the cubs failed to produce?(a) Numbers processing computer(b) Grey cells(c) The fifth generation computer(d) A plastic society

34. What have they succeeded inproducing ?(a) Grey cells(b) A fifth generation computer(c) A knowledge processing computer(d) A product which has a yen for

logical programming35. How is their success likely to affect

Washington’s supremacy ?(a) It is likely to make it look silly(b) It is likely to dislodge it

WWW.UPSCPORTAL.COM96

UPSCPORTAL Current Affairs : http://upscportal.com/civilservices/current-affairs

Integrated Guidance Programme for IAS (Pre) - 2012http://upscportal.com/civilservices/online-course/integrated-free-guidance-programme

(c) It is likely to have the last laugh(d) It is likely to produce jubilation in

the Valley

Passage – II

It is to progress in the human sciencesthat we must look to undo the evilswhich have resulted from a knowledgeof the physical world hastily andsuperficially acquired by populationsunconscious of the changes inthemselves that the new knowledgehas made imperative. The road to ahappier world than any known in thepast lies open before us if atavisticdestructive passions can be kept inleash while the necessary adaptationsare made. Fears are inevitable in ourtime, but hopes are equally rationaland for more likely to bear good fruit.We must learn to think rather less ofthe dangers to be avoided. than ofthe good that will lie within our graspif we can believe in it and let itdominate our thoughts. Science,whatever unpleasant consequences itmay have by the way, is in its verynature a liberator, a liberator ofbondage to physical nature and en tocome, a liberator from the weight ofdestructive passions. We are on thethreshold of utter disaster orunprecedentedly gloriousachievement. No. previous age hasbeen fraught with problems somomentous; and it is to science thatwe must look to for a happy future.

36. What does science liberate us from ?It liberates us from(a) bondage to physical nature(b) fears and destructive passions(c) idealistic hopes of a glorious future(d) slavery to physical nature and from

passions

37. To carve out a bright future a manshould(a) try to avoid dangers(b) overcome fears and dangers(c) cultivate a positive outlook(d) analyse dangers that lie ahead

38. If man’s bestial yearning is controlled(a) the present will be brighter than

the future(b) the present will become tolerant(c) the future will be brighter than the

present(d) the future will be tolerant

39. Pears and hopes, according to theauthor(a) can yield good results(b) can bear fruit(c) are irrational(d) are closely linked with the life of

modem man40. Should human sciences be developed

because they will :(a) eliminate the destruction caused

by a superficial knowledge of thephysical world

(b) make us conscious of the changesin ourselves

(c) make us conscious of the changingworld

(d) provide more knowledge of thephysical world

Passage – III

As heart disease continues to be thenumber one killer in the United States,researchers have become increasinglyinterested in identifying the potentialrisk factors that trigger heart attacks.High-fat diets and “life in the fast lane”have long been known to contributeto the high incidence of heart failure.But according to new studies, the listof risk factors may be significantlylonger and quite surprising.Heart failure, for example, appears to

WWW.UPSCPORTAL.COM 97

UPSCPORTAL Current Affairs : http://upscportal.com/civilservices/current-affairs

Integrated Guidance Programme for IAS (Pre) - 2012http://upscportal.com/civilservices/online-course/integrated-free-guidance-programme

have seasonal and temporal patterns.A higher percentage of heart attacksoccurs in cold weather and morepeople experience heart failure onMonday than on any other day of theweek. In addition, people are moresusceptible to heart attacks in thefirst few hour after waking.Cardiologists first observed themorning phenomenon in the mid-1980and have sinc discovered a numberof possible causes. A earlymorning risein blood pressure, heart rate ailconcentrating of heart stimulatinghormones, plug reduction of blood flowto the heart, may all contribute tothe higher incidence of heart attacksbetween the hours of 8.00 A.M. and10.00 A.M.In other studies, both birthday andbachelorhood have been implicated asrisk factors. Statistics Veal that heartattack rates increase significantly forboth females and males in the fewdays immedetely preceding andfollowing their birthdays.And unmarried men are more at riskfor heart attack than their marriedcounterparts. Though stress is foughtto be linked in some way to all of theaforemecioned risk factors, intenseresearch continues in the hope offuture comprehending why and howheart ilure is triggered.

41. What does the passage mainlydiscuss?(a) cardiology(b) diet and stress as factors in heart

attack(c) seasonal and temporal patterns of

hear attacks(d) risk factors in heart attacks

42. The word ‘potential’ could best be

replace by which of the following ?(a) harmful (b) possible(c) unknown (d) primary

43. According to the passage, which ofthe following is not a possible causeof many heart attars ?(a) decreased blood flow to the heart(b) increased blood pressure(c) lower heart rate(d) increase in hormones

44. Which of the following is cited aspossible risk factor?(a) getting married(b) rainy wather(c) eating fatty foods(d) driven fast

45. Which of the following does the pageinfer?(a) We now fully understand how he

risk factors trigger heart attacks(b) We do not. fully understand he the

risk factors trigger heart attacks(c) We have not identified risk facts

associated with heart attacks(d) We have recently begun to stuff

how risk factorswork

Passage – IV

I do not suggest that the cultural sideof education should be ignored. Onthe contrary, I think, it is essential tothe production of the sort of adultwho best fits the modern world. But Ithink that what is important in culturaleducation should be conveyed, at anyrate, in the early stages, by methodsfar more attractive than those nowused. History and Geography shouldbe taught at first by means of thecinema. When taught this way, theywill give pleasure; attention will bespontaneous and therefore theimpression will be less temporary. Inspite of reforming movements, there

WWW.UPSCPORTAL.COM98

UPSCPORTAL Current Affairs : http://upscportal.com/civilservices/current-affairs

Integrated Guidance Programme for IAS (Pre) - 2012http://upscportal.com/civilservices/online-course/integrated-free-guidance-programme

is still among educators a feeling thatwhat is enjoyed without effort cannothave much educational value. I wouldhave children made aware of themanners and customs of tribes andnations utterly remore from their own.Education conducted on these lineswould do more than many books tocure provincialism in space and timeand to make children realize thatactual human beings with actualfeel ings can be outwardly verydifferent from the people among whomthey live, but inwardly composed ofthe same human material. A Zulu wouldnot appear strange, remote or savagebut one like themselves.

46. The writer said that the cultural sideof education is needed to make man(a) admire his environment(b) suitable for modern civilization(c) appreciate other civilizations(d) appreciate one’s heritage

47. The writer criticized traditionaleducators for not(a) imparting information about

various’ cultures(b) imparting information about remote

tribes.(c) making learning an enjoyable

activity(d) using cinema as a method of

teaching48. The response of the learners to

teaching through cinema is(a) superficial (b) enthusiastic(c) long-lasting (d) absorbing

49. The expression “to cure provincialismin space and time” means to get ridof(a) selfish opinion(b) narrow views(c) obscure ideas (d) regional feelings

50. One of the goals of education shouldbe to make children realise that(a) there is a unified system of human

values underneath culturaldiversity

(b) they must respect values of alienculture

(c) regional cultures should not beignored

(d) human culture is diverse

ORDERING OF WORDS IN A SENTENCE

Directions: In the following items someparts of the sentence have been jumbledup. You are required to rearrange theseparts which are labelled P, Q, R and S toproduce the correct sentence. Choose theproper sequence and mark on your answersheet accordingly. Example ‘Z’ has beensolved for you.

Z. It is well-known thatthe effect is very bad (P) (Q)on children of cinema (R) (S)The proper sequence should be(a) P S R Q (b) S P Q R(b) S R P Q (d) Q S R P

ExplanationThe proper way of writ ing thesentence is ‘It is well known that theeffect of cinema on children is verybad.’ This is indicated by the sequencePSRQ and so (a) is the correct answer.

51. Alexander was a disciple of Aristotle(P)

who was a great conqueror(Q)

whom the world acknowledges thegreatest philosopher (R)

WWW.UPSCPORTAL.COM 99

UPSCPORTAL Current Affairs : http://upscportal.com/civilservices/current-affairs

Integrated Guidance Programme for IAS (Pre) - 2012http://upscportal.com/civilservices/online-course/integrated-free-guidance-programme

the world has ever known(S)

The proper sequence should be:(a) P Q S R (b) Q P R S(c) R P Q S (d) S P Q R

52. Jawaharlal Nehruunder the Cabinet Mission Scheme

(P)was the first to articulate

(Q)long before such an assembly was set up

(R)the idea of a Constituent Assembly

(S)The proper sequence should be(a) P Q S R (b) Q S R P(c) R P Q S (d) S R P Q

53. The man is generally the one(P)

who can work very hard(Q)

when he must work(R)

who can play most heartily when hehas the chance of playing

(S)The proper sequence should be(a) P S Q R (b) Q R P S(c) Q R SP (d) S P Q R

54. The reporter of the newspaper informsus that Lala Lajpat Rai spent

(P)lecturing widely and attracting manyAmerican leaders

(Q)to sympathise with the Indiannationalist position (R)almost five years in America

(S)The proper sequence should be

(a) PSQR (b) PSRQ(c) RPSQ (d) SPQR a

55. The practice of taking performanceboosting drugs among athletesbut checking it is not going to be easy

(P)is generally conceded to be unfair

(Q)of the detection technology

(R)for the user is generally one jumpahead (S)The proper sequence should be(a) Q P R S (b) Q P S R(c) R Q P S (d) R Q S P

56. As lightningaccompanies thunderwas mingled with (P)so in my character (Q)the mutterings of my wrath

(R)a flash of humour (S)The proper sequence should be(a) P R S Q (b) Q P R S(c) Q R P S (d) Q S P R

57. During hibernationthey have lost a great deal of weight

(P)of course the animals consume

(Q)and when they awake again

(R)the fats accumulated in the summer

(S)The proper sequence should be(a) P Q R S (b) Q S R P(c) R Q S P (d) S Q P R

58. The house

WWW.UPSCPORTAL.COM100

UPSCPORTAL Current Affairs : http://upscportal.com/civilservices/current-affairs

Integrated Guidance Programme for IAS (Pre) - 2012http://upscportal.com/civilservices/online-course/integrated-free-guidance-programme

As lightningaccompanies thunder(P)

that stands in front of us(Q)

was built of stones(R)

which were dug out of its own site(S)

The proper sequence should be(a) Q P R S (b) Q R P S(c) Q R S P (d) Q S P R

59. The administeration will soon launchto educate the people about theirrights

(P)a consumer protection drive

(Q)against using fraudulent means

(R)and to warn traders

(S)The proper sequence should be(a) P Q S R (b) P S Q R(c) Q P S R (d) S P Q R

60. Some of our social sciencesthat cannot be attacked by modemsociety(P)such as psychology and sociology

(Q) seem to have become

(R)sacred cows (S)The proper sequence should be(a) P Q R S (b) Q R S P(c) R S P Q (d) R S Q P

Directions: The following items consist oftwo statements one label led as theAssertion (A) and the other as Reason (R).You are to examine these two statements

carefully and select the answer of theseitems using the codes given below

Codes :(A) Both A and R are individually true and

R is the correct explanation of A.(B) Both A and R are individually true but

R is not thecorrect explanation of A.(C) A is true but R is false.(D) A is false but R is frue.61. Assertion (A): Subash Chandra Bose

resigned from the presidentship of theIndian National Congress in his secondterm.Reason (R): He opposed to theGandhian Movement based on non-violence.(a)A (b) B (c) C (d) D

62. Rice is the product of(a) tropical region(b) subtropical region(c) temperate region(d) none of the above

63. Where is Pir Panjal range of mountainssituated ?(a) Uttar Pradesh(b) Assam(c) Jammu and Kashmir(d) West Bengal

64. Match the followingList-I List-II

A. World driest place1. 65.33B. Literates in India 2. Calama,

(As of desert Atacama population) percentage

of desertC. Bhilai 3. Iron and

steelindustry

D. Kanchipuram 4. Silk industryCodes:

A B C D(a) 2 1 3 4

WWW.UPSCPORTAL.COM 101

UPSCPORTAL Current Affairs : http://upscportal.com/civilservices/current-affairs

Integrated Guidance Programme for IAS (Pre) - 2012http://upscportal.com/civilservices/online-course/integrated-free-guidance-programme

(b) 3 1 2 4(c) 4 1 2 3(d) 2 3 4 1

65. What is a cloud ?(a) It is a kind of mist(b) It is a kind of frost(c) It is a kind of condensed form of

dew(d) It is a kind of fog lifted above the

earth surface66. About 85% of the raw silk produced

in India comes from :(a) Kerala (b) M.P.(c) Maharashtra (d) Karnataka

67. The term secular was added in thepreamble to the Indian Constitutionby(a) 41st Amendment(b) 42nd Amendment(c) 43rd Amendment(d) 44th Amendment

68. Who finally approves the draft of fiveYear Plan ?(a) Union Cabinet(b) President(c) National Development Council(d) Parliament and State Legislatures

69. Which country is the latest to jointhe U.N. as a member ?(a) Switzerland(b) Hong Kong(c) Taiwan(d) E. Timor

70. In which mode of expression, theconcentration of a solution remainsindependent of temperature ?(a) Molality (b) Normality(c) Formality (d) Molarity

71. Oxidation number of Fe in Fe3O4 is(a) 1/2 (b) 2/6(c) 8/3 (d) 3/2

72. The molecule of ortho hydrogen is

distinguished from para hydrogen by(a) two electrons moving in opposite

directions(b) two electrons moving in the same

directions(c) two protons revolving in the

opposite directions(d) two protons revolving in the same

directions73. Sodium cannot exhibit +2 oxidation

state because its(a) IE2 valuee is less than IE1 value(b) IE2 value is higher than IE1 value(c) IE2 value and IE1 value are equal(d) IE2 value is zero

74. The conjugate base of a strong acidis(a) strong base (b) strong acid(c) weak acid (d) weak base

75. An element with atomic number 84and mass number 218 loses one a-particle and two b-particle in the threesuccessive stages. The resultingelement will have(a) At. no. 84 and mass no. 214(b) At. no. 82 and mass no. 218(c) At. no. 84 and mass no. 218(d) At. no. 82 and mass no. 214

76. What is the percentage (%) of H2O inFe(CNS)3×3H2O ?(a) 45 (b) 30(c) 19 (d) 25

77. The Saka era commencing from AD78 was founded by(a) Kanishka(b) Asoka(c) Chandragupta(d) Vikramaditya

78. Which ‘sufi-saint’ dargah is situatedat Ajmer ?(a) Baba Farid(b) Qutub-din-Bakhtiyar Kaka(c) Moinuddin Chisti

WWW.UPSCPORTAL.COM102

UPSCPORTAL Current Affairs : http://upscportal.com/civilservices/current-affairs

Integrated Guidance Programme for IAS (Pre) - 2012http://upscportal.com/civilservices/online-course/integrated-free-guidance-programme

(d) Khwaja Bahuddin79. Who was the architect who designed

“Taj Mahal” ?(a) Mohammad Hussain(b) Ustad Isa(c) Shah Abbas(d) Ismail

80. Which of the following States has thelargest concentration of tea plantationin India ?(a) Assam(b) Bihar(c) Meghalaya(d) Arunachal Pradesh

81. Among the following rivers which riverknown was as ‘Arun river’ in thebeginning?(a) Sone (b) Kossi(c) Chambal (d) Gomti

82. Which place is called as ‘Sangam’ ofBhagirathi and Alaknada ?(a) Dev Prayag (b) Rudra Prayag(c) Chambal (d) Vishnu Prayag

83. Din-i-illahi of Akbar was not a successbecause:(a) After Akbar it was not patronised(b) the muslim found it difficult to

accept other religions(c) it was not suitably projected to

the masses(d) all of the above reasons

84. Study the following map and matchthe list-I (R, S, T U, V show themountain) list-II (their names)

INDIA

List-I List-IIA. R 1. Rajmahal HillB. S 2. Vindhya ParvatC. T 3. Maikal HillD. U 4. Satpuda MountainE. V 5. Nilgiri

Codes:A B C D E

(a) 3 5 1 2 4(b) 1 5 3 4 2(c) 4 3 2 1 5(d) 5 1 4 3 2

85. In this map 1, 2, 3 and 4 indicatethat the correct sequence of Indianrivers

INDIA

WWW.UPSCPORTAL.COM 103

UPSCPORTAL Current Affairs : http://upscportal.com/civilservices/current-affairs

Integrated Guidance Programme for IAS (Pre) - 2012http://upscportal.com/civilservices/online-course/integrated-free-guidance-programme

(a) Vaigai, Cauvery, Tungbhadra andGodavari

(b) Mahanadi, Godavari, Krishna, andCavery

(c) Cauvery, Krishna, Venganga andDamodar

(d) Cauvery, Tungbhadra, Godavariand Mahanadi

86. In this map 1, 2, 3 and 4 indicatethat group of cities is:

WORLD

(a) Washington, Paris, Stockholm,Kabul

(b) New York, Madrid, Moscow,Tehran, Basra

(c) Tehran, Washington, Paris, Moscow(d) New York, Rome, Berlin, Tehran

87. The number of floating ribs in thehuman body is(a) 6 pairs (b) 5 pairs(c) 3 pairs (d) 2 pairs

88. Fish eradicate the mosquito larvae is:(a) Cuttler fish (b) Anabus(c) Gambusia (d) Rohu

89. A pair of insectivorous plant is:(a) Venus fly trap and Rafflesia(b) Nepenthes and bladderwort(c) Drosera and Rafflesia(d) Dionaea and Viscum

90. Consider the following organs:1. Cerebrum 2. Diencephalon3. Henle of loop 4. NephronThe organs related to brain are(a) 1 and 3 (b) l, 2 and 4

(c) 1 and 2 (d) 1, 2, 3 and 491. The smallest bird is:

(a) humming bird (b) rhea(c) kiwi (d) cock

92. Which of the following diseases spreadthrough insects ?1. Gambian fever2. Chagga’s disease3. Cholera 4. Oriental sore(a) 1, 2 and 3 (b) 1, 3 and 4(c) 2, 3 and 4 (d) 1, 2, 3 and 4

93. The child shown in the figure issuffering from some malnutrition. Thename of the disorder related to this

(a) obesity (b) kawashiorkar(c) myxoedema (d) marasmus

94. Which of the following hormones issecreted by pituitary gland ?1. STH 2. FSH3. Oxytocin 4. PTH(a) 1 and 2 (b) 3 and 4(c) 1, 2 and 3 (d) 1, 2, 3 and 4

95. Consider the following statements:1. Rods are more in eyes of diurnal

animals.2. Cones are more in eyes of

nocturnal animals.Correct statement(s) is/are(a) only 1(b) only 2(c) neither 1 nor 2(d) both 1 and 2

96. Fat soluble vitamin(s) is/are1. Pantothenic acid2. Tocopherol

WWW.UPSCPORTAL.COM104

UPSCPORTAL Current Affairs : http://upscportal.com/civilservices/current-affairs

Integrated Guidance Programme for IAS (Pre) - 2012http://upscportal.com/civilservices/online-course/integrated-free-guidance-programme

3. Pyridoxine4. Folic acid(a) only 1 (b) 1, 2 and 3(c) only 2 (d) 1, 2, 3 and 4

97. Consider the following statements :1. Lungs are found in all vertebrates

except birds.2. Book lungus are respiratory organs

in scorpion.3. Aquatic animals like prawns, fishes

and tadpole respire in water withgills.

4. Cutaneous respiration is notpresent in earthworm.

Incorrect statements are:(a) 1 and 2 (b) 2 and 4(c) 1 and 3 (d) 1 and 4

98. The countries included in ‘Plus three”of ASEAN, are :1. Northern Koria2. Bharat3. South Koria4. China5. JapanCodes:(a) 3, 4, 5 (b) 1, 2, 3(c) 2, 4, 5 (d) 1, 4, 5

99. Which of the following is mosturbanised State of India ?(a) Maharashtra (b) Mizoram(c) Goa (d) Tamil Nadu

100.Which State tourism department’smotto is “Gods own country” ?(a) Tamil Nadu(b) Himachal Pradesh(c) Assam (d) Kerala

101.All the following individuals havesomething common, what is that ?1. John Kennedy Jr.2. Zia-ul-Haq3. Dag Hammas Skjoeld(a) All were sons of famous politician

(b) All died at young age(c) All died in the plane crash(d) All died in war

102.Chipko Movememt(a) is originated in the Garhwal region

of Western Himalayas(b) is related to environment(c) is related to mass education(d) both ‘a’ and ‘b’ are true

103.The line of map connecting points ofequal temperature at a given time is(a) isohyet (b) isobar(c) isthmus (d) isotherm

104.Arrange the following rivers from southto north1. Cauvery 2. Krishna3. Godavari 4. MahanadiCodes:(a) 1 , 3, 2, 4 (b) 3, 4, 1, 2(c) 2, 4, 3, 1 (d) 1, 2, 3, 4

105.“Kuchipudi” dance was originated by(a) Andhra Pradesh(b) Karnataka(c) Punjab (d) Rajasthan

106.No money bill can be introduced inthe Lok Sabha without the approvalof the(a) Vice President(b) President(c) Prime Minister(d) Finance Minister

107.Which of the Mughal emperors was awell known poet ?(a) Akbar(b) Shahjahan(c) Humayun(d) Bahadurshah Jafar

108.When a mango is kept in dilute aqueoussolution of HCl, it(a) swells (b) bursts(c) shrinks (d) none of these

WWW.UPSCPORTAL.COM 105

UPSCPORTAL Current Affairs : http://upscportal.com/civilservices/current-affairs

Integrated Guidance Programme for IAS (Pre) - 2012http://upscportal.com/civilservices/online-course/integrated-free-guidance-programme

109.Arrange H2SO4(I), H3PO4(II) andHClO4(III)in decreasing order of acidicstrength(a) I > III > II (b) I > II > III(c) III > II > I (d) III > I > II

110.Which of the following is a chemicalchange?(a) Melting of sodium in nuclear reactor(b) Melting of steel(c) Sublimation of iodine(d) Explosion of dynamite

111.The composition of Duralumin is(a) Al = 94%, Mg = 6%(b) Cu = 56%, Zn = 24%, Ni = 20%(c) Cu = 95%, Al = 5%(d) Al = 95%, Cu = 4%, Mn = 0.5%,

Mg = 0.5%112.Which one of the following pairs is

correcty matched ?(a) Electron - J.J. Thomson(b) Charge on electron - James

Chadwick(c) Nucleus - Millikan(d) Neutron - Rutherford

113.Which one of the following is thecommon glass ?(a) K2SiO3. CaSiO3.4SiO2(b) Na2SiO3. CaSiO3.4SiO2(c) K2SiO3. PbSiO3.4SiO2(d) Na2SiO3

114.Which one of the following is theanhydride of sulphurous acid ?(a) H2SO4 (b) SO2(c) SO3 (d) H2S

115.Match list-I with list-II and select thecorrect answer using the codes givenbelow the lists:

List-I List-II(Chemical) (Formulae)

A. Washing soda1. Pb(OH)2. 2PbCO3B. Lime stone 2. Pb3O4

C. Red lead 3. Na2CO3.10H2OD. White lead 4. CaCO3

Codes:A B C D

(a) 1 2 3 4(b) 3 4 2 1(c) 4 3 2 1(d) 4 3 1 2

116.Which of the following statements aretrue regarding graphite ?1. It is the crystalline allotrope of

carbon.2. C—C bond distance is greater than

diamond.3. The hybridisation of carbon in

graphite is sp3.4. Graphite is the good conductor of

heat and electricity.Which of these statements arecorrect?(a) 1 and 2 (b) 3 and 4(c) 1 and 4 (d) 2 and 4

117.Heavy water is a compound of :(a) hydrogen and heavier isotope of

oxygen(b) heavier isotopes of hydrogen and

oxygen(c) oxygen and heavier isotope of

hydrogen(d) none of the above

118.Which general was responsible forJallianwala Bagh tragedy ?(a) General Dalhousie(b) General Dyer(c) General Mountbatten(d) General William Bentinck

119.Which is the southrnmost place in theIndian union ?(a) Kanyakumari (b) Rameshwaram(c) Indira Point (d) Vivekanand

120.Who is legally competent under theIndian Constitution to declare war or

WWW.UPSCPORTAL.COM106

UPSCPORTAL Current Affairs : http://upscportal.com/civilservices/current-affairs

Integrated Guidance Programme for IAS (Pre) - 2012http://upscportal.com/civilservices/online-course/integrated-free-guidance-programme

conclude peace ?(a) Prime Minister(b) The President(c) The Parliament(d) The Council of Ministers

1. Which of the following statement isnot correct about vinyl chloride ?(a) It can be prepared by addition of

hydrogen chloride to ethyne.(b) It can be co-polymerised with vinyl

acetate to give strong adhesivecopolymers suitable for surfacecoatings.

(c) It polymerises to give P.V.C. athermoplastic polymer.

(d) It is more reactive towardsnucleophiles than ethyl chloride.

2. Which one of the following is knownas westrosol ?(a) Acetylene tetrachloride(b) Acetylene dichloride(c) Trichloroethylene(d) Allyl chloride.

3. The compound with no dipole momentis(a) methyl chloride.(b) carbon tetrachloride.(c) methylene chloride.

PHYSICAL SCIENCE

(d) chloroform.4. If chloroform is left open in air in the

presence of sun rays,(a) explosion takes place.(b) poisonous gas phosgene is formed.(c) polymerisation takes place.(d) No reaction takes place.

5. Which of the following statements isnot correct about the haloalkanes ?(a) They are derived from alkenes by

replacement of one of its hydrogenatoms by halogens.

(b) They can be prepared by theaddition of hydrogen halides toalkenes.

(c) They undergo nucleophil icsubstitution reactions in which thehalogen atom is replaced byanother atom or group.

(d) They can be reduced with lithiumaluminium hydride to give thecorresponding alkanes.

6. Which of the following statements isincorrect about the alkyl halides ?

(a) They belong to the homogolousseries of general formulaCnH2n+1 X where X = Cl, Br or Iand n = 1, 2, 3 etc.

(b) They can be prepared by thereaction of the hydrogen halide andthe appropriate alcohol e.g.ROH + HX ® RX + H2O.

(c) They show reactivity in the orderRCl > RBr > RI where R = alkylgroup.

(d) They form nitrites when refluxedwith potassium cyanide in ethanol.

7. Which of the following statements isnot correct about chloroform ?(a) It is a highly inflammable liquid.(b) It is a colourless, sweet-smelling

liquid almost insoluble in water.(c) It can be used as an anaesthetic.(d) It can be prepared by the haloform

reaction.8. Freon 114 is an organic substance

used in refrigerators and airconditioners. It is chemically called 1,2 dichloro tetrafluoro ethane. Its

WWW.UPSCPORTAL.COM 107

UPSCPORTAL Current Affairs : http://upscportal.com/civilservices/current-affairs

Integrated Guidance Programme for IAS (Pre) - 2012http://upscportal.com/civilservices/online-course/integrated-free-guidance-programme

correct formula is

(a)

F F| |

Cl — C — C — H| |Cl Cl

(b)

F F| |

H — C — C — H| |Cl Cl

(c)

CI F| |

F — C — C — CI| |F F

(d)

F Cl F| | |

F — C — C — C — F| | |CI H F

9. When a solution of sodium chloride,containing ethyl alcohol is eletrolysed,it forms(a) ethyl alcohol. (b) chloral.(c) chloroform. (d) acetaldehyde.

10. When chlorobenzene is heated withchloral and cone. H2SO4 which one ofthe following compounds is formed ?(a) Phenol (b) Benzoic acid(c) Polythene (d) D.D.T.

11. 1, 2-Dichloropropane is refluxed undera water-cooled condenser for sometime with sodium hydroxide solution.The product is(a) propionaldehyde.(b) propanol.(c) propionic acid.(d) 1-chloro-l-hydroxypropane.

12. A compound was refluxed with excessKOH solution for an hour. Afteracidification with nitric acid, silvernitrate was added. No precipitate,appeared. Water compound couldhave been(a) CH3CH2I (b) CH3CH2CH2Br(c) CH3COOl (d) C6H5Cl

13. When chloroform is treated with aprimary amine and KOH we get(a) rose odour smell.(b) sour almond smell.(c) bad nectar-like smell.(d) sour oil of winter green like smell.

14. A sample of chloroform being used asan anaesthetic is tested by(a) Fehling solution.(b) ammoniacal CuCl.(c) AgNO3 solution.(d) AgNO3 solution after boiling with

alk KOH.15. The reaction least likely to occur is

(a) + HNO3

NO2

H SO2 4

(b) + Cl2

Cl

ultravioletlight

(c) + H SO2 4

SO H2

(d) + Br2

BrFe

16. Which of the following is the correctformula for dichlorodiphenyltrichloroe­thane ?

WWW.UPSCPORTAL.COM108

UPSCPORTAL Current Affairs : http://upscportal.com/civilservices/current-affairs

Integrated Guidance Programme for IAS (Pre) - 2012http://upscportal.com/civilservices/online-course/integrated-free-guidance-programme

(a) Cl— —C—C—Cl

Cl

Cl

H Cl

(b) —C—C—Cl

Cl

Cl

H Cl

(c) Cl— —C—C—Cl

Cl

Cl

H Cl

(d) — —C—C—Cl

Cl

Cl

Cl Cl

17. An attempt was made to prepare ethyliodide by gently warming methylatedspirit, potassium iodide crystals andconcentrated sulphuric acid, no ethyliodide was produced because(a) the mixture should not have been

warmed.(b) the alcohol was dehydrated to

ethylene.(c) methyl iodide was produced since

methylated spirit is mainly methylalcohol.

(d) the potassium iodide was oxidisedto iodine, which does not reactwith alcohol under theseconditions.

18. When glycerol is heated with oxalicacid at 260°C, it gives(a) formic acid.(b) allyl alcohol.(c) glyceryl oxalate.(d) alkyl alcohol.

19. When glycerol is heated with oxalic

acid at 170°C, it yields(a) HCOOH.(b) HCOOH and allyl alcohol.(c) tartaric acid.(d) allyl alcohol.

20. Which of the following is the structureof cellosolve ?(a) ClH5 —O—CH2—CH2—O—CH2

—CH2—OH(b) CH3—O—CH2—CH2—O—CH2—

CH2—OCH3(c) CH3CH2O—CH2CH2OH(d) None of these.

21. In cold countries glycerol is added tothe water in the radiator of carsduring winter. This helps to(a) bring down the specific heat of

water.(b) lower the freezing point.(c) reduce the viscosity.(d) make water a better lubricant.

22. Dynamite is obtained by mixingnitroglycerine with(a) saw dust and NH 4NO3.(b) cellulose nitrate.(c) cellulose nitrate and vaseline.(d) T.N.T.

23. Diketopiperazine is obtained by heating(a) methylamine.(b) glycine.(c) propionic acid.(d) ammonium acetate.

24. When ethyl alcohol is heated withconc. H2SO4 at 140°C, the productobtained is(a) ethylene.(b) diethyl ether.(c) diethyl sulphate.(d) ethyl hydrogen sulphate.

25. Ethyl alcohol can be distinguished frommethyl alcohol by(a) action of Cl2.

WWW.UPSCPORTAL.COM 109

UPSCPORTAL Current Affairs : http://upscportal.com/civilservices/current-affairs

Integrated Guidance Programme for IAS (Pre) - 2012http://upscportal.com/civilservices/online-course/integrated-free-guidance-programme

(b) action of NH3.(c) determining their solubilities in H2O.(d) iodoform test.

26. Which of the following statements isincorrect about ethylene glycol ?(a) It is a colourless liquid b.p. 197°C,

used as an ‘antifreeze’.(b) It is a dihydric alcohol.(c) It forms two series of esters, ethers

and halides.(d) It reacts with sodium to give the

di­sodium salt.27. Which of the following statement is

not true ?(a) Oxidation of a primary alcohol with

chromic acid (Na2Cr2O7 andH2SO4) yields an aldehyde as thefirst oxidation product.

(b) Oxidation of a primary alcohol withpotassium permanganate yields acarboxylic acid.

(c) Oxidation of secondary alcohol withchromic acid yields a. ketone.

(d) A secondary alcohol with chromicacids yields the correspondingcarboxylic acid.

28. Which of the following reacts withLucas reagent most readily ?(a) CH3—CH2—Cl(b) CH3—CH2—CH(c) CH2 = CH2CH3

(d)

3

3

3

CH|

CH — C — OH|CH

29. Which of the following products isobtained by heating glycerine withBa(OH)2 ?(a) Methylamine(b) Acetamide

(c) Acetic acid(d) Barium acetate.

30. Which product is obtained whenglycerol is heated with potassiumbisulphate ?(a) Acrolein(b) Allyl alcohol(c) Acetic acid(d) None of these.

31. When a mixture of ethanol andmethanol are heated in the presenceof concentrated H2SO4the organicproduct(s) is (are)(a) CH3OC2H5(b) CH3OCH3 and C2H5OC2H5(c) CH3OC2H3 and CH3OCH3(d) CH3OC2H5, CH3OCH3, and

C2H5OC2H5

32. The hydrolysis of stearin producesstearic acid and(a) sodium hydroxide.(b) glycerol.(c) hydrochloric acid.(d) propyl alcohol.

33. Primary alcohols cannot be preparedby(a) hydration of alkenes in the

presence of acid.(b) reduction of ketones with zinc and

hydrochloric acid.(c) hydrolysis of alkyl halides with

aqueous alkali.(d) hydrolysis of ethers in strongly

acidic conditions.34. Rectified spirit is a mixture of

(a) 95% ethyl alcohol +5% water.(b) 94% ethyl alcohol +4.53% water.(c) 94.47% ethyl alcohol +5.43%

water.(d) 95.57% ethyl alcohol +4.43%

water.35. Lucas reagent for identification of

alcohol is(a) P4, I2 and AgNO2